Вы находитесь на странице: 1из 175

Final Year OSCE Guide

(Based on last 5 years)

Contents
Ctrl + click to go to relevant section
1 COUNSELLING STATIONS.........................................................................................................................................................3 1.1 MRSA INFO GIVING (2009) ............................................................................................................................................................3 1.2 EXPLAIN PEAK FLOW AND TTO (2009 (TTO 2006))........................................................................................................................5 1.3 SURGICAL OPTIONS FOR AAA (2005, 2009).....................................................................................................................................8 1.4 SMOKING CESSATION (2005 & 2009)...............................................................................................................................................9 1.5 COMPLAINTS PROCEDURE (2005, 2007, 2008, 2009)......................................................................................................................11 1.6 BREAST CANCER HX (PAIR WITH COUNSELLING) (2008).....................................................................................................................13 1.7 BREAST CANCER COUNSELLING (PAIR WITH BREAST CA HISTORY) (2008)..............................................................................................14 1.8 EPILEPSY EXPLANATION (2006, 2008)............................................................................................................................................16 1.9 POST MI & INFORMED CONSENT (2008)..........................................................................................................................................18 1.10 EXPLAIN POST MI DRUGS & TTO (2008)....................................................................................................................................20 1.11 BREAKING BAD NEWS (2005).......................................................................................................................................................22 1.12 HYPERTENSION MED NON ADHERENCE (12 MIN).......................................................................................................................................23 1.13 DIABETES COUNSELLING TYPE 1 (6 MIN).....................................................................................................................................26 1.14 TYPE II DIABETES........................................................................................................................................................................30 1.15 DO NOT RESUSCITATE (2005).....................................................................................................................................................33 1.16 INHALER TECHNIQUE NEWLY DIAGNOSED ASTHMATIC (2007).............................................................................................................35 1.17 EXPLAIN GASTROSCOPY/SIGMOIDOSCOPY PROCEDURE (2006; 12 MIN)...............................................................................................39 1.18 EXPLAIN SIGMOIDOSCOPY RESULTS = CROHNS DISEASE (2007 - 6 MIN).............................................................................................42 1.19 GORD (2006, 12 MIN).............................................................................................................................................................44 1.20 EXPLAIN AF AND INITIATE WARFARIN THERAPY (2009) .................................................................................................................46 1.21 CYSTIC FIBROSIS COUNSELLING (2006) .........................................................................................................................................48 2 CLINICAL SKILLS......................................................................................................................................................................50 2.1 BLOOD GAS & INTERPRETATION (2008, 2009)................................................................................................................................50 2.2 BLOOD CULTURES, INTERPRET, PRESCRIBE ANTIBIOTICS (2005, 2009)..................................................................................................53 2.3 MANAGING HANDOVER & PRIORITISING............................................................................................................................................55 2.4 PRESCRIBING IN RENAL FAILURE (2009)...........................................................................................................................................56 2.5 CANNULATION AND FLUID MANAGEMENT (2008, 2009).....................................................................................................................58 2.6 PRE-OP ASSESSMENT (2006)...........................................................................................................................................................62 2.7 INTERPRET BLOOD TEST RESULTS ADDISONS (2007) .......................................................................................................................64 2.8 CREMATION FORM (2005, 2007) ...................................................................................................................................................65 2.9 TTO FORM TRANSCRIBE FROM DRUG CHART (2006)..........................................................................................................................66 2.10 CATHETERISATION.......................................................................................................................................................................67 2.11 VIDEO OF WARD ROUND WRITE NOTES (2006, 2008) ......................................................................................................................68 2.12 NEEDLESTICK INJURY...................................................................................................................................................................69 2.13 ADVERSE REACTION TO BLOOD TRANSFUSION...................................................................................................................................71 2.14 BLOOD TRANSFUSION: EXPLAIN , TAKE BLOOD, FILL IN FORM (2006, 2008).........................................................................................72 ECG -PERFORM AND INTERPRET II.......................................................................................................................................................74 2.14.1 Atrial Fibrillation potential examiners questions........................................................................................................78 2.15 URINALYSIS, PRESCRIBING (PENICILLIN ALLERGY) ............................................................................................................................80 2.16 ALS (2005, 2007)...................................................................................................................................................................81 2.17 ANAPHYLACTIC DRUG REACTION, TREAT & BLS (2007).................................................................................................................83 2.18 DRUG CHART POST GI BLEED (2008)............................................................................................................................................84 2.19 CONFIRM DEATH AND DOCUMENT (2008).......................................................................................................................................86 2.20 CHEST X-RAY & PRESCRIBE.........................................................................................................................................................89

............................................................................................................................................................................................................91 HISTORIES......................................................................................................................................................................................92 2.21 TIA, MANAGEMENT, GUIDELINES (2009).......................................................................................................................................92 2.22 GI HX IBD (2009)...................................................................................................................................................................96 2.23 RENAL CALCULI HX (2007, 2009)...............................................................................................................................................97 2.24 THYROTOXICOSIS (2005).............................................................................................................................................................99 2.25 PVD - INTERMITTENT CLAUDICATION (2006 12 MIN) ...................................................................................................................101 2.26 BACK PAIN (RENAL CAUSE, 2007) ..............................................................................................................................................103 2.27 FATIGUE - CHRONIC RENAL FAILURE (2008).................................................................................................................................105 2.28 BPH & INTERPRET BLOODS (2005, RENAL FAILURE).....................................................................................................................107 2.29 DIARRHOEA - HYPERTHYROID (2006, 12 MIN).............................................................................................................................108 2.30 ELDERLY PT COLLAPSE (2007)..................................................................................................................................................109 2.31 PAINLESS JAUNDICE - PANCREATIC CANCER (2008)........................................................................................................................111 2.32 POLYMYALGIA RHEUMATICA (2005)............................................................................................................................................113 2.33 SLE (2006: YR 4 STATION)......................................................................................................................................................115 2.34 CYSTIC FIBROSIS (2006, 12 MIN)...............................................................................................................................................117 2.35 PSEUDOMEMBRANOUS COLITIS (2007) .........................................................................................................................................119 2.36 GORD (2005).......................................................................................................................................................................121 2.37 DISCHARGE PLANNING (2006)...................................................................................................................................................123 2.38 ASSESSMENT POST OVERDOSE (2007).........................................................................................................................................125 3 EXAMINATIONS........................................................................................................................................................................126 3.1 GAIT (2005, 2007 PREVIOUS YR 4 STATION).................................................................................................................................126 3.1.1 Abnormal Gaits with video links.....................................................................................................................................127 3.2 HAND ((RHEUM ARTHRITIS) 2005 (YR 5), 2006, 2008 (YR 4))......................................................................................................128 3.3 KNEE (2007, 2008: YR 4).........................................................................................................................................................130 3.4 HIP (2005, 2006 PREVIOUS YR 4 STATION)...................................................................................................................................132 3.5 SHOULDER (HASNT COME UP PREVIOUSLY).....................................................................................................................................134 3.6 GALS (HASNT COME UP PREVIOUSLY)..........................................................................................................................................136 3.7 SPINE (HASNT COME UP BEFORE)..................................................................................................................................................137 3.8 GI EXAM (2005, 06, 07, 08, 09)................................................................................................................................................139 3.9 CVS - IDENTIFY MURMUR (2005, 06, 07, 08, 09)........................................................................................................................142 3.10 RESPIRATORY (BRONCHIECTASIS 05, 07, 08, LOBECTOMY 06, ? 09)................................................................................................145 3.11 LOWER PERIPHERAL VASCULAR EXAMINATION................................................................................................................................147 3.12 LOWER PERIPHERAL ARTERIAL EXAMINATION ................................................................................................................................150 3.13 CRANIAL NERVES II, III, IV, & VI AND FUNDOSCOPY (2005, 2006, 2007, 2008).........................................................................151 3.14 CAPACITY AND CONSENT............................................................................................................................................................154 3.15 NEURO EXAM - LEGS - PERIPH NEUROPATHY (6 MIN) .....................................................................................................................156 3.16 NEURO EXAM ARMS................................................................................................................................................................158 ....................................................................................................................................................................................................159 3.17 DERMATOMES...........................................................................................................................................................................160 4 FORMULARY.............................................................................................................................................................................161 5 BMJ LEARNING MODULES....................................................................................................................................................174 5.1 COUNSELLING.............................................................................................................................................................................174 5.2 CLINICAL SKILLS.........................................................................................................................................................................174 5.3 HISTORIES..................................................................................................................................................................................174 5.4 EXAMINATIONS...........................................................................................................................................................................175

1 Counselling stations
1.1 MRSA info giving (2009)
Wash hands, introduce self, check ID, get consent, summarise patients presenting symptoms Determine how much the patient already knows, and check they want to know more. Elicit main concerns What is MRSA? The SA stands for Staphylococcus aureus - usually harmless germ often found on skin/nose of healthy people SA sometimes invades skin causing infection. More likely if cut or graze. Cause of boils, pimples, impetigo, skin abscesses and common cause of wound infections. May get into bloodstream to cause more serious infections. More likely in people already unwell. Most SA infections can be treated with commonly used antibiotics MR stands for methicillin-resistant - some healthy people are carriers of MRSA some get infections. MRSA infections resistant to antibiotic called methicillin and also other types i.e. they dont kill the bacteria Infections with MRSA can sometimes become more severe. Who gets MRSA? Most common in people already in hospital, esp. if very ill or have wounds or open sores Catheters and tubes going into veins or other parts of the body ('drips' etc) can be contaminated by MRSA. You can be a carrier or have an infection caused by MRSA If you know you are a carrier you should let the hospital know before you go in (e.g. for an operation). You can then be offered treatment (e.g. ointment to put in the nose or antiseptic washes) MRSA infections usually associated with high fevers, boils, abscesses, can cause pneumonia and urine infections. How is MRSA diagnosed? Sample of blood, urine, body fluid or a swab of a wound can be sent to laboratory. MRSA identified by seeing which antibiotics kill the bacteria How is MRSA spread? Usually by direct skin-to-skin contact, usually in hospitals. Also sheets, towels, clothes, dressings, etc. How can it be avoided? Wash hands regularly (e.g. alcohol), cover cuts, wear gloves if in contact, avoid sharing towels etc. Patients with an MRSA infection may be put in a single bed room Is MRSA screened for in the UK? No universal screening but recommended for: hospital transfers, ICU, orthopaedic and vascular surgery wards. If you are undergoing planned surgery you will be screened for MRSA before you go in (unless emergency) What is the treatment? Usually treated with antibiotics. Limited choice. May need to be IV. Often several weeks, side-effects Summarise, Check Understanding, Ask Questions, offer Leaflet, arrange Follow Up Possible examiners questions: Q. What are the infection screening procedures for MRSA for: i) elective admissions? ii) Emergency admissions? (March 09: http://www.leedsth.nhs.uk/sites/infection_control/documents/LTHT-MRSAscreening-policy.doc ) Screen in pre-operative assessment clinics. Elective admissions to hospitals unless: day case opthalmology, dental, endoscopy; minor derm (eg warts), paeds unless high risk group, 3

maternity/obstetrics unless elective caesareans or high-risk. Swab both sides, separate swabs: anterior nares, groin, axilla, Children < 1 month old umbilicus, sites likely to be colonised. Put in a single specimen bag and send to lab labelled MRSA screening elective admission. Documented in notes. Swabs for sites thought to be clinically infected need separate request forms & specimen bag. Q. What are the eradication and treatment procedures for patients with MRSA? Letter to GP & patient info leaflet to patient. Chlorhexidine gluconate 4% in a surfactant solution 500ml all over body wash to be used 5 days before surgery. Wash hair day 1 and 5. Mupirocin 2% nasal ointment 3g to inner surface of both nostrils three times daily for the 5 days prior to surgery. Alternative is Naseptin but avoid in nut allergy. Another alternative is Prontoderm.

1.2 Explain peak flow and TTO (2009 (TTO 2006))


Explain peak flow to an asthmatic patient and discharge them. Their peak flows were above 75% percent predicted with variability of less than 25%. Explain when to take inhalers, the warning signs for a bad attack and discuss triggers. Introduce self. Elicit name age and occupation. Establish rapport. Elicit patients understanding of asthma and peak flow measurement Brief asthma explanation: narrowing, inflammation, and increased mucus production Brief peak flow meter explanation: simple device that measures air flow out of lungs. Tells us how good you are at pushing air out. In people with asthma this is usually lower than normal. Definition of peak flow: Fastest rate of air you can blow out of your lungs in litres per minute (l/min). Explain procedure o Position: Standing o Preparation: set pointer to zero and use fresh mouthpiece o Inhale fully o Hold meter horizontal, fingers not over pointer. Make tight seal around mouthpiece o Blow out as hard and as fast as possible o Repeat 3 times and take highest reading Record reading in a diary Check understanding by asking patient to do what you have just shown them Why should you measure it? Helps assess how well treatment is working. Assess asthma severity Identify triggers Allows Self Management Plan to be developed What is an acceptable reading? Depends on your gender, age and height (see figure) Work out percentage actual/predicted x 100 Asthma severity: Normal > 80%; Mild-Mod 80-50%; Severe 55-33%; Life threatening < 33% What are the different inhalers for? Reliever inhalers contain bronchodilators. Take as needed to ease symptoms when breathless or wheezy. Relaxes muscle in airways. E.g. salbutamol and terbutaline. Usually blue or grey. If needed >= 3x week preventer inhaler usually advised. Preventer inhalers usually contain a steroid. Taken every day to prevent symptoms. Work by reducing inflammation in airways. Take 7-14 days or more for preventers to build up effect. Usually brown, orange, or red. Long acting bronchodilators work in similar way to 'relievers', but up to 12 hours. E.g. salmeterol and formoterol. Used in addition to a steroid inhaler if symptoms are not fully controlled. How do I know if Im having a bad asthma attack? Very wheezy, 'tight' chest, difficulty breathing. Unable to finish sentences, PEFR <50% best, HR > 100, RR>25 What are the triggers for asthma? Infections, pollens, moulds, exercise, aspirin, beta-blockers, smoking, fumes & chemicals, stress, emotional upset, laughing, animals, house dust mites, some foods (rare). Summarise, Check Understanding, Ask Questions, offer Leaflet, arrange Follow Up Can the patient be discharged? 2008 BTS/SIGN guidelines: Patients should have: Been on discharge meds for 24 hrs, had inhaler technique checked and recorded, PEF >75% of best or predicted and PEF diurnal variability <25%. If PEF<50% on presentation, give prednisolone 40-50mg/day for 5 days, own PEF meter and written asthma action plan, GP follow up arranged within 2 working days, follow up appointment in respiratory clinic within 4 weeks 5

1.3 Surgical options for AAA (2005, 2009)


Mr J is a 66 y.o. man referred to vascular outpatients dept following discovery of a 4cm AAA on US scan for kidney stones. You are the FY2 in the vascular clinic, please explain the diagnosis to the patient and counsel him on treatment options. Wash hands, introduce self, check ID, get consent, summarise patients presenting symptoms Determine how much the patient already knows, and check they want to know more Elicit the patients main concerns Anatomy and Pathology Explain that the aorta is the largest blood vessel in the body which carries blood from the heart down to the legs passing through the chest and abdomen on the way. Usually 2.5 - 3cm in diameter. May stretch like a balloon = aneurysm. AAA is most common type. On average, an AAA tends to get larger by about 10% per year. 6x more common in men and affects 1/20 > 65 years. Exact cause unknown but furring up of the arteries, high BP, FH and increasing age all play a role. Increased risk if obese, diabetic May have no symptoms or may have mild abdominal or back pain Small blood clots from lining of AAA may break off and be carried down the aorta and block a smaller artery US is easiest way to diagnose. All men >65 should now be offered screening. Check understanding and give opportunity to ask questions. Potential complication: rupture As size of AAA increases walls weaken, more likely to burst. Only 50% of treated survive rupture. Preventative surgery carries some risks. Need to balance risk of surgery with risk of bursting. At 5.5cm we know the risk of bursting outweighs the risks of surgery At 4cm we will monitor every 6/12. Stop smoking and control blood pressure. Check understanding and give opportunity to ask questions. Treatment options Two main options: 1. Remove aneurysm and replace with a graft. 3-5% mortality, but usually successful (5 year survival 50-80%). Most subsequent deaths due to co-existing cardiac disease. Alternative called endovascular aneurysm repair (EVAR): Tube passed into blood vessel in leg and up to the AAA. Attached to either side of the aneurysm, forming a tunnel. Safer but failure rate up to 25%. Long term outcome less certain. Need regular check ups for life. Summarise, Check Understanding, Ask Questions, offer Leaflet, arrange Follow Up Possible examiner questions What might indicate the need for an urgent repair? Symptoms: Severe constant or colicky abdominal pain radiating to the back, collapse or feeling faint Signs: Expansile abdominal mass. HR, BP, RR, pale, sweating, cold extremeties, distention, tenderness, peripheral pulses. What should you do if an urgent repair is needed? Fast bleep senior help and vascular surgeon immediately. Order urgent O-ve blood and X match 8 units. Keep systolic <=100mmHg. What are the risk factors? age, male, age, smoking, IHD, cholesterol How would you manage an emergency situation? Give O2, resuscitate, IV access for bloods, stat influsion of colloid blood (keep systolic 90100). If stable keep NBM, prep for theatre, keep under observation. 8

1.4 Smoking cessation (2005 & 2009)


Smoking Facts Greatest cause of illness and premature death in UK. Mainly cancer, COPD, and heart disease. Half of all smokers die from smoking-related diseases. Life expectancy 10 years less. Nicotine: Next cig to avoid withdrawal; Tar: deposits lungs, blood vessels, other parts of the body. Over 4000 chemicals, including 50 known carcinogens; Carbon monoxide: O2 to self or baby. Which diseases are caused or made worse by smoking? Causes 29% of all cancer deaths. 80% lung cancer & COPD. 1/7 heart disease deaths. Other cancers: mouth, nose, throat, larynx, oesophagus, pancreas, bladder, cervix, blood, kidney Strokes, vascular disease, aneurysms, sexual problems, wrinkles, reduced fertility.. etc Worsens: asthma, colds, infections, rhinitis, diabetic retinopathy, hyperthyroidism, MS etc Pregnancy - increased risk: miscarriage, premature birth, illness, stillbirth, long-term development. How does smoking affect other people? Children: asthma, infections, SIDS, academic achievement, asthma, becoming smoker Adults: lung cancer, heart disease, worsens asthma. Benefits of stopping smoking? Benefits straight away. Sooner you stop, the greater the reduction. Never too late! Chest infections and colds less frequent. Smell, taste, financial. If smoked since teens and stop before 35: life expectancy only slightly less. Before 50: risk 50%. How can I stop smoking? 2 in 3 smokers want to stop. Willpower and determination are most important aspects NRT up to 50% quit (4 wks). Available in many forms. Can help control weight. SEs: nausea, dizziness, flu-like, palpitations, dyspepsia, hiccups, insomnia, vivid dreams Bupropion (Zyban) up to 55% quit (4 wks). NOT in pregnancy, breastfeeding, bipolar, seizures, eating disorders. Risk of seizures and drug interactions. SEs include: seizures 1/1000, insomnia, dry mouth, hypersensitivity Varenicline (Champix) up to 74% quit (4 wks). reduces craving. NOT in pregnancy or < 18. SEs: nausea, insomnia, dreams, headaches, flatulence. Irritability, depression and/or insomnia on stopping the drug. Tips To Help Write a list of the reasons why you want to stop, and keep them with you. Set a date for stopping, and stop completely. Get rid of ashtrays, lighters, and all cigarettes. Tell everyone that you are giving up smoking. Get a stop buddy. Be prepared for withdrawal: worse at 12-24 hours, gradually eases over 2-4 weeks. Avoid risky situations: e.g. avoid pub for a while. Take one day at a time. Mark off each successful day on a calendar. Put away the money you would have spent on cigarettes for a treat or holiday Anticipate an increase in appetite: try sugar-free gum and fruit Don't despair if you fail. Learn from your mistakes. May take 3 or 4 attempts Further information: www.smokefree.nhs.uk 9

Summarise, Check Understanding, Ask Questions, offer Leaflet, arrange Follow Up

10

1.5 Complaints procedure (2005, 2007, 2008, 2009)


Due to your poor handwriting, a patient has been given 20mg of warfarin instead of 2mg. This patient also has a peptic ulcer. A (now angry) relative has been informed of this and you have to talk to them. They wish to make a formal complaint. Wash hands, introduce self, check identity of relative Find out why the person is angry and acknowledge this anger Validate their feelings. Give them a chance to vent their anger Offer to do something Technique Sit at same level and make eye contact Do not raise voice, avoid dismissive or threatening body language Encourage the person to speak. Ask open rather than closed questions, use verbal and non verbal cues. Empathise as much as you can Dont: glare, confront, interrupt, patronise, touch, blame others, make unrealistic promises Action what would you do next? Keep the patient in hospital and monitor INR Major bleed Get help ASAP. Stop warfarin, give 5-10 mg Vit K1 slow IV injection INR>8.0 no/minor bleeding stop warfarin restart when INR < 5. If other risk factors for bleed give 500micrograms VitK1 by slow IV injection or 5mg by mouth INR 6-8 no or minor bleeding stop warfarin, restart when INR < 5 INR < 6 reduce dose or stop warfarin, restart when INR < 5 How to make a complaint (for the patient) from http://www.leedsth.nhs.uk/patients/patient-relations/index.php Complaints are valuable because they help us improve. If possible, ask patient for permission to pursue problem on their behalf. What if I dont want to complain but want something done? The Patient Advice and Liaison Service (PALS) can help. If possible, tell someone as soon as you are aware of the problem, e.g. the matron, charge nurse, ward sister, doctor or receptionist. Often they can sort out your problem straight away or provide information, clarification or advice to help you decide what to do. Or, contact the PALS telephone line (0113) 206 7168 and staff will help. There are also Comment Boxes around the hospitals to enable you to have your say, whether this is good or not so good! What if I do want to complain? Who do I complain to? Chief Executive or Patient Relations Manager at St Jamess Hospital, Trust Headquarters, Beckett Street, Leeds LS9 7TF. Be as specific as you can and describe what happened. Include any helpful information such as your date of birth or hospital unit number. Or, I can supply you with a complaint form. Or, contact the Patient Relations Department (see contact details above) whose staff are happy to give help and advice if you are unsure whether or how to complain. There is a 24 hour Patient Relations answerphone. I can give you a leaflet with the contact details (0113 2066261 patient.relations@leedsth.nhs.uk) What will you do? Investigate the complaint, and acknowledge that we have received it within 2 working days. We will let you know how long we think it may take to resolve. We will reach agreement with you if more time is needed Do everything possible to resolve complaint satisfactorily. This may involve meeting you or using mediators. Learn from the complaint you have made and make changes where appropriate What else should I know? Making a complaint will not affect the care you or your relative receives. You should make complaint within 12 months of the event Where else can I get help? 11

The Yorkshire and Humberside Independent Complaints Advocacy Service (ICAS) can provide independent, free information, advice, support and representation to people wanting to make a complaint about NHS treatment. Tel: 0300 456 8349 Advocacy Network Leeds can provide information about other specialist advocacy services. Tel: (0113) 244 9045. Click on Useful Contacts below for more detail. Summarise, Check Understanding, Ask Questions, offer Leaflet, arrange Follow Up

12

1.6 Breast cancer Hx (pair with counselling) (2008)


Introduction, explain purpose, gain consent Name, age, occupation

Presenting Complaint & History or presenting complaint Use open questions Ask specifically about pain, lumps, nipple discharge If pain SOCRATES Associated symptoms: o Lump; site, size, onset, duration, cyclicity, associated symptoms o Nipple discharge; amount, colour, unilateral/bilateral, one duct or several, spontaneous or not o Skin changes, nipple retraction/inversion Systemic symptoms: tiredness, fever, night sweats, weight loss, chest or back pain Cyclicity (i.e. menstrual cycle) Similar symptoms in the past? Any other changes? Past medical history Age at menarche/menopause Children? Ages? Breastfed? Current past and childhood illnesses Surgery Previous breast investigations Recent visits to the doctor Drug History Prescribed meds, esp OCP, HRT, antipsychotics (some can cause hyperprolactinaemia/galactorrhoea) Over the counter medications Recreational drug use Allergies Family history Parents, siblings and children. Specifically breast problems and cancers Social History Smoking, alcohol, employment (past and present), housing, recreational activities Systems enquiry S Summarise Follow up C - Check understanding, A Ask questions, L Leaflet, F arrange a

Possible examiners questions Q. What investigations might you order? A. Mammogram, ultrasound scan, fine-needle aspiration cytology (FNAC) Q. Differential diagnoses? A. Fibroadenoma, Fibrocystic disease, Mastitis, Breast abscess, Mammary duct ectasia, intraductal papilloma (See overleaf for breast cancer counselling) 13

1.7 Breast cancer counselling (pair with Breast ca history) (2008)


You are an FY1 on a general surgery firm. Mrs Cohen (46, Jewish) has had her histology results back. The tests have shown carcinoma of the breast, and her lymph nodes are also positive for the disease. At the MDT meeting it was decided that a mastectomy followed by chemotherapy would be the best course of treatment. Preparation Check notes carefully and establish what youre going to say Read minutes from MDT Introduction Introduction, explain purpose, gain consent Name, age, occupation Explanation Tell her results are back and you have some bad news Tell her unfortunately the tests have shown the lump to be cancerous Ask her if there is anyone she would like her to be with her at this time Explain the cancer is malignant and that it has spread to the lymph nodes Explain that the best treatment would be to have her breast removed Allow time for her to absorb this information Check understanding. Allow her to express her feelings Give tissues if appropriate and comfort her. Does she have any questions? Further details How long the initial operation would take (between 1 and 3 hours, usually 1 hours) Discuss the appearance of the breast after mastectomy (i.e will probably lose shape and nipple) Discuss breast reconstruction. Different options include; o Immediate or delayed (pros and cons, e.g. only one op, but radio may damage new breast, may delay chemo) o Removing breast and skin and inserting implant to gradually stretch remaining skin o Removing breast tissue, leaving the skin, inserting implant o Reconstruction with own living tissue (from back, abdomen, buttocks, thigh) o A combination Will have chance to discuss finer details with oncologist Chemotherapy following mastectomy o Treats cancer by using drugs which kill cancer cells, or stop them multiplying. o In addition to surgery known as 'adjuvant chemotherapy'. Effects of chemotherapy tiredness, nausea and vomiting, immunity, mouth problems, hair loss, constipation, Diarrhoea, Nerve problems, Fertility Ashkenazi Jews genetically predisposed to breast cancer. Screening of relatives Discussion of bras and prostheses available to maintain body shape Conclusions S Summarise whats been said C Check understanding A Give patient opportunity to Ask questions L Offer a post MI Leaflet F arrange a Follow up. Offer support of hospital chaplain or Rabbi Ask Macmillan nurse to come and see patient 14

Document conversation in notes

15

1.8 Epilepsy explanation (2006, 2008)


Wash hands, introduction, check patient name and DOB Ascertain what patient already knows and what they want to know Explain or confirm that the patient has been diagnosed with epilepsy Ask if they know what epilepsy is and if they would like you to tell them more What is epilepsy? The human brain works by sending tiny electrical impulses to other parts of our brain to control our movements and thoughts. In epilepsy these signals get interrupted or misfire and this is what causes an epileptic fit. What causes it? Often we do not know the cause. Genetic factor 1 parent 4%, 2 parents up to 20% Sometimes there are more serious causes. If we have not done so already, we will carry out tests to make sure that we can rule these out. Find out how much detail they want. If they want more other causes include - Metabolic: Can be due to chemical imbalances caused by alcohol, benzos, glucose or , sodium or calcium imbalances, Infectious: meningitis or encephalitis for example Structural: trauma, stroke, space occupying lesions, etc How is it diagnosed? After 2 or more seizures. Eye witness accounts useful: location, often no warning, confused & tired afterwards with no memory of event. Medical history. Blood tests ( BG), ECG, EEG (usually need to be having a fit at time), video telemetry, MRI, CT (may not help) What else could it be? Febrile convulsions, breath holding, reflex anoxic (triggers cause heart to stop or slow down, reducing blood flow to brain), syncope. How is it treated? First wed try to find and treat potential causes -such as chemical imbalances, infection, or structural problems with the brain. You may need to take medication (possibly for years or even for life). Drug treatments are 70% effective, more experimental: high fat diets, vagal nerve simulator. AED side effects - drowsiness, dizziness, weight gain or loss, rash etc. most quite rare. Who should I tell and what should I tell them? Inform friends and family, and work/school/youth leaders etc. Let these people know what to do if you have a fit: clear area, cushions if needed, do not restrain or put anything in mouth, put in recovery position afterwards. If it lasts > 5 min call an ambulance. What else should I do? o Carry an epilepsy id card or wear bracelet o Driving: Inform DVLA & insurance company need to be seizure free for a year (10 years if PSV) o Safety: Best not to lock bathroom doors. Best to take showers rather than baths. o Sports: Get swimming buddy, inform lifeguard, avoid cycling in traffic. Wear a crash helmet. o Contraception: COC/POP etc may not work. Consider IUCD, mirena, or higher dose COC o Pregnancy: 90% normal, but risks increased. AEDs teratogenic eg. Cleft palate. risk of miscarriage, fits in pregnancy. You should request specialist obstetric care. May be able to beastfeed with some older anticonvulsants. o Triggers: include tiredness, stress, alcohol, flashing lights (5%). o Benefits: Contact benefits agency, you may be entitled to help (e.g. travel costs) o Career: May affect your career e.g. army, HGV or bus driver etc. Ideas Concerns Expectations S Summarise C - Check understanding, A Ask questions, L Leaflet, F arrange a Follow up

Potential examiners questions Q. What is the first line drug treatment? Carbamazepine 100mg/12h Q. Toxic effects? Rash, nausea, diplopia, dizziness, fluid retention, hyponatraemia, blood dyscrasias Q. What is the 2nd line drug treatment? Sodium valproate 300mg/12h 16

Q. Toxic effects? Sedation, tremor, weight gain, hair thinning, ankle swelling, hyperammonaemia, liver failure. Q. What is status epilepticus? One continuous seizure > 30 minutes, or recurrent seizures without regaining consciousness between seizures for > 30 minutes.

17

1.9 Post MI & informed consent (2008)


Explain the results of high troponin to a patient, explain her medications to her, convince her to stay in hospital for further investigation and treatment. Answer questions on informed consent and capacity. She was given the following once only medications: GTN spray, aspirin 300mg, Clopidogrel 300mg, Morphine 10mg, Metoclopramide 10mg. She has also been prescribed the following regular prescriptions; Oxygen, enoxaparin70mg sc bd, aspirin 75mg po od, clopidogrel 75mg po od, simvastatin 10mg po od, atenolol 50mg po od, ramipril 5mg po od Introduce self. Elicit name age and occupation. Establish rapport. Elicit patients understanding of what has happened Brief MI explanation: A heart attack (Coronary thrombosis, myocardial infarction or MI) means that part of the heart muscle suddenly loses its blood supply. Without prompt treatment, this can lead to damage to the affected part of the heart. It is usually caused by one of the arteries that supply the heart getting blocked Brief troponin explanation: A blood test that measures a chemical called troponin can confirm an MI. This chemical is present in heart muscle cells and damage to heart muscle cells releases troponin into the bloodstream. The blood level of troponin increases within 312 hours from the onset of chest pain, peaks at 24-48 hours, and returns to normal over a week or two. Explain drugs she has been prescribed; o GTN spray: Nitrates mainly work by relaxing the blood vessels in the body. This causes them to widen. This then makes it easier for the heart to pump blood and reduces the 'strain' on the heart. o Aspirin: Used to thin the blood helping to prevent unwanted blood clots. Caution if pregnant, gout, clotting disorder, stomach ulcer, asthma, liver or kidney problems o Clopidogrel: Works like aspirin. When used along with aspirin has been shown to increase its effectiveness o Morphine: This was given to used to relieve pain as you were in quite a lot of pain when you came in. o Metoclopramide: This is used to relieve sickness as morphine can make some people sick o Oxygen: Your heart pumps blood around your body delivering oxygen to places that need it. As your heart was not working as well as it should this will help ensure your body gets the oxygen it needs o Enoxaparin: This is used to prevent you getting a blood clot. People in hospital who do not move around as much as they might at home are at increased risk of this o Simvastatin: This helps to lower your cholesterol. If levels of cholesterol are too high in the bloodstream, it is deposited on the walls of blood vessels. Eventually this leads to the narrowing of the blood vessels and can cause them to block completely. o Atenolol: slows down the heart, decreases blood pressure, reduces hearts need for oxygen o Ramipril: relaxes blood vessels, lowers blood pressure and increases blood supply and oxygen to the heart. Further investigations: ECG (type of MI), CXR (pneumothorax, aortic dissection), d-dimers (PE) etc. Consent: o You can only consent for procedures you can perform yourself o Patients need the mental capacity to give consent o Informed consent requires a full explaination of what is to be done o Patient should be able to understand the procedure and to weigh up benefits and risks o Patient should be given opportunity to ask questions Possible examiners questions 18

Q. What Troponin value is considered elevated? A. Troponin T >0.1 ng/mL, Troponin I > 1ng/ml Q. What specific type of Troponin is most informative? A. In renal impairment, even against second generation cTnT assays, cTnI is superior. In muscle damage, cTnI is as least as useful as cTnT. Q. What other cardiac markers are there? Creatine Kinase (CK) Q. What else may raise cardiac markers? Troponin CRF, PE, septicaemia; CK: rhabdomyolysis (check kidneys), exercise, surgery, hypothyroidism. Q. What is informed consent? Individual's ability to make an informed decision. Q. What is capacity? Ability to retain and understand information and weigh up risks and benefits Q. What is competence? Degree of mental soundness necessary to make decisions about a specific issue or to carry out a specific act.

19

1.10 Explain post MI drugs & TTO (2008)


You are an FY1 in Cardiology. Mr Manson has been in hospital for 10 days post anterior myocardial infarction. He will be discharged in the next day or two with anti-cholesterol medication, aspirin, ramapril, atenolol and GTN spray (prn). Find out what the patient understands about his discharge and give him any appropriate advice. Preparation Check how long ago he had his MI and if he had any previous MIs, check prescription chart Introduction and review Introduction. Confirm name and age Mention purpose of consultation. Ask them how they feel and what they know about why they were admitted. Elicit understanding about discharge Ask if any dizziness or chest pain at the minute Ask how he will be getting home Social aspects Do they have any particular concerns about going home? Living situation? Flat/house? Stairs? Lift? Employment situation? Can certify him sick for time in hospital and 6 weeks thereafter (then see GP) o Can return to work within two months but should try to replace manual work with lighter duties Social support: live alone? Support from family? Services in place prior to admission? Does he drive? Doesnt have to tell DVLA but should not drive for 1 month (checked govt website) Sex? Would advise him to abstain for a month and take it easy thereafter. Lifestyle issues Explain that a number of factors will reduce the likelihood of subsequent attacks; o Smoking cessation o Diet: Fruit, vegetables, fibre. Reduce red meat, fried food, saturated fats, increase polyunsaturated o Reduce alcohol (weekly: 14 units for woman 21 for man) o Exercise - light exercise built up slowly. Start with just walking for first 6 weeks. Travel: May not be able to fly for 2 months. Medication advice Aspirin: advise importance of 75mg daily. Thin blood and reduces risk of second attack GTN: Take 2 sprays under tongue if chest pain. Repeat in 10 mins but seek medical help if pain persists Statin: Advise need to reduce cholesterol and should be taken at night Ramapril: (ACEI) strengthens heart muscle, provides good BP control and protects against further MI Atenolol: (B blocker) reduces how hard the heart has to work Concordance: Meds will help prevent future MIs, important to take on time as directed S Summarise, C Check understanding, A Ask questions, L Leaflet, F Follow up echo and treadmill.

Additional points for Establishing and maintaining rapport Active listening. Demonstrating interest, concern and empathy Pauses. Appropriate pacing allowing patient to express feelings freely 20

Verbal cues. Use of verbal and non verbal cues.

Potential interview questions BP should be treated post MI if over what? 140/90 What is Dresslers syndrome? Persistent low-grade fever, chest pain (usually pleuritic), pericardial rub, and/or pericardial effusion. Tends to occur a few weeks or even months after MI and tends to subside in a few days. Associated with an elevated ESR.

21

1.11 Breaking bad news (2005)


1. Preparation Prepare the Ground Know all the facts before the meeting. Find out who the patient wants present, and ensure privacy and chairs to sit on. 2. What does the patient know? Ask for a narrative of events from the patient (eg How did it all start?) 3. Is more information wanted? Test the waters, but be aware that it can be very frightening to ask for more information (eg Would you like me to explain a bit more?) 4. Give a warning shot eg Im afraid it looks rather serious then allow a pause for the patient to respond. 5. Allow denial Denial is a defence mechanism and a way of coping. Allow the patient to control the amount of information they receive. 6. Explain (if requested) Increase the patients information to match the professionals to the appropriate level. Details of this information may not be remembered, the way you offer the information will. 7. Listen to concerns Ask What are your main concerns at the moment? and then allow space for expression of feelings. 8. Encourage ventilation of feelings KEY phase in terms of patient satisfaction with the interview, because it conveys empathy. 9. Summary and plan Summarise concerns, plan treatment, foster appropriate hope. 10. Offer availability Most patients need further explanation (the details will not have been remembered) and support (adjustment takes weeks or months) and may benefit greatly from a family meeting. Summarise, Check Understanding, Ask Questions, offer Leaflet, arrange Follow Up

22

1.12 Hypertension med non adherence (12 min)


You are an FY1 at a general practice. Mr Obewi, a 56 year old man has had elevated BP on three previous visits despite being on antihypertensives. He is of African descent and has been on a calcium channel antagonist for the past two years. Preparation Obtain notes and familiarise self with past medical history, check date of last antihypertensive prescription Introduction Introduce self and establish rapport. Confirm patient identity and DOB, ask him how much he knows Consultation Explain that his BP has been elevated on three previous visits despite being on medication Ask patient if they have any thoughts about what might be causing this Management Ask how he is finding his medications. Is he taking them? Has he ever forgotten to take his medication? When and how often? Ask patient to describe in his own words which medication he takes, when and how he takes it. Check that he understands the reasons for taking his medication Let him know that best time to take meds is in morning Complications Explain hypertension can lead to problems with headaches, eyesight, kidneys, heart, lungs, inc risk strokes Monitoring Ascertain if patient would be amenable to the idea of checking his own BP o If so, recommend one to buy (see: http://www.bhsoc.org/bp_monitors/automatic.stm) Check FH of strokes, vision problems, kidney problems. Explain may be genetic predisposition (Afro-Caribbeans). Lifestyle issues Ask specifically about occupation, suggest stress reduction techniques if appropriate Suggest losing weight if overweight Regular physical activity (5 days a week, at least 30 minutes). Diet: 5 portions fruit and veg, more fibre, reduce red meat & sat fat, increase mono-, or poly-unsaturated fats, eat 2-3 portions of fish/week, reduce salt (<5-6 g/day), recuce caffeine & alcohol (f14:m21/w, <=f3:m4/day) Stop smoking give help if necessary Diet & exercise alone can reduce BP by at least 10 mmHg. Medications Advise may need to increase dose if BP remains high Advise need for tests (bloods, echo) Summarise, Check Understanding, Ask Questions, offer Leaflet, arrange Follow Up Potential examiners questions 23

Drug therapy shoud be offered to? Persistently >= 160/100 mmHg or CVD risk >= 140/90 mmHg. First line drug for >=55 or black patients of any age? calcium-channel blocker or a thiazide-type diuretic. First line drug < 55s? ACE inhibitor (or an angiotensin-II receptor antagonist if ACEI not tolerated). When would you refer? BP>= 180/110 mmHg, suspected phaeochromocytoma (e.g. postural hypotension, headache, palpitations, pallor and diaphoresis), fall in systolic when standing of >=20 mmHg). What investigations might you wish to do? FBC, U&Es, creatinine, lipids, GGT (alch), ECG, Echo (LVH) See over for guidelines

BETA-BLOCKERS Beta-blockers are no longer preferred as a routine initial therapy for hypertension. But consider them for younger people, particularly: women of childbearing potential patients with evidence of increased sympathetic drive patients with intolerance of or contraindications to ACE inhibitors and angiotensin-II receptor antagonists. If a patient taking a beta-blocker needs a second drug, add a calcium-channel blocker rather than a thiazide-type diuretic, to reduce the patients risk of developing diabetes. If a patients blood pressure is not controlled by a regimen that includes a beta-blocker (that is, it is still above 140/90 mmHg), change their treatment by following the flow chart above. If a patients blood pressure is well controlled (that is,140/90 mmHg or less) by a regimen that includes a beta-blocker, consider long-term management at their routine review. There is no absolute need to replace the beta-blocker in this case. When withdrawing a beta-blocker, step down the dose gradually. 24

Beta-blockers should not usually be withdrawn if a patient has a compelling indication for being treated with one, such as symptomatic angina or a previous myocardial infarction.

25

1.13 Diabetes counselling Type 1 (6 min)


What is diabetes? Two types: Occurs when the level of glucose in blood becomes higher than normal. Normally after we eat, foods broken down in gut into sugars, mainly glucose. Absorbed into blood. Glucose used by the cells in body for energy. Should not go too high or too low. Usually when blood glucose rises hormone called insulin also rises. Insulin normally made by pancreas and released into blood stream Insulin lets cells take in glucose for fuel. Some converted into glycogen or fat (energy store) Blood glucose also maintained by breaking down glycogen or fat back into glucose. What is Type 1 diabetes? A.k.a. juvenile, early onset, or insulin dependent diabetes. In UK about 1 in 250 people. Usually develops quickly, over days or weeks, as pancreas stops making insulin. Thought to be 'auto-immune': trigger may be viral, severe inflammation, or surgical removal. Genetic predisposition: First degree relative has 6 in 100 chance What are the symptoms of Type 1 diabetes? Very thirsty, pass a lot of urine, tiredness, weight loss, malaise Without treatment, acids form in the bloodstream ('ketosis'), dehydration, coma, death How is diabetes diagnosed? If 'dipstick' shows glucose in urine, likely but not diagnostic: need confirmatory blood test Possible complications? Long-term: damage blood vessels, atheroma, angina, heart attacks, stroke, poor circulation, eye problems, kidney damage, nerve damage, foot problems, impotence Treatment & complications Need insulin injections for rest of life. ~2-4 injections each day. But too much can cause hypos: sweaty, confused, unwell, coma. Emergency treatment of hypoglycaemia: sugar or a glucagon injection then starchy snack Aim to keep blood glucose near normal, reduce 'risk factors' (e.g. BP), detect complications asap Healthy diet: not special foods: low fat, high fibre diet, starch, fruit and vegetables. See dietician Monitor glucose & adjust insulin: between 4 and 7 mmol/l before meals, < 10 mmol/l 2 hrs after. Stop smoking, do regular physical activity, lose weight if overweight Regular checks HBA1c checks control over the last 2-3 months. Aim for 7% Blood pressure: may need antihypertensive; Cholesterol - may need statins / low dose aspirin Eye checks - detect problems with the retina or glaucoma Kidney function, Foot checks - prevent foot ulcers; Coeliac and thyroid disorders more common Immunisation against 'flu (each autumn) and pneumococcus (once). Further info: www.diabetes.org.uk Summarise, Check Understanding, Ask Questions, offer Leaflet, arrange Follow Up 26

See over for 2004 NICE guidelines

27

28

29

1.14 Type II diabetes


What is diabetes? Two types: Occurs when the level of glucose in blood becomes higher than normal. Normally after we eat, foods broken down in gut into sugars, mainly glucose. Absorbed into blood. Glucose used by the cells in body for energy. Usually when glucose rises insulin also rises. Insulin normally made by pancreas and released into blood Insulin lets cells take in glucose for fuel. Some converted into glycogen or fat (energy store) Blood glucose also maintained by breaking down glycogen or fat back into glucose. What is Type 2 diabetes? a.k.a. Maturity onset, or non-Insulin Dependent. Mainly in over 40s. 3% over 40, 10% over 65 You dont make enough insulin OR cells dont use it properly (resistance), or combination More common in overweight/obese. Tends to run in families. South Asians, AfricanCaribbeans What are the symptoms? Common: thirst, passing large amounts of urine, tiredness, weight loss. Symptoms may develop so gradually (weeks/months) you may not recognise that you are ill Also: blurred vision, frequent infections (candiadisis) Diagnosis - if 'dipstick' shows glucose in urine, likely but not diagnostic: need confirmatory blood test Possible complications Very high blood glucose level: dehydration, drowsiness, can be life-threatening. Long-term: damage blood vessels, atheroma, angina, heart attacks, stroke, poor circulation, eye problems, kidney damage, nerve damage, foot problems, impotence Treatments & complications Address lifestyle factors stop smoking diet, weight control and physical activity. May be enough. Medications: some helping insulin to work (biguanides e.g. Metformin), others boost amount made (Sulphonylureas e.g. gliclazide), or slow glucose absorption (Acarbose not used often) Insulin injections may be needed if above do not work well enough. Hypo (< 4 mmol/l): too much meds, missed meal, unplanned exercise. Symptoms: trembling, sweating, anxiety, blurred vision, tingling lips, paleness, mood change. Take sugar then starch. Aim to keep blood glucose near normal, reduce 'risk factors' (e.g. BP), detect complications asap Healthy diet: not special foods: low fat, high fibre diet, starch, fruit and vegetables. See dietician Monitor glucose & adjust insulin: between 4 and 7 mmol/l before meals, < 10 mmol/l 2 hrs after. Regular checks HBA1c checks control over the last 2-3 months. Aim for 7% Blood pressure: may need antihypertensive; Cholesterol - may need statins / low dose aspirin Eye checks - detect problems with the retina or glaucoma 30

Kidney function, Foot checks - prevent foot ulcers; Coeliac and thyroid disorders more common Immunisation against 'flu (each autumn) and pneumococcus (once).

Further info: www.diabetes.org.uk Summarise, Check Understanding, Ask Questions, offer Leaflet, arrange Follow Up

See over for guidelines

31

2009 NICE Guidelines

32

1.15 Do Not Resuscitate (2005)


You are a doctor in MAU. Mr Ryder, aged 55 has advanced Huntingtons disease. For the last year he has required full time nursing care with specialist psychiatric input due to behavioural disturbance. He is not currently competent to make decisions for himself, but before the disease progressed he made a living will and requested he should not be resuscitated in the event of organ failure resulting from his condition. He has been admitted with aspiration pneumonia and acute renal failure due to sepsis. He is being managed with antibiotics, fluids and careful monitoring of blood chemistry. Haemodialysis may help in the short term. You have discussed the case with a consultant from ICU who feels Mr Ryder is unlikely to survive the next few days. His sister is here to talk with you, she is not genetically affected. Your task is to discuss the appropriateness of haemodialysis and intensive care along with resuscitation status. Introduce yourself and establish that she is Mr Ryders sister Establish what she already knows e.g. what have you been told so far about your brothers condition? Ask if she has anyone with her, or if she would like to call for someone else Seek further information about the patient from his sister using open questions Be empathic, pace your explanation slowly and carefully Explain the situation e.g. o Pneumonia - Pneumonia is inflammation of the tissues in one or both lungs. It's usually caused by an infection. At the end of the airways in the lungs there are clusters of tiny air sacs called alveoli. In pneumonia, these tiny sacs become inflamed and fill up with fluid. This can leading to coughing and also makes it hard to breathe. It also means the body is less able to absorb oxygen. o Acute renal failure The kidneys filter the blood to remove waste products and produce urine. In acute renal failure the kidneys suddenly (acutely) become unable to do this. This means that they are unable to remove salt, water and waste products from the bloodstream. Haemodialysis is the term given to a procedure that helps filter blood for a patient. This may help your brother in the short term o Sepsis Your brothers renal failure was caused by sepsis this is when the body develops an inflammatory response to bugs in the blood, urine, lungs, skin, or other tissues. What is being done? Explain that his condition is being managed with antibiotics, careful fluid replacement and monitoring of blood chemistry. Check if she has any questions at this point Fire a warning shot for example We are treating your brothers condition as vigorously as we can in the hope that things may improve. But he is seriously ill, and there is a chance he may deteriorate further Check that she understands his life is in danger Broach the subject of his DNR request e.g. I am not sure if you are aware, but your brother has made a living will and has set out what he would like to be done should a situation such as this arise Give her some time to take this on board Tell her his wishes e.g. Before his disease had progressed your brother made a living will and requested that he should not be resuscitated in the event of organ failure resulting from his condition. This means that if he should stop breathing, or if his heart should stop, he would prefer that we did not make any attempt to re-start his heart again or ventilate him artificially. Confirm understanding and invite questions make it clear you are happy to answer questions now or later Conclude with assurances assure her that although her brother requested he should not be resuscitated you and your team will continue to do all they can to treat his conditions and relieve symptoms. Reassure her that if she needs to talk to someone about his care, you will be willing to do this. 33

Potential examiners questions Q1. If someone has not appointed an agent to act on their behalf in the event of them being unable to make decisions about their treatment, who is ultimately responsible for decisions about their care the most immediate relative, or the consultant in charge of their care? A: The doctor in charge of their care. Q2. Can proxy decision makers (i.e. those appointed by the patient) over-rule decisions made by a doctor regarding a patients treatment ? A: Proxy decision makers can not demand treatment that is judged to be against a patients best interests.

34

1.16 Inhaler technique newly diagnosed asthmatic (2007)


Mr X has recently been diagnosed with asthma. Explain this diagnosis to the patient and show him how to use his Salbutamol inhaler. Introduction: Introduce self, status, purpose of interview. Clarify name, age and occupation of patient. Understanding: Establish what patient knows of condition already. Explanation: o What is it? Asthma is a lung condition which is characterised by difficulty in breathing. People who have asthma may have sensitive airways that narrow when irritated, making it difficult for air to move in and out. This is what causes the symptoms you have been experiencing. Symptoms include cough, shortness of breath and wheeze. o Causes and triggers: FH, More common with atopic disease : eczema, hayfever, allergy, cig smoke, dust-mites, pollen, pets, cold air, exercise. o What to do during an attack: Dont panic, may make it worse. If symptoms dont resolve within 10 minutes call an ambulance. o How is it treated?: There are a number of treatments available. Salbutamol works by opening the airways so that air can flow in and out more freely. The inhaler allows salbutamol to get delivered exactly where it is needed. Id like to show you the correct technique for using one of these inhalers. Do you have any questions so far? Position: Ensure the patient is sitting upright or standing. Inhaler: Chose blue inhaler and explain that you need to shake it first. Exhale: Advise patient to fully exhale air from lungs and remove cap from inhaler Inhalation: Demonstrate how to co-ordinate inhaler whilst taking a deep breath in o hold the inhaler between your thumb and index finger, with your index finger on the canister. Make a seal with your lips around the mouth of the inhaler. As you press down on the inhaler it is important to simultaneously start taking a deep breath in. Hold breath: Advise patient to hold breath for ten seconds after taking inhaler Repeat: Wait a minute before repeating the process. Understanding: Check pt has understood what you have discussed Side effects: fast heart rate, shakiness or headaches Regularity: two puffs PRN up to 4x daily When to seek help: If needing more than 2 puffs qds consult doctor Demonstrate: Ask the patient to talk you through what you have just discussed SCALF: Summarise. Check understanding, any Concerns? Follow-up. Ask questions. Leaflet.

See also: http://www.nhs.uk/video/pages/medialibrary.aspx? Tag=Children+and+babies+&Page=4 See over for BTS 2009 guidelines

35

36

BTS 2009 guidelines for adults

BTS 2009 Guidelines for children aged 5-12 37

38

1.17 Explain Gastroscopy/Sigmoidoscopy procedure (2006; 12 min)


Gastroscopy (Sigmoidoscopy overleaf) Introduction: Introduce self. Elicit name, age, occupation. Establish rapport. Understanding: Find out what the patient knows so far. Concerns: Find out if the patient has any particular concerns. Explanation: Gastroscopy is a test to look inside the oesophagus (food pipe), stomach and duodenum (end bit of stomach). It is sometimes called endoscopy. Endoscope: An endoscope is a thin, flexible, telescope. It is about as thick as a little finger. The endoscope is passed through the mouth, into the oesophagus and down towards the stomach and duodenum. The tip of the endoscope contains a light and a tiny video camera so the operator can see inside your gut. The endoscope also has a 'side channel' down which various instruments can pass. For example, the operator may take a small sample (biopsy) from the inside lining of the stomach by using a thin 'grabbing' instrument which is passed down a side channel. Why? E.g. recurring indigestion, heartburn, abdo pain, repeated vomiting, difficulty swallowing. Help confirm/rule out: oesophagitis, stomach ulcers, duodenitis and gastritis, cancer of the stomach and/or oesophagus. Preparation: Do not eat for 6 hours before the test. Small sips of water may be allowed up to two hours before the test. If you have a sedative you will need somebody to accompany you home. Some medications may need to be stopped before the test. Procedure: Usually done as outpatient 'day case'. Common and routine. Operator may numb back throat with local anaesthetic. May be given sedative to help relax. This is usually given by an injection into a vein in the back of your hand. The sedative can make you drowsy but it does not 'put you to sleep'. Lie on side on a couch. Asked to put a plastic mouth guard between teeth to protect them. Swallow the first section of the endoscope. Quite thin and easy to swallow. Gently pushes it further down into your stomach. Video camera at tip sends pictures to screen. Air is passed down endoscope to make the stomach lining easier to see. May cause you to feel 'full' and want to belch. Operator may take biopsies (painless). Sent to the lab for testing. Endoscope then gently pulled out. Usually takes about 10 minutes. Allow at least two hours for the whole appointment, to prepare, give time for the sedative to work (if you have one), for the gastroscopy itself, and to recover. Does not usually hurt, but it can be a little uncomfortable, particularly when you first swallow the endoscope. Post procedure: May be ready to go home after resting for half an hour or so. If had sedative - may take longer. You should not drive, operate machinery or drink alcohol for 24 hours after having the sedative. You will need somebody to accompany you home and to stay with you for 24 hours until the effects have fully worn off. Not foolproof. Repeat gastroscopy sometimes advised. Side-effects/complications: Usually no problems. May have a mild sore throat for a day or so. May feel tired or sleepy for several hours if you have a sedative. Slightly risk of chest infection or pneumonia. May cause damage to gut -> bleeding, infection, and rarely, perforation. If any of the following occur within 48 hours, consult a doctor immediately: Abdominal pain, Fever, Difficulty breathing, Vomiting blood. Serious complications (rare): heart attack stroke, allergic reaction

39

SCALF: Summarise. Check understanding, any Concerns? up. Sigmoidoscopy overleaf

Ask questions. Leaflet. Follow-

40

Sigmoidoscopy. Introduction: Introduce self. Elicit name, age, occupation. Establish rapport. Understanding: Find out what the patient knows so far. Concerns: Find out if the patient has any particular concerns. Explanation: Sigmoidoscopy is a procedure where a doctor or nurse looks into your back passage (the rectum) and the part of the bowel before this (the sigmoid colon) using an instrument called a sigmoidoscope. Definitions: The sigmoid colon is the final portion of bowel joined to the rectum. A sigmoidoscope is a small tube with an attached light source about the thickness of your finger. A doctor or nurse inserts the sigmoidoscope into the anus and pushes it slowly into the rectum and sigmoid colon. This allows the doctor or nurse to see the lining of the rectum and sigmoid colon. The procedure should not be painful but it may be a little uncomfortable. Preparation: In order for us to get a clear view, your rectum and lower bowel need to be empty of faeces (stools or motions). You may be asked to clear your bowel by taking laxatives for a day or two, or by using one or two enemas prior to the procedure. A commonly used laxative to clear the bowel is called Picolax. A common plan is: For three days before the procedure - eat a light diet. On the day before the procedure - take one Picolax sachet (by mouth) at 8am and one at 6 pm. Read the instructions carefully on the Picolax sachet on how much water to add. For 12 hours before the procedure - have fluids only. You may be given an enema on arrival in the hospital to clear the very bottom of the bowel of faeces. The test itself: It usually takes 15-20 minutes. Usually you do not need an anaesthetic or sedation. Youll wear a hospital gown so that the lower half of your body is exposed. You will be asked to lie on your left side with your knees drawn up toward your chest. First the doctor or nurse will gently insert a gloved and lubricated finger into the rectum to check for blockage and to widen the anus. Then the sigmoidoscope will be inserted and gently pushed further into the rectum and colon. Air is gently pumped through the sigmoidoscope to help viewing. This can cause you to feel bloated and uncomfortable, and give you an urge to defecate ('move your bowels'). As the sigmoidoscope is slowly removed, the lining of the bowel is carefully examined. You may pass this air as wind when the sigmoidscope is being removed. A small sample (biopsy) of bowel lining may be taken during the procedure. (A channel in the sigmoidoscope allows the doctor or nurse to pass forceps or other instruments to take biopsies, or for therapy.) The sample is sent to the laboratory to be looked at under the microscope. It may also be tested for various conditions that can affect the bowel. SCALF: Summarise. Check understanding, any Concerns? up. Ask questions. Leaflet. Follow-

41

1.18 Explain sigmoidoscopy results = Crohns disease (2007 - 6 min)


Introduction: Introduce self. Elicit name, age, occupation. Establish rapport. Find out what the patient knows. Concerns: Find out if the patient has any particular concerns. What is it?: Inflammation in gut. Flares up periodically. Symptoms vary, depending on part of gut affected. Common symptoms: diarrhoea (+/- blood), abdominal pain, malaise. Medication may ease symptoms & may prevent flare ups. Surgery to remove gut sections may be needed to treat some flare-ups. Understanding the gut: Long tube starts at the mouth and ends at anus. When we eat, food passes down the oesophagus, into stomach, and small intestine where it is digested and absorbed into blood. Gut becomes large intestine (colon and rectum, sometimes called the large bowel). Function of colon: Absorbs water, contains undigested food e.g. fibre. Last part stores faeces, then passed out. Who gets Crohn's disease? ~ 1 in 1500 people. Common 15- 30 yrs. Women slightly more than men. Which part of the gut is affected in Crohn's disease? Any part of the gut. Commonly starts in last part of small intestine (ileum). The mouth, gullet and stomach affected less commonly. ~3/10, inflammation just in small intestine. ~2/10 cases inflammation occurs just in colon. What causes Crohn's disease? Unknown. ~3/20 people have close relative with it. Bacterium or virus may be involved. Germ may trigger immune system to cause inflammation in genetically predisposed. X3 in smokers. Symptoms of flare-up? inflammation may cause; Diarrhoea (most common first symptom). May be mucus, pus or blood. Urgency is common. Wanting to go but nothing to pass is also common (tenesmus). * Pain occurs in ~7/10. Lower right side of the abdomen. * unintentional weight loss. * Ulcers. Mouth ulcers and GI ulcers. Blood in stools (motions or faeces). * Generally feeling unwell may include loss of appetite, fever, and tiredness. * Anaemia * Anal fissures. Anal skin tags. * May not absorb food well, and may become deficient in vitamins and other nutrients. Symptoms vary depending on part(s) of gut affected. Other symptoms? Inflammation and pain of joints; skin rashes; inflammation of the eye; liver inflammation. How does it progress? Chronic, relapsing condition. Ongoing but times when few or no symptoms What are the possible complications? * Stricture. Narrowing of part of gut - can cause difficulty in food passing through -> pain and vomiting. * Perforation (hole in the wall of gut). Can cause infection or an abscess inside the abdomen. Can be lifethreatening. *Fistula (channel between two parts of body). *Cancer. Small increased risk of cancer of the colon. *Osteoporosis ('thinning of the bones') related to poor absorption. How is it diagnosed? Colonoscopy: typical cobblestone appearance. Usually biopsied. May have gastroscopy (endoscopy). May have special X-ray of large intestine (barium enema) or small intestine (barium meal). Barium coats the lining of gut and shows up white on X-ray. More sophisticated tests such as an MRI or CT scan may be needed. Blood tests may help to assess level of inflammation, check for anaemia etc. Aims of treatment? 1. Clear flare-up symptoms. 2. Prevent further flare-ups. Treatment options? 1. No treatment, 2. Corticosteroids (e.g. budesonide and prednisolone). Improves 7/10, dose reduced gradually, and stopped once symptoms ease. Tablets, suppositories, injections. 3. Immunosuppressant drugs: a) Immunomodulators (suppress immune system, e.g. azathioprine, mercaptopurine, and methotrexate. Used in more severe cases. b) Biological therapies - target specific chemicals of immune system involved in inflammation (TNF-alpha) e.g. infliximab and adalimumab 4. Aminosalicylate drugs e.g. mesalazine, ofsalazine, balsalazide and sulfasalazine. Exact mechanism unclear, 5. Antibiotics if infection suspected. 6. Dietary treatments: strict liquid diet (controversial). 7. Surgery: resection. 8. General measures: Iron tablets, vitamins and supplements, parenteral nutrition in severe cases, painkillers, hospital admission if severe. 42

Preventing flare-ups?: case by case basis: Regular mesalazine, immunomodulators, biological therapy. BUT do not always work. Steroid medication is not generally used longterm to prevent flare-ups. Giving up smoking may reduce number and severity Pregnancy: Discuss in advance with doctor. May need extra folate, and certain drugs must not be used. Prognosis? Variable. Without treatment ~3 in 20 have frequent flare-ups. Sometimes flare-ups life-threatening. 15/20 remain in work 10 years after diagnosis. Majority of cases disease is manageable enough to maintain a near normal life. Up to 8/10 require surgery at some stage. Half within the first 10 years. Commonly to remove stricture. Young adultsexpect to have 2-4 operations in life. Inflammatory bowel disease? Crohn's or ulcerative colitis. Similar. But UC inner lining of gut, Crohn's can spread through whole wall. UC only affects colon and rectum. Features of both = indeterminate colitis. SCALF: Summarise. Check understanding, any Concerns? Ask questions. Leaflet. Follow-up.

43

1.19 GORD (2006, 12 min)


Introduction: Introduce self. Elicit name, age, occupation. Establish rapport. Find out what the patient knows. Acid Reflux & Oesophagitis: When acid from the stomach leaks up into gullet (oesophagus). May cause heartburn and other symptoms. A drug which reduces the amount of acid made in stomach is common treatment and usually works well. Some people need long-term daily medication to keep symptoms away. Understanding oesophagus and stomach: Cells lining stomach make acid that helps digest food. Stomach also makes mucus which protects them from damage. Cells lining oesophagus different and less protected from acid. Circular band of muscle (a 'sphincter') at junction between oesophagus and stomach relaxes to allow food down, but tightens to stop food and acid leaking back up (reflux) into the oesophagus. Acts like a valve. What is reflux and oesophagitis? Some acid leaks up into oesophagus. Oesophagitis inflammation of lining of oesophagus. Usually due to reflux. Gastro-oesophageal reflux disease (GORD): General term for - acid reflux, with or without oesophagitis. Symptoms? Main symptom is heartburn; Others: pain in upper abdomen and chest, feeling sick, acid taste in mouth, bloating, belching, burning pain swallowing hot drinks. Usually worse after meal. Uncommon symptoms: persistent cough, particularly at night (acid irritating trachea). Cough and wheeze. Gum problems, bad breath, sore throat, hoarseness, and a feeling of a lump in the throat. Severe chest pain (may be mistaken for a heart attack). What causes acid reflux? Sphincter at bottom of oesophagus normally prevents reflux. Problems if sphincter not working well. Cause usually unknown. May be due to pressure e.g. pregnancy, large meal, bending forward. Hiatus hernia (part of stomach protrudes into chest through diaphragm) increases chance of reflux. Who gets it? ~1 in 3 adults have some heartburn every few days, ~1 in 10 adults have heartburn at least once a day. More common in smokers, pregnant, heavy drinkers, overweight, and aged 35 - 64. Tests? Not usually necessary. Many people diagnosed when they have typical symptoms eased by treatment. Tests may be advised if symptoms: are severe, or do not improve with treatment, or are not typical of GORD. * Endoscopy - thin, flexible telescope is passed down the oesophagus into the stomach. Allows doctor to look inside. With oesophagitis, the lower part of the oesophagus looks red and inflamed. Some people very sensitive and can have symptoms with little or no inflammation. Oesophagitis - oesophagus seen to be inflamed. Endoscopy -negative reflux disease - typical symptoms of reflux but endoscopy normal. Test to check acidity inside oesophagus may be done if diagnosis not clear. ECGs, chest X-ray, etc, may be done to rule out other conditions. What can I do to help symptoms? Stop smoking, avoid foods that cause symptoms, (e.g. peppermint, tomatoes, chocolate, spicy foods, hot drinks, alcohol), some drugs (e.g. ibuprofen or aspirin). Others include: diazepam, theophylline, nitrates, and calcium channel blockers e.g. nifedipine. Tell doctor if you suspect these. Lose Weight if overweight; Correct posture; Don't eat 3 hours before bedtime, drink 2 hours; raise head bed by 10-20 cms. What are the treatments? Antacids: neutralise acid. Usually quick relief. Mild or infrequent bouts. Acid-suppressing drugs: if symptoms frequent see doc. May prescribe PPIs or histamine receptor blockers (H2 blockers). Suppress amount of stomach acid. PPIs include: omeprazole, lansoprazole, and esomeprazole. H2 blockers include: cimetidine, famotidine, nizatidine, and ranitidine. PPIs 1st line. May only need for month or so. Some need long-term treatment. Thought to be safe, SEs uncommon. Prokinetic drugs: Speed up food passage (e.g. domperidone and metoclopramide). Less common, help bloating or belching. 44

Surgery: 'tighten' lower oesophagus. May be an option if drug treatment not working well or not wanted long-term. Are there any complications from oesophagitis? Stricture (scarring and narrowing) uncommon. Barrett's oesophagus - changed cells more prone to cancer (1-2%). Most people do not develop these complications. Tell doc if you have pain or difficulty (food 'sticking') swallowing - may be complication. SCALF: Summarise. Check understanding, any Concerns? up. Ask questions. Leaflet. Follow-

45

1.20 Explain AF and initiate Warfarin therapy (2009)


Scenario: You are the FY1 in general medicine. Mrs Smith is a 73 year old lady who was newly diagnosed with atrial fibrillation whilst on your ward. Start Mrs Smith on appropriate anticoagulation with a view to cardioversion in outpatients in a few weeks. Counsel the patient and prescribe a loading regimen of anticoagulant therapy. Wash hands, introduce self, check patient ID, explain purpose and gain consent.

Counselling Check knowledge of atrial fibrillation and explain diagnosis Check knowledge of treatment and explain cardioversion Explain need for need for anticoagulant (stroke, limb ischaemia, kidney failure) Check knowledge of warfarin tablet, explain what it does in simple terms Any questions so far? Explain risk of excessive bleeding though need for careful monitoring (INR) Explain need to visit warfarin clinic to get dose at right level. Stress compliance. Check with pharmacist before starting any new drugs (warfarin interactions). This includes over the counter tables and herbal remedies Avoid NSAIDS (ibuprofen, aspirin, diclofenac) and changes in levels of consumption of alcohol, cranberry juice , Vit K rich foods such as green leafy vegetables, herbal remedies eg. Ginseng, St Johns Wort, Cod liver oil Avoid cuts and bruises (sewing, shaving, contact sports) Screen for contraindications: stomach ulcer, bleeding disorder, high blood pressure, kidney disease, pregnant, trying for a baby or breastfeeding Youll be given a yellow warfarin therapy book check your details and read this You will be given a warfarin card or medic alert bracelet keep with you at all times Tell GP, dentist and other HCPs you are on Warfarin Not recommended in pregnancy, periods may be heavier. SEs = bleeding, bruising, diarrhoea, purple toes (3-8 weeks)

Prescribing warfarin Use Fennertys Protocol: o Young/healthy patients - 10mg at 18:00 daily for 3 days with adjustments based on daily INR at 09:00. Elderly 5mg at 18:00 daily for 3 days with adjustments based on daily INR at 09:00

Possible questions What drugs might you wish to stop before commencing warfarin? What is the target INR; 46

o o o

In this case (i.e. AF)? (INR of 3.0 for up to 4 weeks before procedure) For treatment of DVT & PE? (2.5, unless recurrent then 3.5) For mechanical valves? (3.0 for aortic, 3.5 for mitral)

How would you manage the situation if Mrs Smith had been given 50mg of warfarin initially by accident? (stop warfarin, if INR >12 5mg VitK PO/IV, if bleeding seek senior help urgently) How does warfarin work? (inhibits vitamin K-dependent clotting factors)

47

1.21 Cystic fibrosis counselling (2006)


Introduction: Introduce self. Elicit name, age, occupation. Establish rapport. Find out what the patient knows. Concerns: Find out if the patient has any particular concerns. What is it? A serious inherited disease which mainly affects the lungs and pancreas, but also other organs. Symptoms usually begin in early childhood and include persistent cough, wheeze, repeated chest infections, malabsorption of food, and general ill health. Treatments: antibiotics, physiotherapy, mucus thinning drugs, pancreatic enzyme replacements and other therapies. Can also affect liver, nose and sinuses, reproductive organs and sweat glands. May not function correctly and make thicker than normal secretions. What causes cystic fibrosis and how common is it? Genetic disorder. Pair of genes (on chromosome 7) faulty. Control the way the cells handle sodium and chloride ions. Too much sodium travels into the cells, water follows leaving water outside cells. Watery secretions outside the cells thick. ~1 in 2500 UK babies. Autosomal recessive - in order to develop it you need to inherit two cystic fibrosis genes, one from your mother and one from your father. If you inherit only one you are a carrier. 1 in 25 UK caucasians are carriers. Less common in Afro-Caribbeans and Asians. If two parents carriers 1 in 4 chance of CF, 1 in 2 chance of being a carrier and 1 in 4 chance neither. What are the symptoms? Usually first develop within the first year may not appear until later. Lung symptoms: thicker mucus and sputum, infection & inflammation * Persistent cough which typically produces a lot of sputum. * Wheezing, shortness of breath and breathing difficulties. * Recurring chest infections. These can be severe such as pneumonia. lung function. Gut symptoms (17 in 20): Thickened secretions block normal flow of the digestive juices from pancreas. Thus absorption of food. esp fatty foods and fat-soluble vitamins (vitamins A, D, E and K): * Malnutrition, growth weight gain * Large, smelly, greasy, fatty faeces (stools or motions) occur in about a third of cases. * Bloated abdomen, constipation. At Birth: meconium ileus - thick, dark, sticky substance which is made by the baby's gut before being born. Urgent surgery may be needed to relieve the blockage. Failure to thrive Other symptoms and complications: * Repeated sinus infections. * Polyps forming in the nose. * Infertility (especially in males (97%) as the tubes which carry the sperm can become blocked). * Damage to the liver which may lead to cirrhosis occurs in about 1 in 12 cases. * Diabetes. Rare in children, more common in adults who have had cystic fibrosis for years. * Pancreatitis, Rectal prolapse, Osteoporosis How is cystic fibrosis diagnosed?: Sweat test: sodium and chloride in skin sweat. Genetic test: detects the CF gene. Screening test: All newborn babies 'heel prick' (immunoreactive trypsinogen in CF) What is the treatment for cystic fibrosis? Multidisciplinary: Paeditricians, nurses, physiotherapists, dieticians, counsellors, psychologists, PCT. Regular checks. LUNG: Physiotherapy and exercise. Twice daily chest physiotherapy is common practice. Exercise. Antibiotics and antifungals. Inhalers, e.g. salbutamol. Dornase alfa helps break down and 'thin' mucus. Oxygen. PANCREATIC: Nutrition: high fat and carbohydrate diet. Enzyme supplements to help to digest food. 48

OTHER TREATMENTS: Salt supplements, specialist liver treatments, insulin (diabetes), nasal steroids (polyps), antacids, laxatives, routine immunisations, lung or heart/lung transplantation. FUTURE: Gene therapy, What is the outlook (prognosis)? People now living into their 40s Genetic counselling: People with family history may wish to have genetic counselling and testing Further help and information: Cystic Fibrosis Trust www.cftrust.org.uk

49

2 Clinical Skills
2.1 Blood Gas & interpretation (2008, 2009)
You are the medical house office on call. Mr Gower was admitted under yoiur care two days ago for infective exacerbation of COPD. The nurse in charge has bleeped you stating that he is deteriorating. Demonstarte how you would take an arterial blood gas on the manikin provided. Explain what youre doing as you go. Orientation: Establish location of nearest blood gas analyser, and if not one nearby, what procedure is Introduction: Introduce self. Elicit name and age. Establish rapport Explain: I understand you are having some difficulty breathing. In order to check how things are progressing, I need ot take some blood from your wrist. Although this procedure is quite painful, I can reassure you that it is a quick procedure that is essential for us to help you get better. Do you have any questions? Consent: Obtain consent and re-check ID Position: Ensure patient is sitting or lying comfortably Equipment: Clean trolley and gather relevant equipment: pre-heparinised syringe, (optional local anaesthetic), alcohol swabs, cotton wool/gauze, 23G arterial gas needle, pair of non-sterile gloves, sharps box. Preparation: Wash hands and don gloves Position: Have patient lying down or sitting with arm well supported. Hyper extend wrist on rolled up towel. Locate radial pulse, noting size, depth and direction Clean skin with alcohol swab (and infiltrate with local anaesthetic if necessary) and drape the area Attach 23G ABG needle to pre-heparinised syringe. Expel excess heparin. Hold in dominant hand like a dart. Fix chosen area on skin between index and middle fingers of non-dominant hand (do not repalpate area) Warn patient to expect a sharp scratch Insert needle at 30 against the direction of arterial flow (i.e. towards the elbow) When a flash of blood appears in the hub of the needle stop advancing. The blood should fill the syringe under arterial pressure. Gentle aspiration may be required in some cases (eg manikins). Obtain 2ml blood Withdraw needle and press firmly on puncture site with a gauze for 5 minutes (check for haematoma). Immediately place needle in sharps bin Expel bubbles from syringe and immediately cap it State that you would immediately label the specimen and take it to the blood gas analyser, or if it needs to be sent to the lab, label the specimen and arrange immediate transport Case 2.

Examiners questions Case 1.

50

Case 1: Elderly patient breathing room air Q1. Outline the abnormalities in this ABG Q2. What are the likely causes for these abnormalities? Case 2: 68 yr old man, 40 pack years, SOB, productive cough, green sputum. Tachypnoeic and using accessory muscles. Widespread expiratory wheeze with bibasal coarse crepitations. On 28% oxygen. Q1. Outline the abnormalities and give a likely diagnosis Q2. What type of ventilation could be considered in this case? Answers to case 1 and 2: Case 1: A1. PaO2, pH (alkalosis), PaC02 normal, HCO3. Therefore uncompensated metabolic alkalosis A2. Common causes of poor oxygenation are pulmonary oedema or pneumonia. Causes of metabolic alkalosis: vomiting, 1 or 2 hyperaldosteronism, hypercalcaemia, diuretics, bicarbonate ingestion. Case 2: A1. pH, PaO2, PaCO2, HCO3, BE. Therefore partially compensated respiratory acidosis. Type 2 respiratory failure (i.e. both PaO2 and PaCO2 are abnormal). Likely to be COPD. A2. Bi-level positive airway pressure ventilation. If more serious intubation might be considered. Case 3. Case 4.

Case 5.

51

Case 3: A 27 year old student with SOB and tingling in hands. RR 28. Chest exam unremarkable. CXR and bloods normal. ECG sinus tachycardia. ABG taken on 35% O2. Q1. Outline the abnormalities in this ECG Q2. What is the likely cause? Q3. Outline treatment. Case 4: A 32 year old. Generally unwell. No history available. Appears dehydrated. RR 22, ABG on room air. Q1. Outline the abnormalities seen Q2. What tests should you request Q3. Which drugs or substances taken in overdose could give similar findings? Case 5: 57 year old former shipyard worker. Exercise tolerance now 10 metres on flat. Life long non smoker. CT shows diffuse bibasal interstitial changes. ABG on room air Q1. Outline the abnormalities seen Q2. What are the likely causes? Q3. What long term therapy might be considered? Blood Gas Answers Case3: A1. pH, PaO2~ (should be roughly 10% less than inspired i.e. 35-10=25%), PaCO2, HCO3, BE. Therefore partially compensated respiratory alkalosis. A2. Probably caused by hyperventilation. May also occur due to stroke or SAH affecting respiratory centre. A3. Remove oxygen. Reassure and encourage to breathe slowly into a paper bag. Case 4: A1. pH, PaO2 normal, PaCO2, HCO3, BE. Therefore metabolic acidosis with partial respiratory compensation. Na+, K+ normal, urea, creatinine normal, Cl- normal, HCO3. Anion gap = (131 + 4.5) (96.1 + 12.6) = 26.8. Thus raised anion gap metabolic acidosis. The anion gap is an 'artificial' and calculated measure that is representative of the unmeasured ions in plasma or serum. Metabolic acidosis with normal ion gap < 16mmol/L = HCO3 loss from gut, e.g. diarrhoea, renal tubular acidosis Metabolic acidosis with raised ion gap > 16mmol/L = ketoacidosis, rneal failure, lactic acidosis, salicylate toxicity, methanol, antifreeze ingestion A2. Urinalysis for ketones, plasma lactate levels, salicylate levels. A3. Aspirin, methanol or ethylene glycol. Case 5: A1. pH normal, PaO2, PaCO2 normal, HCO3 normal, BE normal to high A2. Type 1 respiratory failure (i.e. O2 low but CO2 normal). Possible causes include pulmonary fibrosis (most likely), pulmonary oedema, pneumonia, pulmonary embolism. A3. Long Term O2 Therapy (LTOT) may be considered, however strict guidelines for this must be met.

52

2.2 Blood cultures, interpret, prescribe antibiotics (2005, 2009)


Introduction. Elicit name, age, occupation. Establish rapport. Explanation: I have been asked to take a sample of blood from your arm as you have a raised temperature. This will help us find out if an infection is causing your temperature and will help us find the best treatment to give you. Do you have any questions? Obtain consent. Double check patients ID Collect necessary equipment: 2 culture bottles, butterfly needle & vaccutainer attachment (or 2 needles and a syringe), sharps bin, alcohol swabs x 3, cotton wool, 20ml syringe, gloves, disposable tourniquet

N.B. These are the most recent Leeds guidelines Wash hands with soap and water, then dry hands and put on disposable apron. Prepare equipment (2 culture bottles, butterfly needle and vaccutainer (or needle and 20ml syringe), sharps bin, chlorhexidine/isopropyl alcohol, cotton wool, sterile gloves, disposable tourniquet Disinfect the rubber tops of culture bottles using a wipe containing 2% chlorhexidine in 70% isopropyl alcohol. Identify patient e.g. double check name band and verbally confirm identity Select venepuncture site and clean visibly soiled skin with soap and water then dry. Apply tourniquet if required. Clean site using a 2% chlorhexidine in 70% isopropyl alcohol wipe - press swab in centre of chosen venepuncture site. Then apply the disinfectant with a spiral outward motion from the centre of the venepuncture covering 1-2 finger breadth to each side. Allow to air dry (the drying process kills the bacteria). Put on sterile gloves while skin disinfectants dry. Sterile examination gloves allow for repalpation if necessary Attach a butterfly needle (needle-safe if available) to the needle-safe bottle-filling device. Perform venepuncture ensuring asepsis is maintained. Collect blood sample by placing needle-safe bottle-filling device onto blood culture bottles (aerobic followed by anaerobic). Remove bottles once 10ml has been obtained in each. Remove tourniquet. Place cotton wool over site and apply gentle pressure while removing needle. Press firmly over site until bleeding has stopped (the patient can be asked to do this if appropriate). Discard needle and blood bottle filling device into a point of use sharps bin. Write patient details and clinical information on blood culture bottles according to Trust policy. Wash hands with soap and water, then dry hands Arrange transport of the sample to the laboratory. Ensure that sampling details and any subsequent positive results communicated by the microbiology department are accurately documented in the patients notes and advice is acted on. If using needle and syringe follow the same procedure and do not change needle between venepuncture and inoculation, or between bottles.

Possible examiners questions Q1. How long might it take to get the results back from microbiology? A1. Depends on the bacteria, but can take up to 72 hours. Q2. The lab has grown E.Coli from your blood cultures. What can e.coli in the blood cause? A2. Neonatal meningitis, septicaemia Q3. The patient is allergic to penicillin which of the following would you use to treat?: Gentamicin - ok 53

Quinolone - ok Ceftriaxone - avoid Ciprofloxacin - ok Cefotaxime use with caution Chloramphenicol - ok

54

2.3 Managing handover & prioritising


No idea how to write a mock station for this!

55

2.4 Prescribing in renal failure (2009)


You are an FY1 working on an elderly ward. Mrs Reiter has just been admitted from a nursing home on to the Ward. She has passed 300mL of urine in the last 24 hrs. The results of a recent blood test are; Ref Na+ 134 mmol/L 135-145 K+ 6.2 mmol/L 3.5-5 Urea 19.7 mmol/L 2.5-6.7 Creatinine 403 mol/L 70-150 Cl98 mmol/L 95-105 HCO312.6 mmol/L 24-30 Mg2+ 0.72 mmol/L 0.75-1.0 PO430.89 mmol/L 0.8-1.45 The nursing home provides you with the following drug list; Ramipril Furosemide Spironolactone Metformin Aspirin Voltarol Pilocarpine eye drops Paracetamol Q1. Why do you think Mrs Reiter has been admitted? (underlying condition) Q2. Describe one indication for each drug on her list and common dosages for each? Q3. Using the Cockcroft and Gault formula (supplied) calculate Mrs Reiters Creatinine Clearance Rate. Mrs Reiter is 84 years old and 65kg in weight. Estimated Creatinine Clearance in mL/min = (140 Age) x Weight in kg x Constant Serum creatinine Constant = 1.23 for men, 1.04 for women Q4. Q5. Q6. Q7. Q8. What degree of renal impairment does Mrs Reiter have? Which drugs would you wish to cross off? Which drugs should be stopped permanently? Write up a drug chart for Mrs Reiter. What is a normal urine output? Answers overleaf

56

A1. Acute Renal Failure A2. Ramipril: (1.25 10mg max daily) Mild to moderate hypertension, congestive heart failure Furosemide: (20-40mg daily) Oedema, resistant hypertension Spironolactone: (100-200mg daily up to 400mg) Oedema and ascites in cirrhosis of the liver, nephritic syndrome, CHF, primary hyperaldosteronism Metformin: (500mg 2g daily max) diabetes mellitus, polycystic ovary syndrome Aspirin: (300-900mg every 4-6 hrs, max 4g) analgesia, angina, MI (75mg daily), post-TIA Voltarol: (75-150mg daily in 2-3 divided doses) Pain and inflammation in rheumatic disease, gout, postoperative pain Pilocarpine eye drops: (1-2 drops max 4x daily) glaucoma, dry mouth Paracetamol: (0.5-1g every 4-6 hours max 4g daily) pain, pyrexia A3. Estimated Creatinine Clearance in mL/min = (140 84) x 65 x 1.04 403 = 56 x 65 x 1.04 / 403 = 3785.6/403 = 9.39 A4. Degree of renal failure eGFR mL/min/1.73m2 Normal stage 1 > 90 with other evidence of kidney damage Mild Stage 2 60-89 with other evidence of kidney damage Moderate Stage 3 30-59 Severe Stage 4 15-29 Established Stage 5 < 15 Therefore Mrs Reiter is in Stage 5 Renal failure. A5. Drugs I would wish to cross off; Spironolactone Avoid in severe renal impairment Metformin contraindicated in patients with severe renal impairment Aspirin oral anticoagulants should be avoided if creatinine clearance is < 10mL/min Voltarol - should be avoided if creatinine clearance is < 10mL/min Other things to note; Ramipril max initial dose 1.25mg once daily (dont exceed 2.5mg daily if eGFR < 10mL/min Furosemide may need higher doses. Deafness may follow IV injection Pilocarpine eye drops manufacturers advise caution Paracetamol OK but may need infusion A6. If her eGFR improved, some of the drugs could be re-started cautiously, such as Aspirin and Voltarol A8. > 400mL/24 hr

57

2.5 Cannulation and fluid management (2008, 2009)


You are an FY1 doctor on a medical ward. A nurse comes up to you and states that she thinks Mr Jones is very dry. You decide to assess his hydration status. Q. What are the signs of early hypovolaemia? A. HR, postural hypotension (drop of > 20mmHg on standing), urine output < 0.5ml/kg/hr., dry mucous membranes, cap refill > 2 sec, absent JVP on lying flat, dark urine Q What are the signs of late hypovolaemia? A. sunken eyes, increased skin turgor, urine output < 17ml/h, BP Q. His blood results are back. What might they show if he is dehydrated? A. urea, packed cell volume, albumin, osmolality, creatinine Q. How much is lost through insensible losses daily? A. 500ml-1L You assess Mr Jones and find that his urine output is < 0.5ml/kg/hr. He is alert, his pulse is 100 beats/min, his blood pressure is 90 mmHg, his respiratory rate 16/min, and his temperature 36C. His chest is clear. His heart sounds are normal. His sats are 95% on air. What is your next course of action? A. High flow oxygen, IV access, regular basic monitoring(e.g. continuous ECG), urinary catheter (if not already), fluid challenge. Q. You decide to give a fluid challenge. Talk/walk through what you would actually do in step by step detail 1. Note baseline observations (HR, BP, RR, sats, cap refill, urine output) Prescribe the appropriate fluids you decide to give 500ml of Volplex (see overleaf) Cannulate the patient (following based on LTHT guidelines): Introduce self, identify patient, explain procedure, gain consent. Wash hands with soap and water and put on apron Clean trolley, gather equipment: cannulation pack (Sterile field/towel, gauze swabs, dressing, cannula Documentation), sterile gloves, cannula (with octopus attachment or filter valvecap, chloraprep, 10mL syringe, needle, 10mL saline, tourniquet, sharps bin, apron. Inspect all packaging for damage and expiry dates Clean hands with alcohol gel Open cannulation pack. Drop on cannula, 10mL syringe, needle. Open saline and leave near sterile field Clean hands with alcohol gel Apply tourniquet & select cannulation site (e.g. non-dominant arm and distal as possible) Clean site with chloraprep using crosshatch patern for 30 seconds. Clean hands with alcohol gel and don sterile gloves. Draw up 10ml of saline using the needle and immediately discard needle into sharps bin If using an octopusattachment, prime each leg. Open dressing towel, make hole in centre, place on patients limb. DO NOT repalpate the vein Remove needle guard from cannula and inspected device for integrity (any faults should be reported) Anchor vein by applying traction a few centimetres below the point of insertion Warn patient to expect sharp scratch, and holding cannula with bevel up, insert at 30 Look for and confirm flashback, decrease angle between device and skin Advance cannula off stylet with dominant hand, but not completely. Remove stopper and keep if necessary Release tourniquet & occlude vein with non-dominant hand Secure cannula with digit of non-dominant hand and remove stylet immediately into a sharps container Attach primed extension set (or white stopper) and flush cannula with 5 -10mL saline Secure device in manner that does not impair visual assessment of insertion site 58

Ensure patient is comfortable. Apply insertion date sticker, with the date and time Document insertion using cannula documentation record. Dispose of waste appropriately

Stop at this point and ask questions about choice of fluid (see over) 2. Fluid challenge: choice of fluid Q. What are the two main types of fluids and give examples of each? A. Crystalloids (Saline, Dextrose, Hartmans) and Colloids (gelofusine, volplex, blood) Q. What is the main disadvantage of crystalloids? A. When used to replace massive loss of fluid, they cause peripheral oedema and occasionally pulmonary oedema. Q. Describe how saline redistributes within the body? About 75% redistributes within extracellular compartment and about 25% remains in the intravascular compartment. Q. Name 3 disadvantages of colloids? Risk of pulmonary oedema/respiratory failure, interference with coagulation, risk of anaphylactic or transfusion reactions, cost Q. What types of fluid loss are crystalloids better for (give specific examples)? severe vomiting, diarrhoea, diabetic ketoacidosis, bowel obstruction Q. What is the advantage of Hartmann's? Less likely than normal saline to cause a hyperchloraemic acidosis. Q. What type of fluid loss are colloids better for (give examples)? Severe acute volume depletion: haemorrhage, burns, severe sepsis. Talk/walk through how you would set up an IV infusion Gather appropriate equipment: fluid bag, giving set, extension tubing if necessary, drip stand Explain to the patient what you are going to do and gain consent Check fluid prescription chart (type of fluid, volume ot be infused, period of time for infusion to run) Check bag with a colleague: expiry date, clarity of solution, presence/absence of additives Close wheel clamp on giving set line Pierce outlet port on bag with spike of giving set Squeeze fluid chamber and release repeat until chamber about half full Slowly open wheel clamp and run through fluid, removing air from tube Secure giving set to arm making sure there are no potential snags with clothing or bedding Check drip runs freely when opened and adjust drip rate according to prescription Sign fluid chart, noting start time.

Q. How do you calculate the drip rate? Drops per minute = volume to be infused (ml) x drops per ml (standard set is 20 drops/ml) / time in minutes - e.g. drops per minute = 1000 x 20 / 480 (8 hours) = 42 drops per minute Q. What are the average fluid requirements of an adult? A. 30-35ml/kg/24hr Q. Name three types of fluid loss? 1. Recordable (polyuria, NG aspirate, diarrhoea, vomiting, drains), 2. Insensible (wound leakage, pyrexia, tachypnoea, burns), 3. Third space (e.g. pancreatitis, post-op) Q. Estimate the fluid deficit for each of the following (in an adult): - HR < 100, BP normal, Urine output > 30ml/hr = < 750ml - HR >100, BP normal, urine output < 30ml/hr = 750-1500ml - HR > 120, BP reduced, urine output < 17ml/hr = 1500-2000ml Q. How might you gain a more accurate estimation of fluid requirements? A. Fluid balance chart Q. What are the average Na+ requirements? A. 2mmol/kg/24hrs ~ 140mmol/day Q. What are the average K+ requirements? A. 0.5-1mmol/kg/24hr ~40-60mmol/day Q. What are the U+Es reference ranges for Na+ and K+? Na+ 135-145 mmol/L, K+ 3.55 mmol/L 59

Q. What might be the best fluid to use in a patient with liver failure and ascites? A. 5% dextrose it containes no Na+ and excess Na+ causes ascities in chronic liver failure. Q. What might be the best fluid to use in a patient with acute renal failure? A. Colloid or saline but avoid K+ Q. What may be required in severe renal failure? A. restriction of Na+, K+ and fluid.

60

61

2.6 Pre-op assessment (2006)


Wash hands, introduce self (full name and position), check patient id (verbally & band) Explain purpose and gain consent HISTORY: Medical History What operation are they having done? (If not already known) Previous surgery and anaesthesia anaesthetic complications (suxamethonium apnoea, malignant hyperpyrexia) Previous hospital admissions Cardio: hypertension, palpitations, angina, MI, cardiac failure, orthopnoea, stroke or TIA Respir: breathlessness, cough, sputum, tuberculosis GI and renal: weight loss, blood in stool, dysphagia, heartburn, liver disease (e.g. hepatitis), renal failure Other: diabetes, sickle cell anaemia, epilepsy, neuromuscular / orthopaedic problems. Drug History Prescribed medication contraceptives, anticoagulants, steroids, insulin, any changes in medication Over-the counter & alternative medications Recreational drug use (esp. ecstasy, cocaine, narcotics) Allergies e.g. previous anaesthetic, antiseptic, plaster, latex Social history Smoking (pack years) & Alcohol (post op chest infections 6x more likely in smokers) Any chance they might be pregnant? LMP? Level of support in post operative period Family history - of allergic reactions, anaesthetic complications, medical and surgical conditions EXAMINATION Record height and weight and calculate BMI (weight (kg)/ height (m2) normal = 20-25) State that you would wish to examine the CV, Resp, GI and neurological systems Assess neck mobility by asking patient to flex and extend neck Assess jaw mobility by asking patient to open and close mouth Assess dentition state Carry out a Mallampati pharyngeal assessment (III and IV are relative contraindications) (1=can see all of ulvula, 2 = can see most of uvula, 3=can only see a bit of uvula, 4 = cant see uvula) MANAGEMENT Express need for: FBC, LFTs (including clotting screen), U&Es, blood glucose, ECG, CXR, group save & cross match, pregnancy test if necessary (In what situations would you need each these?) Assess ASA physical status rating 1= healthy, 2=mild/moderate systemic disease, 3 = severe systemic disturbance, some activity limitation, 4 = life threatening systemic disease, severe activity limitation, 5 = moribund with limited chance of survival, 6 = brain dead for organ removal Meds: regular meds, stop pill 4 weeks before major surgery, pre-meds (e.g. lorazepam 1-2.5mg PO), prophylactic antibiotics, anti-coagulation, extra steroids if taken regularly EXPLANATION Fasting solids 6 hours, milk 4 hours, fluids/gum 2 hours Pre-medication eg benzodiazepines The anaesthetic procedure, post operative pain relief, post operative nausea and vomiting Going home / driving Summarise, Check Understanding, Ask Questions, offer Leaflet, arrange Follow Up Possible examiners questions 62

Q. Which of the following would you use for cardiothroacic surgery if the patient had a penicillin allergy? Flucloxacillin, gentamicin, vancomycin, amoxicillin, co-amoxiclav Q. Which of the following would you use for ENT surgery if the patient had a penicillin allergy? Flucloxacillin, gentamicin, amoxicillin, clarithromycin, co-amoxiclav

63

2.7 Interpret blood test results Addisons (2007)


You are an FY1 in MAU. A patient presents with general malaise. You decide to do a pinprick glucose test. Talk through how you would do this step by step. Introduction, explanation, consent, check ID Wash hands, ask patient to wash hands. Assemble lancet with new tip Put on gloves Does strips tub correspond with calibration chip in glucometer? Turn glucometer on and insert strip Prick side of patients finger furthest from thumb Squeeze finger to extract blood. Wick up blood. Ensure bleeding has stopped Note reading Dispose of contaminated equipment in sharps/clinical waste bin Thank patient Remove gloves & wash hands Record reading on appropriate paperwork If out of normal parameters act immediately on results.

Blood Glucose is 4.2. You order some more blood tests and they come back with the following; Na+ 125 (135-145mmol/L) K+ 6.2 (3.5-5mmol/L) Urea 7.2 (2.5-6.7 mmol/L) Calcium 3.4 (2.12-2.65) TSH 11 (< 10 mU/L) Q. Give a possible diagnosis A. Addisons disease Potential examiners questions Q. What is Addisons disease? Endocrine disorder where the adrenal glands produce insufficient steroid hormones (glucocorticoids (e.g. cortisol) and often mineralocorticoids (e.g. aldosterone)). Q. How is it diagnosed? Short ACTH stimulation test Q. Name some symptoms of Addisons disease: A. fatigue, loss of appetite, weight loss, nausea, abdominal pain, myalgia, salt craving Q. Name some signs of Addisons disease: Areas of hyperpigmentation on skin and mucous membranes. Low BP. Q. How is Addisons diagnosed? Short ACTH stimulation test (Synacthen test): Do plasma cortisol before and 30 minutes after giving 250g Synacthen IM. Addison's excluded if 2nd cortisol >550nmol/L. Q. How is it treated? ~15-25mg hydrocortisone daily, in 2-3 divided doses eg 10mg on waking, 5mg lunchtime. Can cause insomnia. Mineralocorticoid replacement may be needed eg if postural hypotension, Na+, K+ or plasma renin: fludrocortisone PO from 50-200g daily. Adjust both on clinical grounds. If poor response to treatment, suspect associated autoimmune disease (check thyroid, do coeliac serology). Q. What should you tell the patient about their treatment? Warn against abruptly stopping steroids. Inform Doctors/dentists/surgeons of steroid use: give steroid card, advise wearing a bracelet declaring steroid use. Add 5-10mg hydrocortisone to daily intake before strenuous activity/exercise. Double steroids in febrile illness, injury or stress. Patients should be given syringes and in-date IM hydrocortisone, and shown how to inject themselves in case vomiting prevents oral intake. If vomiting, take hydrocortisone 100mg IM, and seek medical help. Q. How might an addisonian crisis present? Patient acutely ill, hypotension (esp postural), weakness, confusion, feeble rapid pulse and soft heart sounds, pyrexia common, 64

anorexia, nausea, vomiting, severe abdominal pain, increased motor activity progressing to delirium or seizures. Q. What is the treatment for an Addisonian crisis? Hydrocortisone 100mg IV stat, cefuroxime 1.5g IV, give IV fluids (i.e. treat shock)

2.8 Cremation form (2005, 2007)


You are an FY1 doctor and have been asked to fill in a cremation form (see appendix) for Mr Jones, a retired car mechanic who you treated last week. The patients details are as follows; Mr Alan Tim Jones, 19 Stanley Grove, Leeds, LS2 4RR, DOB 14/02/1932. He died yesterday at 14:45 in Ward 27 of the LGI, LS9 7TF. You are not related to the deceased. You are not Mr Joness usual medical practitioner, but he has been on the ward you were working on over the last two weeks and you have seen him on several occasions. You last saw him alive 72 hours ago. You have not yet seen the body after death. You have been provided with a death certificate detailing his cause of death I(a) Cerebrovascular accident (b) atherosclerosis, (c) hypertension. II Diabetes mellitus and multi infarct dementia. A post-mortem request has not been made. He had not had any operations the year before he died. He was looked after by a nurse called Sally Biggins 72 Howard Street, Leeds, LS7 3BR during his illness. Your colleague Dr. Harry Hill was present at the death (address 67 Young Street, Leeds, LS2 9BY), who had no concerns about the cause of death. You also have no concerns about the cause of death, and have no reason to suspect it was violent or unnatural. A coronor has not been informed about the death nor has the coronors office been contacted. Your colleague, Harry Hill filled in the death certificate (tel no: 0113 1234567). Mr Jones had a pacemaker fitted seven years ago. You do not know if it has been removed. Q. What do you need to do next? A. You must go to see the body to check the patients identity (appearance and wristband) and ascertain if the pacemaker has been removed. Then fill in the appropriate form (see over or http://www.justice.gov.uk/guidance/docs/cr4.pdf )

65

2.9 TTO form transcribe from drug chart (2006)


Need to look out for obvious errors and check doses etc.

66

2.10 Catheterisation
You are a FY1 in urology. Mr Johnson has presented with acute urinary retention. You decide he needs a urinary catheter. Explain to the patient what you would like to do, then insert the catheter talking through each step as you go. Introduce self. Elicit name, age and occupation. Establish rapport. Because of the problems you are having I would like to insert a flexible plastic tube through your penis and into your bladder to relieve the pressure. It should not be painful but may feel a little uncomfortable. Ill just need to go and get someone else to help me with this. Do you have any questions? Obtain consent and get a chaperone Gather: Catherisation pack, catheter bag, sterile gloves, lignocaine gel, Foley Catheter 16 or 18 (male), antiseptic solution, 10ml sterile water filled syringe, adhesive tape, 10ml saline solution. Wash hands, put on an apron and clean the trolley Ask the patient to undress from the waist and ask him to retract his foreskin Drop the catheter pack on to the trolley and unwrap it, touching only the outside of the packaging. Drop the equipment that is wrapped from its wrapping on to the sterile area. Pour sterile water into small bowl with cotton wool. Don sterile goves Open LA jelly and stick catheter tip into it Make hole in drape and place on patient so that penis passes through the hole. Wrap gauze around penis using left (or non dominant) hand. Clean the penis using the wet swabs and a pair of tweezers. Alternatively use dominant hand being extremely careful not to touch the penis with the hand. If in any doubt, change gloves. Still holding penis with left hand, insert the tip of the syringe and put in 5ml of the local anaesthetic. Hold for 3-5 minutes. Talk about the weather/politics/holiday! Put catheter in kidney dish and place kidney dish between patients legs Holding the penis vertically, insert the catheter into the urethra. Keep the end of the catheter over the tray to catch any urine. If resistance is encountered, lower the penis to a horizontal position to negotiate the prostate. When the catheter is fully inserted, inflate the balloon with 1ml sterile water and ask patient to let you know if they feel any pain. If not, continue with remaining 9ml. Tug on catheter to ensure balloon lodged in neck of baldder. Attach drainage bag to end of catheter and REPLACE THE FORESKIN or ask the patient to do so (paraphimosis) Document in notes, writing in size of catheter, lot numbers and residual volume of urine collected. Dispose of waste appropriately. Thank the patient and ask them to let someone know if they experience any pain or discomfort

Potential examiners questions Q. What are the indications for catheterization? A. Monitoring of urine output, acute urinary retention, chronic obstruction, irrigation of the bladder, imaging Q. What are the contraindications? A. Pelvic trauma, previous stricture, previous failure to catheterize, severe phimosis Q. What are the potential complications? A. Paraphimosis, urethral perforation/trauma, bleeding, infection, urethral strictures 67

2.11 Video of ward round write notes (2006, 2008)


Not sure what to do for this one!

68

2.12 Needlestick injury


You may be shown a video of a student suffering a sharps injury and asked to comment on the students technique, what should be done, and potentially to fill in the blood request forms for both the student and the patient. Technique: Did the student Introduce self correctly? (full name and position) Identify patient correctly? (verbally and wrist band) Wash hands and put on gloves prior to touching patient? Inspect skin and find suitable site? Clean skin using a cloraprep for 10-20 secs and not re-palpate? Obtain sample successfully and safely? (removed tourniquet prior to removing needle, apply pressure etc) Not resheath needle? & Dispose of needle safely in sharps bin? Needlestick injury protocol Encourage bleeding Wash with soap and water, dry and cover with waterproof plaster DO NOT suck the wound Splashes on eyes mouth etc should be rinsed with 1L saline Report incident to person in charge and record incident on IR1 form Report incident to Occupational Health (office hours) or A&E (outside office hours) Request Dr/nurse looking after pt to carry out risk assessment of source patient (not recipient) Ask patient for consent to take blood and test for Hep B, C and HIV Risk assessment Are you HIV positive or do you think you might be? Have you ever injected or been injected with non prescribed drugs (including anabolic steroids) Have you ever had sex in countries where HIV is common (Africa, far East, Caribbean?) Have you ever been given money or drugs for sex? Have you ever had sex with anyone; HIV +ve, sex worker, IVDU, persons who have had sex in above countries? Men only - Have you ever had oral or anal sex with another man with or without a condom? Women only have you ever had sex with a man who may have done the above? In the last 12 months have you ever had an injury that might have put you at risk of Hepatitis or HIV? Taking the Blood sample Get patients consent, take blood and send to Microbiology for Hep B, C, & HIV. Document in patients notes Get recipient blood sample (in clotted blood sample tube) Inform recipient that sample will be stored for 2 years. Obtain Hep B vaccination history of recipient Send recipient sample to microbiology for Hep C, HIV (and Hep B if appropriate) Treatment May be prescribed antiretrovirals (triple therapy within 1hr), 500 units of Hep B immunoglobulin (within 24hr) or Hep B booster depending on significance of exposure Currently no post-exposure prophylaxis for Hep C Possible examiners questions Q. What is the UK prevalence of Hep B, C and HIV? 69

A. < 0.5%, < 0.5%, , 0.1% respectively Q. What is the transmission risk of Hep B, C and HIV? (Hep B if not vaccinated) A. 1 in 3, 1 in 50, 1 in 300 respectively Q. What is triple therapy? A. combination of three drugs including either a protease inhibitor or a non-nucleoside analogue reverse transcriptase inhibitor (NNRTI) and two nucleoside analogue reverse transcriptase inhibitors (NRTIs)

70

2.13 Adverse reaction to blood transfusion


A patient had just been given a blood transfusion. You look in the obs chart and notice that their temperature has quickly risen from 37.5 to 38.4, their heart rate has increased from 70 to 89, their BP has decreased from 140/90 to 120/75, and they have developed an urticarial rash. How would you manage this patient? Stop or slow the transfusion Check the patients name and date of birth. Check the patients blood type in the notes Check the type of blood being given to the patient Management depends on the nature and severity of the reaction o Acute haemolytic: Due to ABO incompatibility. Features: raised temp, decreased BP, flushing, abdominal/chest pain, oozing venepuncture sites, DIC. Tell blood bank/haematologist. Send transfusion blood, FBC, U&E, clotting, cultures & urine to lab. Keep IV line open with 0.9% saline. Activated protein C reduces mortality in disseminated intravascular coagulation (DIC). Seek senior advice re: ITU admission o Anaphylaxis: Bronchospasm, cyanosis, low BP, soft tissue swelling. Maintain airway and give O2. Call anaesthetist. o TRALI: (Transfusion related acute lung injury). Dyspnoea, cough, CXR white out, widespread coarse crackles. 100% O2. ABG monitoring, Call ITU. Remove donor from panel. o Non-haemolytic / allergic: If the blood type is correct and the pyrexia is < 38.5, the rash is mild and they have no other symptoms it should be adequate to give 1g paracetamol and chlorphenamine (10mg slow IV/IM), recommence the transfusion at a slower rate, and monitor closely. o Fluid overload: dyspnoea, hypoxia, tachycardia, raised JVP & basal crepitations. O2, diuretic (e.g. furosemide 40mg IV) consider CVP line + exchange transfusion.

Potential examiners questions: Q. What type of blood may be used if crossmatch not available? A. Rh O Negative Q. When giving blood, what should you monitor? A. HR, BP, RR, Temp Q. What might you use to correct anaemia or blood loss? A. Packed red cells See also notes/OHCM

71

2.14 Blood transfusion: explain, take blood, fill in form (2006, 2008)
Mr David Doolan has had shortness of breath on exertion for two weeks. A recent blood test revealed a haemoglobin of 8g/dL. He needs 2 units of blood. Explain the procedure to him, take a blood sample, and fill in the form to request 2 units of blood. Introduce self. Elicit name, age and occupation. Establish rapport. Establish level of understanding: e.g. Hello Mr Doolan, I understand you have been experiencing shortness of breath over the last couple of weeks. pause, he may help you out, if not Has anyone spoken to you about what might be causing this?

EXPLANATION We have done some tests and have found that you are low on red blood cells. These are the cells that carry oxygen around your body, and this is most likely why you have been feeling short of breath. Normally your own body would make more of these cells but for some reason at the minute it is not doing this. We feel that the best way to treat this is to give you a blood transfusion that is to put blood from someone else into your veins Elicit concerns: Im going to go on and explain this in a bit more depth, but before I do, do you have any concerns. acknowledge these and let her know you will address them. Safety: Like all medical procedures, there are some risks involved with receiving a blood transfusion, but these risks are very low. They include; o Being given the wrong blood but there are procedures in place which makes this very unlikely (If we decide to go ahead, wed need to take a sample of your blood, to check what your blood type is) o Getting an infection from the blood or equipment again, risk is very low. Blood donors are carefully screened. E.g. Risk of Hep B is 1 in 500,000, Hep C 1 in 30 million, HIV 1 in 5 million o There is a very small risk of CJD, but donated blood has the white cells removed to reduce this risk Practicalities: Usually given through little tube directly into a vein in arm. Bag of blood can take up to 4 hours. You may need three bags so this could take a while. How will you feel during transfusion?: Most people do not feel anything. A nurse or doctor will observe you at regular intervals to check that you are feeling well. If you do start to feel unwell you should let a member of staff know straight away. o You might develop a temperature, chills or a rash. These reactions are usually mild and are easily treated with paracetamol, or by slowing the transfusion. Severe reactions to blood are extremely rare, but staff are trained to recognise and treat these. Time to think: How does that all sound so far? Offer an information leaflet, and time to think if time allows. Consent: Obtain written consent. Before we go any further I just need to check that you agree that it is OK for us to go ahead with giving you a blood transfusion (LTHT policy is that written consent should be obtained wherever possible; www.leedsth.nhs.uk/sites/board_meetings/agenda.../TransfusionPolicy.doc ). VENEPUNCTURE Explain: Id like to take some blood so that we can find out what your blood type and give you the correct blood. This would involve putting a band round your arm and inserting a thin needle into one of your veins. You may feel a sharp scratch as the needle is inserted. Its a simple and quick routine procedure. Do you have any questions? Double check patients ID (first & last name & date of birth) verbally and match with wristband. Ensure patient is sitting comfortably. Collect equiptment: Kidney dish, gloves, green needle, tourniquet, vaccutainer, blood bottle (below), chloraprep, cotton wool, micropore

72

Wash hands Apply tourniquet Find vein (mention techniques that may help) Clean with chloraprep in crosshatch & let dry Stabilize vein and insert needle at 15-30 Fill bottle appropriately and remove bottle Release tourniquet Remove needle and place cotton wool over wound site. Dispose of sharp immediately in sharps bin, dispose of other waste appropriately Label the blood bottle at bedside with surname, first name, date of birth, hospital number, date, and sign. Complete the relevant form (see below) FORM COMPLETION Fill in the form as shown below

73

ECG -perform and interpret II


Wash hands, introduce self (full name and position), check patient id (verbally & band) Explain purpose and gain consent

Procedure Explain that upper half of clothing will need to be removed, including bra if female. Explain need for chaperone Advise patient to remove watches and jewellery Provide patient with something to cover themselves Allow patient privacy to undress and get chaperone Advise to lay flat and keep limbs relaxed Plug ECG machine in an apply electrodes as shown Connect limb wires Connect chest wires working from V1 V6 Start machine Allow paper to run through Remove wires and pads Give pt privacy to dress Advise when/where results will be available

Interpretation Intro - Confirm patients name, age, and ECG date Rate (usual paper speed = 25mm/s) divide 300 by the number of big squares per R-R interval (e.g. 3 big squares = 100 bpm). Each big square is 0.2 seconds. Whole page = 10sec so x6 Rhythm mark position of 3 R waves on a piece of card and slide along Axis roughly speaking, if o I and II are positive, axis is normal. o I is ve = Right axis deviation o II, III ve = Left axis deviation P-wave absent = AF, sinoatrial block, disassociated with QRS = complete heart block, bifed = left atrial hypertrophy, peaked = right atrial hypertrophy P-R interval (start of P wave to start of QRS) normal range 3-5 small squares (0.12-0.2 sec). Prolonged = delayed AV conduction (1st degree HB). Short may be congenital, WPW. QRS normal = < 0.12s (3 small squares). If greater suggests ventricular conduction defects (eg BBB). Large QRS suggest ventricular hypertrophy. QT Measure from start Q to end T. Varies with rate. QTC = QT / sqrtR-R. Normal QTC = 0.38 - 0.42. Prolonged QT acute myocardial ischaemia, myocarditis, bradycardia, head injury, hypothermia, U&E imbalance, drugs etc. ST usually isoelectric. Elevation > 1mm = infarction, or depression < 0.5 = ischemia T abnormal if inverted in I, II, V4, V5, V6. Peaked in hyperkalaemia and flattened in hypokalaemia.

Potential Examiners Questions Q. What is atrial fibrillation? Supraventricular arrhythmia characterised by complete absence of coordinated atrial contractions. 74

Q. Interpret the following ECGs (overleaf) 1. Normal, 2. Anteroseptal MI, 3. Atrial Fibrilation, 4. Atrial Flutter, 5. Paroxysmal supraventricular tachycardia, 6. Atrial flutter, 7: VF

75

ECG 1:

ECG 2:

ECG 3:

ECG 4:

ECG 5: 76

ECG 6:

ECG 7:

77

2.14.1

Atrial Fibrillation potential examiners questions

Q. How would you diagnose AF? Regularly irregular pulse, ECG - absent P waves Q. What is atrial flutter? More organised and regular form of atrial activation = sawtooth pattern on ECG. Q. What are the symptoms of AF? May be asymptomatic, palpitations, dizziness, chest pain, anxiety, SOB Q. How is AF classified? (4 categories) 1) First detected AF = first clinical presentation, 2) Paroxysmal AF = recurrent episodes (not more than 7 days) that eventually self-terminate (within 48 hours), 3) Persistent AF = > 7 days, not self terminating. 4) Permanent AF = All attempts to restore sinus rhythm abandoned. Q. Name some causes or risk factors for AF Acute: Binge drinking, surgery, myocarditis or pericarditis, hyperthyroidism, drugs Cardiovascular disease: Hypertension, CHF, Valvular disease (esp mitral) , atrial septal defects, sick sinus. Pulmonary disease: COPD, emphysema, pulmonary hypertension, pneumonia, bronchiectasis, PE, lung cancer Other: Obesity, sleep apnoea, diabetes mellitus Q. What is the CHADS2 score? Clinical prediction rule for estimating the risk of stroke in patients with non-rheumatic atrial fibrillation (AF). C Congestive heart failure 1 , H Hypertension more than 160 Hg (or treated hypertension) 1 , A Age >75 years 1 D Diabetes Mellitus 1, S2 Prior Stroke or TIA 2; Annual Stroke Risk: 0=1.9%, 1=2.8%, 2=4.0, 3=5.9, 4=8.5, 5=12.5, 6=18.2 Q. What are the key principles of management for atrial fibrillation? Restore sinus rhythm, control ventricular rate, prevent recurrence, preventing thromboembolic events Q. When is immediate electrical cardioversion indicated? If rapid ventricular rate & either haemodynamically unstable or evidence of acute ischaemia or heart failure that doesnt respond to pharmacological measures. Q. What INR should you aim for before cardioconversion? If stable & rate controlled 2.0-3.0 three weeks before, if urgent & duration > 48 hours arrange transoesophageal echocardiography to prove absence of thrombus Q. Is cardioversion appropriate for the elderly? Ventricular rate control may be as effective as rythm control Q. What is the first line choice for pharmacological cardioversion? Propafenone, flecainide, or amiodarone. Q. What is commonly used to prevent recurrences? Side effects? Sotalol and amiodarone. Can prolong the QT interval, polymorphic ventricular arrhythmias. (esp sotalol). Q. What other drugs might you consider? In selected pts; beta blockers, calcium channel blockers, and digoxin Q. Whch drugs should be avoided in patients with heart failure? Flecainide and propafenone (risk of ventricular proarrhythmia and sudden cardiac death). Q. Which drugs should be avoided in patients with hypertension and left ventricular hypertrophy Sotalol (increased risk of QT prolongation). Use Amiodarone instead. Q. What is considered adequate rate control? 60-80 bpm at rest and 90-115 bpm during exercise. 78

Q. What drugs are comonly used for rate control? Beta blockers (e.g. atenolol, propranolol), non-dihydropyridine calcium channel blockers (verapamil and diltiazem), Digoxin (restrict to sedentary, less active pts). Q. When should aspirin be considered? When <65, no hypertension, no diabetes mellitus, no previous cerebrovascular accidents, no left ventricular dysfunction, no co-existent ischaemic or valvular heart disease, no hyperthyroidism, and no presence of prosthetic valves. Otherwise Warfarin. Q. If Warfarin is being used, what is the target INR? 2.0 - 3.0.

79

2.15 Urinalysis, prescribing (penicillin allergy)


You are told a patient is delirious and are given some urine to test with a dipstick. Talk through what you are doing; Wash hands and put on gloves Note for clarity, particles and colour Remove lid and check odour Check multistick container is in date Remove strip and replace lid Dip in urine for one second Replace lid on sample Compare test zones with chart TIME CORRECTLY Note any abnormalities Dispose of by folding strip inside gloves and binning Wash hands Record results

You notice that you have given penicillin to this patient, but they have a penicillin allergy. Speak to the patients granddaughter to explain what has happened and what to do from here. See section 1.5

80

2.16 ALS (2005, 2007)


A nurse calls you urgently and tells you that Mr X has stopped breathing and has no pulse. Walk though how you would manage this situation using the dummy.

Potential examiners questions Q. Discuss reversible causes of respiratory/cardiac arrest A. see above Q. Which rhythms are shockable? Whats PEA? Pulseless electrical activity In VF/VT, defibrillation must occur without delay: 360J (150-360J biphasic). Asystole and electromechanical dissociation (synonymous with pulseless electrical activity) are rhythms with a poorer prognosis than VF/VT, but potentially remediable. 81

Obtain IV access and intubation if possible. Look for reversible causes of cardiac arrest, and treat accordingly. Check for pulse if ECG rhythm compatible with a cardiac output. Reassess ECG rhythm. Repeat defibrillation if still VF/VT. All shocks are 360J. Send someone to find the patient's notes and usual doctor. If IV access fails, adrenaline, atropine, and lidocaine may be given down the tracheal tube but absorption is unpredictable. Give 2-3 times the IV dose diluted in 10mL 0.9% saline followed by 5 ventilations to assist absorption. Intracardiac injection is not recommended.

When to stop resuscitation No general rule. In patients without myocardial disease, do not stop until core temperature is >33C and pH and potassium are normal. Consider stopping after 20min if refractory asystole or electromechanical dissociation. After successful resuscitation: Investigations 12-lead ECG; CXR, U&Es, glucose, blood gases, FBC, CK/troponin. Transfer to coronary care unit/ITU. Monitor vital signs. Whatever the outcome, explain to relatives what has happened. When do not resuscitate may be a valid decision (UK DoH guidelines) If a patient's condition is such that resuscitation is unlikely to succeed. If mentally competent patient has consistently stated or recorded DNR request. If the patient has signed an advanced directive forbidding resuscitation. If resuscitation is not in a patient's interest as it would lead to a poor quality of life (often a great imponderable!). Involve patients and relatives in decision before emergency. When in doubt, resuscitate. Should be competent to: Initiate appropriate action o See ALS logarithm Prescribe appropriately, basic knowledge of the doses of COMMON & EMERGENCY drugs o Adrenaline 1mg in 10ml o Atropine 3mg in 10ml (muscarinic receptor antagonist) o Amiodarone 300mg in 10ml (antiarrythmic) o Always give 20ml saline flush to encourage into central circulation Atropine is muscarinic receptor antagonist used to inhibit the effects of excessive vagal activation on heart. Can temporarily revert sinus bradycardia to normal sinus rhythm and reverse AV nodal blocks.

82

2.17 Anaphylactic drug reaction, treat & BLS (2007)


Mr Bloggs has recently been given some penicillin despite his notes clearly displaying the fact that he has a penicillin allergy. His lips have swollen and he is having difficulty breathing. What do you do next?

Potential examiners questions Q. What is Anaphylaxis? A. Type I IgE-mediated hypersensitivity reaction. Release of histamine and other agents causes: capillary leak; wheeze; cyanosis; oedema (larynx, lids, tongue, lips); urticaria. More common in atopic individuals. Q. Common precipitants? Drugs: eg penicillin, and contrast media in radiology, latex, stings, eggs, fish, peanuts, strawberries, semen Q. Signs and symptoms? Itching, erythema, urticaria, oedema, wheeze, laryngeal obstruction, cyanosis tachycardia, hypotension 83

2.18 Drug chart post GI bleed (2008)


Mr Bloggs comes into A&E vomiting blood, complaining of dark foul smelling faeces, with a BP of 90/50 and a pulse of 120 bpm. What do you do next?

Possible examiners questions overleaf Well done. He is now haemodynamically stable and has had an endoscopy. 84

What would you want to do next? - Check his drug chart and stop NSAIDs if possible - Give omeprazole 40 mg IV after endoscopy (reduces risk of rebleeding and need for surgery). - Check FBC, U&E, LFT, and clotting daily. - Keep nil by mouth for 24h. Allow clear fluids after 24h and light diet after 48h, if no rebleeding How is risk assessed in cases of GI bleeds? The Rockall Score

Indications for surgery? - Severe bleeding or bleeding despite transfusing 6U if >60yrs (8U if <60yrs) - Rebleeding - Active or uncontrollable bleeding at endoscopy - Initial Rockall score >=3 or final Rockall score >6. Common causes of GI Bleeding? Mallory-Weiss tear, Oesophagitis, Oesophageal varices, Peptic ulcers, Gastritis/gastric erosions, Duodenitis, Malignancy, Drugs (NSAIDs, aspirin, steroids, thrombolytics, anticoagulants), Rarer causes of GI Bleeding? Bleeding disorders, Portal hypertensive gastropathy, Aorto-enteric fistula, Angiodysplasia, Haemobilia (bleeding in biliary tree), Dieulafoy lesion (arteriole in stomache erodes and bleeds), Meckel's diverticulum, Peutz-Jeghers' syndrome (hereditary intestinal polyposis), Osler-Weber-Rendu syndrome (hereditary hemorrhagic telangiectasia (HHT)) 85

2.19 Confirm death and document (2008)


You are a foundation year house officer. You have been called to the ward at 1am to confirm the death of Mr Smith. Tell the examiner what you would require and demonstrate how you would confirm death. CONFIRMING DEATH Details of death: Elicit the details of the patients death by consulting the nurse over the time of death and who witnessed it. Hospital notes: View the patients hospital notes, looking at the patients diagnosis, drugs, previous medical history, and resuscitation status. Patients ID: Confirm the patients identity by looking at his wrist band for his name and hospital number. Inspection: Dignity: Mention that you would draw the curtain/keep it drawn to maintain patient dignity General: Note general appearance, colour, lack of physical movement & absence of respiratory effort Pupils: Check for fixed, dilated pupils. Check for direct and consensual light reflex and note absence of these. Palpation Elicit lack of response to painful stimulus such as sternal rub or pressure on the orbits Palpate major pulses: carotid arteries & femoral arteries, checking both sides for 1 minute Auscultation Auscultate over the precordium listening for heart beats for 1 minute Listen over the lung areas for breath sounds for 3 minutes Documentation Confirm the patients death in the notes; Pupils fixed and dilated, no spontaneous respiratory effort, no response to deep stimuli, absent heart and breath sounds for 3 minutes. Time of death 0100, date of death 12-032010. Note patient has a pacemaker implant (remove before cremation). Signature. Bleep number. Extras Request an ophthalmoscope to examine the fundi for palisading (aka rail roading, tracking, boxcarring, retinal segmentation) see below;

Offer to complete a death certificate, inform the patients GP and the next of kin. Offer to inform the coroner if relevant

Potential examiners questions Q. When does rigor mortis set in? A. 3 hours after death Q. Legally, when can you fill in a death certificate? A. If I have seen patient in last 14 days and cause of death does not require referral to coroner. Q. When would you need to inform the coroner? A. Within 24 hours of admission, 86

suspicious, accidental, violent (including RTAs), suicide, operation, anaesthetic or procedure in last year (including endoscopy, chest drain, central lines etc.), alcohol or drugs, cause of death uncertain, due to industrial disease. Not seen by doctor in last 14 days. See over for death certification DEATH CERTIFICATION John Smith was a 85 year old retired teacher. Two years ago he was diagnosed with multi-infarct dementia. He has had hypertension and diabetes for the last 15 years. One evening his wife found him collapsed on the floor unable to move his right side. He was brought into hospital and admitted under your team with a diagnosis of stroke. Despite the efforts of your team he died 6 days later. You last saw him the day before he died and there was no sign of pneumonia or heart failure. Complete the death certificate as accurately as possible. See sample death certificate overleaf Use black ink Enter: o patients name, date of death, the age at death, place of death o Date last seen alive by you In this case there is no need for a post-mortem, so you would ring 3 and a I (a) Disease of condition directly leading to death: Cerebrovascular accident (b) Other disease or condition, if any, leading to I (a): Atherosclerosis (c) Other disease or condition, if any, leading to I (b): Hypertension(I checked & hypertension is thought to cause atherosclerosis). Employment: The disease or condition was not thought to be related to employment in this case. Your details: Sign, print full name, medical qualification (e.g. MBChB), date. Consultant: Enter the name of the consultant in charge of the patients care Notice to informant: Complete notice to informant, sign and date Coroner: There are no indications for referring to the coroner in this case.

Potential examiners questions: Q. What are the reasons for referring a patients death to the coroner? A. Within 24 hours of admission, suspicious, accidental, violent (including RTAs), suicide, operation, anaesthetic or procedure in last year (including endoscopy, chest drain, central lines etc.), alcohol or drugs, cause of death uncertain, due to industrial disease. Q. Can you fill in a death certificate if one of your colleagues (but not you) has seen the deceased in the last 14 days? A. No See death certificate overleaf.

87

88

2.20 Chest X-ray & prescribe


You are an FY1 in A&E. Mrs Johnson, a 66 year old lady, presents with confusion, a productive cough, purulent sputum, pleuritic chest pain, breathlessness, haemoptysis, and fever. Her ABG shows the following; pH 7.28, CO2 7.6, PO2 9.0, HCO3, and SaO2 89%. This is her CXR;

Q1. What is the likely diagnosis and what would your initial management be? Q2. What further investigations would you like to carry out? Start simple! Q3. How would you assess how severe her pneumonia is? Q4. What is the MEWS score? Q5. Assume this is community acquired pneumonia talk through the drug treatment you would prescribe depending on severity Q6. Assume this is hospital acquired pneumonia talk through the drug treatment you would prescribe depending on severity Q7. What else may be required? Q8. What is the commonest cause of community acquired pneumonia? Q9. What is the commonest cause of hospital acquired pneumonia? Q10. What are the potential complications of pneumonia? Answers 89

A1. Right middle lobe pneumonia. Her SaO2 is < 92% so she should be given high flow oxygen. A2. Further investigations: - Simple obs: Respiratory Rate, Blood Pressure, - Bloods: FBC, U&Es, LFT, CRP, atypical serology - pleural fluid maybe aspirated for culture - Bronchoscopy or bronchoaveloar lavage if patient immunocompromised A3. CURB65 score. If urea >7, CURB65 >=3 (confused, urea>7, age>=65), so manage in hospital. Describe as below A4. Modified early warning Score (see overleaf) used to determine risk of death and hence level of intervention reqd A5 & A6. Drug treatment for community acquired pneumonia depends on CURB65 score (see below). Drug treatment for hospital acquired different see Leeds Guidelines below (September 2009 guidelines). A7. If her BP is low (systolic<90 = shock) or dropping, she will require fluid resuscitation. She may need analgesia for chest pain (e.g. paracetamol 1g/6h or NSAID). She may require intubation or ventilatory support. She should be offered Pneumococcal vaccine in future. A8. Streptococcus pneumonia, haemophilus influenzae, mycoplasma pneumoniae in that order A9. Gram negative enterobacteria or staph aureus. A10. Pleural effusion, empyema, lung abscess, respiratory failure, septacemia, brain abscess, pericarditis, myocarditis, cholestatic jaundice

(from the Leeds THT Guidelines September 2009)

90

MEWS (Modified Early warning Score)

91

Histories
2.21 TIA, management, guidelines (2009)
You are a FY1 working in A&E. Mr George presents having had a funny turn. Take a history, suggest a diagnosis, and outline the management. Instructions to actor: You are Mr George and are 55 years old. Yesterday, you experienced a weakness in your right arm and your wife noticed that you slurred speech at the same time. It lasted 30 minutes and now you are fine. You have not had anything like this before. You are a taxi driver. You have not been to your doctor for many years. You didnt fall. Fill in the blanks, but stick to what would be a TIA diagnosis. Introduction and establish rapport. Confirm name, age, occupation Explain that you would like to ask them some questions about the problem theyve experienced HPC: Site of problem Onset of symptoms (sudden or gradual?) Character of problem (deteriorating or improving?) Radiation (did it go anywhere else?) Associated symptoms Timing (how long did it last?) Exacerbating/Relieving factors/Predisposing events (any history of trauma?) Specific questions Was the arm weakness associated with any sort of fit / jerking motion? (no) Did anyone witness the event? (yes, wife) What happened before and afterwards? (sudden onset & resolved quickly) Did the patient fall? (no) Other neurological symptoms o Headache, change in consciousness, Drowsiness / sleeping (no) o Changes in smell/vision/hearing, sensory disturbances, parasthesiae (no) Features of anterior cerebral artery involvement: Paralysis of opposite leg, sensory deficits, gait apraxia Middle cerebral artery As above + hemianopsia, amaurosis fugax, aphasia, agnosia Posterior cerebral artery Hemianopsia, visual agnosia, cortical blindness Vertebrobasilar Vertigo or nystagmus, facial numbness, dysarthria, loss of pain and temp, diplopia & visual field defects, bilateral spasticity PMH: Investigations and treatment to date, Previous stroke/TIA, BP, IHD, AF, PVD, Diabetes, cholesterol. DH: Including over the counter, Allergies FH: First degree relatives with a history of CVA, TIA etc SH: Occupation, smoking, alcohol, marital status, living accommodation, ADLs Review of systems: Cardio, Respiratory, GI, Urogenital, Metabolic Possible examiner questions Q1. What are your differential diagnoses? Q2. What is a TIA? Q3. What investigations would you like to do and why? Q4. How would you assess the risk of stroke in this patient? Q5. What features would suggest a carotid Doppler is NOT required? Q6. How would you treat this patient? Q7. What advice would you want to give this patient? Q8. What is the prognosis? (combined risk of stroke & MI?) (See also 2008 SIGN guidelines on following pages) 92

TIA Management A1. TIA, focal epilepsy, migraine aura, MS, peripheral neuropathy, hypoglycaemia A2. Sudden focal CNS phenomenon due to temporary occlusion lasting <24 h A3. Investigations based on Leeds guidelines; http://www.leedsteachinghospitals.com/sites/emibank/LGICDUProtocolTIA-13.07.swf o Obs: temp, BP, pulse o Examination: Full examination including a neurological examination o Investigations: FBC, U&Es, Glucose, Cholesterol, ECG, CXR (if smoker or chest complaint), urinalysis A4. Assess risk of stroke using ABCD2; * A Age: 60 years of age or older, 1 point. * B Blood pressure at presentation: 140/90 mmHg or greater, 1 point. * C Clinical features: unilateral weakness, 2 points; speech disturbance without weakness, 1 point. * D Duration of symptoms: 60 minutes or longer, 2 points; 1059 minutes, 1 point. * D Diabetic: 1 point. People with a score of 4 or more are regarded as being at high risk of an early stroke. A5. According to Leeds guidelines, a carotid Doppler is not required if symptoms are confined to those characteristic of posterior cerebral artery or vertebrobasilar artery involvement. A6. Treatment depends on risk assessment: o o If symptoms present during examination, refer to medicine (<65 years) or stroke unit (>65 years) If ABCD2 score >=4 aspirin (300 mg daily) started immediately specialist assessment and investigation within 24 hours of onset of symptoms secondary prevention measures introduced as soon as diagnosis confirmed If ABCD2 score < 4 as above, but specialist investigation should be within 1 week rather than 24 hours Secondary prevention: smoking cessation, antihypertensive, statin If AF, mitral stenosis, septal MI consider warfarin Carotid endarterectomy in carotid TIA if operative risk is good Further treatment details on following pages

o o o o o

A7. Avoid driving for >=1 month. Inform DVLA if multiple attacks or residual deficit. A8. Combined risk of stroke and MI following a TIA is ~9%/year (See also 2008 SIGN guidelines on following pages)

93

94

95

2.22 GI Hx IBD (2009)


Instructions to Candidate: A 24 year old man presents to the emergency department with a history of bloody diarrhoea. You are the FY1 on call with the medical team. Please take a history of the presenting complaint with a view to making a diagnosis. Actor: I am a 24 year old Engineering student, normally fit and well. I have been revising hard for my finals and am very stressed. During the last 3 weeks I have had bloody diarrhoea. I open my bowels once or twice an hour and I am passing semi formed/loose stools with blood and mucus, mixed and separate to the stools. Have lost 5kg and am too tired to play squash, which I normally do twice a week. Tried eating normally but felt too unwell most of the time to manage to do anything. No relationship between diet and diarrhoea. Last week developed red, painful lesions on my shins, and am feverish. No recent trips abroad and no-one in my family has anything similar. No risk factors for food poisoning. Not on any medication and only drink 1-2 pints a week. Smoke 10 cigarettes/day. No-one in my family has bowel disease. Am really worried about health and exams. I do not feel well enough to go out. Appropriate introduction (full name and role) Explains purpose of interview Establishes duration of the illness Establishes the normal bowel habit of the patient Establishes what the patient means by diarrhoea, i.e. frequency and volume of stool Establishes how often the patient is opening their bowels in a 24hr period Establishes the consistency of the motion Establishes the presence of blood and mucus in the stool Establishes/excludes weight loss Establishes associated GI symptoms vomiting and abdominal pain Asks about systemic features Establishes any other risk factors for diarrhoea travel, recent contacts, food poisoning Establishes/excludes and positive family history of IBD Elicits patients concerns and responds appropriately Uses an appropriate questioning technique Avoids or explains jargon Summarises history back to the patient, including concerns Systematic organized approach Makes a reasonable attempt at the diagnosis (Inflammatory Bowel Disease)

Instructions to Candidate: Below are some test results. What is the likely diagnosis? Hb ESR CRP WCC Albumin 11.5g/dL (13-18) 25 mm/L (<20) 15 mg/L (<10) 15 x 109/L (4-11) 20 g/L (35-50)

Instructions to Candidate: The diagnosis is Crohns, how is it managed? Mild attacks are treated with Prednisolone 30mg/d PO for 1 week, the 20mg/d for 1 month. 2-4 week follow ups, reducing prednisolone by 5mg every 2-4 weeks with aim of stopping when parameters normal. Severe attacks: admit for IV steroids, nil by mouth and IV hydration (e.g. 1 L saline = 2L dextro-saline /24h + 20 mmol K+), then hydrocortisone 100mg/6h IV, topical steroids for rectal disease, metronizadole 400mg/8h. If improving after 5d transfer to oral prednisolone 40mg/d Additional therapies include: Azathioprine, Methotrexate, Surgery (50-80% need =>1 op in life), Infliximab effective at reducing remissions. 96

Instructions to Candidate: Explain probable diagnosis, investigations & management to the patient (see 1.17 & 1.18)

2.23 Renal calculi Hx (2007, 2009)


Instructions to candidate: You are an FY1 attached to a surgical firm. Please take a history from this 24 year old female patient with a view to making a diagnosis. You should be prepared to discuss the patients management. Instructions to actor: I am 24 years old and am normally fit and well. I have come to the emergency department with severe left sided pain in my lower back. I have had the pain now for eight hours and it has gradually increased in intensity and is now unbearable. The pain comes and goes in waves and does not radiate. I have had two similar episodes in the last year, but they were not as severe. I have not had any symptoms of urinary infection, fever, rigors, or pain on passing urine. Appropriate name and introduction (full name and role), explain purpose of interview HPC: Establish duration of present illness Establish left sided loin pain Establish character and nature of the pain Establish history of two similar but less severe episodes Exclude symptoms of UTI (frequency, dysuria) Exclude haematuria Ask about fever, rigors, nausea, vomiting Ask about passage of renal stones PMH: Ask about metabolic illnesses (hyper/hypo calcaemia, hyperparathyroidism, neoplasia, sarcoidosis, addisons, cushings). Ask if any previous renal problems. DH: Ask about drugs (precipitating drugs include diuretics, antacids, acetazolamide, corticosteroids, theophylline, aspirin, thiazides, allopurinol, vitamin C, D, and indinavir, lithium) FH: Ask if any family history ( risk of stones x3) SH: Ask about diet (chocolate, tea, rhubarb and spinach oxalate levels) Ask about job (can she drink freely? Is she getting dehydrated?) Ask about allergies Technique: Use an appropriate questioning technique, avoid or explain jargon Summarises history back to patient Ask about concerns, ideas, and expectations Question: What is your preliminary diagnosis? Make a reasonable attempt at diagnosis (Renal colic secondary to renal stones) Instructions to candidate: These test results have come back. Urea 9.5mmol/L (2.5-6.7), Creatinine 250umol/L (70-150), Calcium 4.9 mmol/L (2.12-2.65), Urine dip shows protein Q1. What are the causes of raised urea? 1) Pre-renal: a) increased hepatic production (high protein diet), GI haemorrhage, increased catabolism - trauma, major surgery, extreme starvation with muscle breakdown b) increased renal reabsorption - reduced renal perfusion, e.g. congestive cardiac failure, shock, severe diarrhoea c) iatrogenic - e.g. tetracyclines, corticosteroids 2) Renal: Acute or chronic renal failure 3) Post-renal: urinary outflow obstruction (as in this case) Q2. What is the significance of raised creatinine? Filtering of the kidneys inadequate Q3. What may have caused her hypercalcaemia? Malignancy, hyperparathyroidism, sarcoidosis, familial hypocalciuric hypercalcaemia Q4. What further investigations would you like to do? 24 hr urine for calcium, oxalate, urate, citrate, sodium, creatinine, stone biochemistry KUB (kidneys, ureter + bladder) XR (80% of stones visible)

97

Ultrasound to look for hydronephrosis or hydroureter Q5. How would you manage this lady? Analgesia (diclofenac 75mg IV or IM). IV fluids if unable to tolerate orally, antibiotics (e.g. cefuroxime 1.5g/8h) if infection, seek urological help urgently if evidence of obstruction: extracorporeal shockwave lithotripsy (ESWL), or percutaneous nephrolithotomy (PCNL). If no obstruction: fluids & sieve urine, <5mm pass spontaneously.

98

2.24 Thyrotoxicosis(2005)
Instructions to candidate: You are an FY1 on a general medical firm. Please take a history off this 48 year old female who has presented with weight loss. Instructions to actor: I am a 48 year old (female) hairdresser. I have lost over a stone in the last three months but I have not been trying to lose weight. My appetite has changed I am eating more than I used to but still losing weight. I am concerned because this happened to my mum before she died of cancer. My husband says that I am crabby and irritable lately. He also says that I am unable to sit still. My muscles ache a bit recently. Ive noticed my hand trembles slightly when Im drinking a cup of tea. I have some palpitations (Ive got AF). Im hot all the time and I sweat more than I used to. My stools are a bit loose than they used to be. My son says my eyes look a bit goggley. I get a bit of double vision when Im tired. My periods have become infrequent, but I put this down to the change. Introduction and establish rapport. Confirm name, age, occupation PC/HPC: Confirm weight loss and ask about onset, duration, amount & if intentional. Also ask about; o change in appetite o difficulty concentrating o muscle weakness o heat intolerance, excessive sweating o agitation or restlessness o diarrhoea/changes in bowel habit o eye changes/changes in vision o palpitations o tremor o difficulty swallowing or breathing o pretibial myxoedema o changes in periods PMH: Other autoimmune disorders: diabetes, Addisions, vitiligo, myasthenia gravis, cancer, thyroid problems, previous surgery DH: e.g. thyroxine SH: Alcohol, tobacco, exercise, diet, social circumstances, employment FH: Any FH of autoimmune disorders? Allergies

Potential examiners questions Q. What are the common causes of hyperthyroidism? Graves disease (due to autoantibodies directed at the TSH receptor. Also toxic multinodular goitre, toxic adenoma (Plummers disease), Iatrogenic, drug induced (lithium, amiodarone), transient thyroiditis (e.g. de Quervains). Q. What treatment options are available? Anti-thyroid drugs: Carbimazole (methimazole), propylthiouracil, or methimazole. Side effects: nausea, bitter taste, hepatotoxiciy, vasculities, agranulocytosis: warn patients to come for FBC if they develop sore throat etc. (0.5% of patients). Radioiodine: given as drink (not if pregnant or breast feeding females or planning to get pregnant in next 4 months), may worsen eye disease in Graves'. Avoid close contact with children and pregnant women. Sleep alone for a week. Surgical: Sub-total or near total thyroidectomy (98% cure rate). Complications: haemorrhage, hypoparathyroidism, vocal cord paralysis, hypothyroidism. Q. What is a thyroid storm? Exxagerated manifestation of hyperthyroidism precipitated by major stressors (20-50% mortality). Symptoms: palpitations, tachycardia, tachyarrythmia, cardiac failure, anxiety, agitation, high fever. Jaundice is a late and ominous sign.

99

Q. What is the immediate management of a suspected thyroid storm? ABC, IV infusion, beta blocker infusion, cooling blankets, paracetamol, propylthiouracil (blocks T4 production), iodine (blocks release of t4)

100

2.25 PVD - intermittent claudication (2006 12 min)


Instructions to candidate: You are an FY1 on a general surgical firm. Mr Clarke, a 58 year old plumber presents with leg pain. Take a history with a view to making a diagnosis and discussing management. Instructions to actor: You are a 58 year old plumber. Over the last few months you have been getting a cramp like pain in your left calf after walking. Originally you first noticed this about a year ago in both legs after walking about 5 miles. Since then it has come on at preogressively shorter distances until now, when it comes on after walking about 50 yards. It goes away after 10 minutes of resting. You smoke 50 cigarettes a day and have a fry for breakfast every morning. Youve previously had a heart attack and suffer from angina occasionally. Your father died of a stroke and your mum died of a heart attack in her 50s. Full introduction, explain purpose of discussion and develop rapport Establish patients name, DOB and occupation PC/HPC: Can you tell me more about the pain in your leg? SOCRATES Does the pain come on when you walk? How far can you walk before the pain comes on? When you stop walking, how long does it take for the pain to go away? How long have you had the problem? Has it got worse or better over time? How does it affect your lifestyle? Have you any pain in your leg or foot at rest? What relieves your pain? In a man, ask about erectile dysfunction (Leriches syndrome) Does the pain come on at rest? PMH: especially; angina, MI, strokes, TIAs, diabetes, hypertension, high cholesterol o Respiratory: Chest pain, SOB, o Cardiovascular: palpitations, ankle swelling o Musculoskeletal: loss of power o Integumentary system: loss of sensation o Neurological: loss of vision, speech problems o Previous operations and investigations FH: Cardiovascular / cerebrovascular / peripheral vascular disease SH: smoking, alcohol, diet, ADLs (does it interfere?), social support

Potential examiner questions: Q1. What investigations would you like to do? ABPI, Buergers test (<20 and filling < 15sec = severe ischaemia) Bloods: FBC (anaemia/infection),Glucose (diabetes), ESR/CRP (exclude arteritis), U&E (renal disease contrast contraindicated), lipids, ECG (cardiac ischaemia). Contrast arteriography or colour duplex imagining to assess extent and location of stenoses. Q2. What drug should you stop before doing angiography? Metformin to prevent metabolic acidosis Q3. How would you interpret ABPI results? >=1 normal, 0.9-0.6 = claudication, 0.3-0.6 = rest pain, <=0.3 = impending gangrene. Q4. What are the signs of peripheral arterial disease? 6 Ps Pain, pallor, pulseless, parasthesia, paralysis, perishing cold Q5. How would you manage this patient? Quit smoking, weight, supervised exercise programme, blood glucose control if diabetic, treat hypertension and high cholesterol (avoid B blockers). Percutaneous transluminal angioplasty, surgical reconstruction (arterial bypass graft (femoral-popliteal, femoro-femoral, aorto-bifemoral bypass grafts). Aspirin and warfarin help grafts remain patent. Sympathectomy (interruption of nerve supply), amputation. Q6. What is the treatment for acute limb ischaemia? Open surgery, angioplasty, surgical embolectomy (fogarty catheter), thrombolysis (e.g. tissue plasminogen activator 101

(tPa)), anticoagulate with heparin after either procedure. Be aware of reperfusion injury and subsequent compartment syndrome. Q7. How can you classify the severity of PV disease? Fontaine classification: I: Asymptomatic, II: Intermittent claudication, II-a: Pain free, claudication walking more than 200 metres, II-b: Pain free, claudication walking less than 200 metres, III: Rest or nocturnal pain, IV: Necrosis or gangrene.

102

2.26 Back pain (renal cause, 2007)


Instructions to candidate: You are an FY1 on MAU. A 35 year old joiner presents with back pain. Take a history with a view to making a diagnosis. Instructions to actor: You are John Smith, a 35 year old joiner. Last night you woke up with an excruciating pain in the right hand side of your back. It radiated down to your lower back. It was a constant pain that gradually got worse for about 2 hours then died down. Then in the morning it came back and lasted for 3 hours. I ended up getting some of my mums diclofenac which is what I think made it go away. I hadnt been doing anything out of the ordinary the day before. Im certain that I havent pulled a muscle or anything. Im not depressed. I dont have any saddle anaesthesia or incontinence. When I went to the loo in the morning it was very painful and I noticed some blood in my urine. I have no other medical complaints. My dad died of a ruptured AAA and my uncle has had kidney problems. PC/HPC: SOCRATES - Site (thoracic pain usually due to an organic cause, e.g. TB, osteoporetic crush fracture, myeloma) - Onset (What were they doing when the pain came on? May be acute (disc prolapse, crush fracture), insidious (malignancy, infection, inflammatory) or chronic) - Character (shooting down the leg sciatica) - Radiation (e.g. down the distribution of the sciatic nerve after lumbar disc prolapse) - Associated symptoms (systemic symptoms: ankylosing spondylitis (polyarteritis, dyspnoea), malignancy, renal failure) - Timing (is it chronic and unrelenting? (inflammatory), intermittent or relapsing? (disc disease)) - Exacerbating/Alleviating factors (mechanical pain may be exacerbated by exercise, inflammatory worse after exercise, spinal stenosis worse on walking, relieved by leaning forwards) - Severity what impact is it having on quality of life? Sleep? PMH: Consider also psychiatric disorders, especially depression which may be a result (or cause) of the pain. DH: What analgesics have they taken? How much do they help? Long term steroid use? SH: How does the pain limit functional activity? Time off work? Stress? Systems: Especially weight loss, fevers, sweats, polyarthritis, sexual dysfunction Red Flags: Saddle anaesthesia, bladder or bowel problems, gait change ICE: Ideas, concerns expectations. Question: What are your differentials? Some possibilities: Renal colic, pyelonephritis, renal infarction, muscular pain, pulmonary embolism, kidney tumour, pelvi-ureteric junction obstruction, dissecting abdominal aortic aneurysm Instructions to candidate: The following blood test results have just come back. Sodium = 140 mmol/L (135-145), Urea = 7.2mmol/L (2.5-6.7), Creatinine = 185umol/L (70-150), potassium 4mmol/L (3.5-5), calcium 3.7 mmol/L(2.12-2.65). What do you now think might be the cause of this patients pain? Renal Colic secondary to a calcium stone. What might be the cause of his hypercalcaemia? Malignancy, hyperparathyroidism, sarcoidosis, hyperthyroidism, Addisons, Cushings, lithium, Vit D excess. What investigations would you request? KUB (Kidneys, ureter, bladder) XR. US to look for hydronephrosis or hydroureter. 80% show up on XR, 99% on CT. (Other tests, parathyroid hormone, ESR, TFTs, ACTH stimulation test, 24h urinary cortisol.) How would you manage this patient? Analgesia (diclofenac 75mg IV or IM). IV fluids if unable to tolerate orally, antibiotics (e.g. cefuroxime 1.5g/8h) if infection, seek urological help urgently if evidence of obstruction: extracorporeal shockwave lithotripsy (ESWL), or percutaneous nephrolithotomy (PCNL). If no obstruction: fluids & sieve urine, <5mm pass spontaneously. 103

What helps prevent renal stones? fluid intake, if hypercalciuria, thiazide diuretic calcium excretion. If oxalate intake (tea, chocolate, nuts, strawberries, rhubarb, spinach, beans, beetroot). If urate urine alkalinization may help.

104

2.27 Fatigue - Chronic renal failure (2008)


Instructions to candidate: Mrs Jones is a 47 year old shop assistant who presents with fatigue. Take a history with a view to making a diagnosis. Be prepared to answer questions about causes and management. Instructions to actor: You are Mrs Jones, a 47 year old shop assistant. You have been very tired recently, but are reluctant to talk about the true reason. As a child you had recurrent UTIs which scarred your kidneys and ever since youve had problems bedwetting. This happens 2 or 3 times a week, you have to get up in the middle of the night. This disturbs your sleep, and you have an early start in the shop. You can not afford to be late, the job is not flexible. Full introduction, explain purpose of discussion and develop rapport Establish patients name, DOB and occupation PC/HPC o Onset & Duration o Exacerbating factors o Change through the day o Sleep quality and hygiene, Early morning waking, Snoring o Weight loss o Urinary frequency o Thirst o Bone pain, joint swelling o Recent colds or sore throat o SOB, Cough, haemoptysis o Worries, mood, suicidal thoughts, Anhedonia o Anorexia PMH: JADE TAB MARCH, esp. past UTIs, BP, DM, pruritus, restless legs, impotence, dyspnoea, ankle swelling DH: Prescribed, OTC, herbal etc SH: Smoking, alcohol, diet, working late, caffeine late at night? FH: Similar problems in family Allergies

Differential diagnoses? Renal failure, DM, DI. Others: DM, cancer, depression, anxiety, alcohol, recreational drugs, anaemia, hypothyroid, infections, menopause, insomnia, heart failure, antihistamines, sedatives, chronic fatigue. Investigations? FBC (Hb; normochromic, normocytic), ESR, U&E (urea, creatinine), glucose (DM), if Ca, PO, Alk phos - renal osteodystrophy, PTH hyperparathyroidism, Urine MC&S, urine PCI or 24 hr protein. Imaging renal US may be larger in CRF with DM (> 9cm), polycystic kidney, amyloidosis, myeloma, systemic sclerosis, asymmetric renal vascular disease. Consider DTPA scan. CXR cardiomegaly, pleural pericardial effusions or pulmonary oedema. Bone X-Rays may show osteodystrophy. Renal biopsy if cause unclear and normal sized kidneys. Causes of chronic renal failure? Common: Glomerulonephritis, DM, renovascular disease, BP, pyelonephritis, polycystic disease. Rarer: Myeloma, amyloidosis, SLE, scleroderma, vasculitis, haemolytic uraemic syndrome, nephrocalcinosis, gout, renal tumour. Tests? Hb (normochromic, normocytic), ESR, U&E (urea, creatinine), glucose (DM); Ca2+, PO, alk phos (renal osteodystrophy); PTH. Urine: MC&S, dipstick, urine PCI or 24h urinary protein. Imaging: Renal ultrasound to exclude obstruction and check renal size (usually small, eg <9cm, but may be normal or large with CRF in DM, polycystic kidney disease, amyloidosis, myeloma, systemic sclerosis, asymmetric renal vascular disease). Consider DTPA scan. CXR: Cardiomegaly, pleural/pericardial effusions or pulmonary oedema. 105

Bone X-rays may show renal osteodystrophy. Renal biopsy should be considered if the cause is unclear and there are normal-sized kidneys. Treatment? Refer early to nephrologist. Treat reversible causes: relieve obstruction, stop nephrotoxic drugs, deal with Ca2+ and cardiovascular risk: Treat BP, oedema, anaemia, diet restrictions as deemed necessary

106

2.28 BPH & Interpret bloods (2005, renal failure)


Instructions to candidate: You are an FY2 working in General Practise. Mr Jones is a 45 year old caretaker. Take a history with a view to making a diagnosis. Be prepared to answer questions about causes and management. Instructions to actor: You are Mr Jones, a 45 year old caretaker. You have a few different problems. The one thats bothering you most is problems when you urinate (hesitancy, frequency, poor urinary stream, terminal dribbling, incomplete evacuation, nocturia sometimes and double micturition). You had many UTIs as a child. You take NSAIDs for lower mechanical back pain. Full introduction, explain purpose of discussion and develop rapport Establish patients name, DOB and occupation PC/HPC ask about: o Dysuria, strangury, frequency (day and night), nocturia o Haematuria o Incontinence o prostatism: hesitancy, frequency, poor stream, terminal dribbling, incomplete evacuation, double micturition o abdominal pain o polyuria, polydipsia o cloudy offensive urine PMH: JADE TAB MARCH, esp. past UTIs, BP, DM, pruritus, restless legs, impotence, dyspnoea, ankle swelling DH: Prescribed, OTC, herbal etc SH: Smoking, alcohol, diet, working late, caffeine late at night? FH: Similar problems in family Allergies

Q. Present your history back to examiner. Q. Comment on the following U&Es sodium 135-145 normal potassium 3.5-5 raised urea 2.5-6.7 raised creatinine 70-150 raised Cl- 95-105 slightly low HCO3 22-30 normal 24 hr urine output 400-3000 normal A plasma creatinine > 300 suggests sever renal failure Q. How could you classify the degree of this patients renal impairment? Using the Cockcroft Gault formula it could be classified into stages 1 5, with 1 being normal and 5 being established renal failure. Q. List some possible causes of chronic renal failure. In this case previous renal damage as a child. Common: Glomerulonephritis, DM, renovascular disease, BP, pyelonephritis, polycystic disease. Rarer: Myeloma, amyloidosis, SLE, scleroderma, vasculitis, haemolytic uraemic syndrome, nephrocalcinosis, gout, renal tumour. Q. Is there anything you would like to change about the way this patient is being managed? Take him off NSAIDS as they exacerbate renal failure. Get his BPH under control. Contact the renal physician. Q. Treatments for BPH? Assess impact on life. PR exam. Tests: MSU; U&E; ultrasound. Rule out cancer: PSA, transrectal ultrasound biopsy. Consider: Transurethral resection of the 107

prostate (TURP). Transurethral incision of the prostate (TUIP), Transurethral laser-induced prostatectomy (TULIP). Drugs: blockers: eg tamsulosin 400g/d. 5 reductase inhibitors: finasteride 5mg/d PO. Saw palmetto said to help.

2.29 Diarrhoea - hyperthyroid (2006, 12 min)


Instructions to candidate: You are an FY1 on a general medical firm. Please take a history off this 48 year old female who has presented with diarrhoea. Instructions to actor: I am a 48 year old (female) hairdresser. In the last three months Ive been troubled by diarrhoea. I have to go about 5 times a day and its almost always watery. I have lost over a stone in the last three months but I have not been trying to lose weight. My appetite has changed I am eating more than I used to but still losing weight. I am concerned because this happened to my mum before she died of cancer. My husband says that I am crabby and irritable lately. He also says that I am unable to sit still. My muscles ache a bit recently. Ive noticed my hand trembles slightly when Im drinking a cup of tea. I have some palpitations (Ive got AF). Im hot all the time and I sweat more than I used to. My son says my eyes look a bit goggley. I get a bit of double vision when Im tired. My periods have become infrequent, but I put this down to the change. Introduction and establish rapport. Confirm name, age, occupation PC/HPC: Confirm diarrhoea and ask about onset, character, duration, amount, blood/mucous. Also ask about; o change in appetite o difficulty concentrating o muscle weakness o heat intolerance, excessive sweating o agitation or restlessness o weight loss and if intentional o eye changes/changes in vision o palpitations o tremor o difficulty swallowing or breathing o pretibial myxoedema o changes in periods PMH: Other autoimmune disorders: diabetes, Addisions, vitiligo, myasthenia gravis, cancer, thyroid problems, previous surgery DH: e.g. thyroxine SH: Alcohol, tobacco, exercise, diet, social circumstances, employment FH: Any FH of autoimmune disorders? Allergies

Potential examiners questions Q. What are the common causes of hyperthyroidism? Graves disease (due to autoantibodies directed at the TSH receptor. Also toxic multinodular goitre, toxic adenoma (Plummers disease), Iatrogenic, drug induced (lithium, amiodarone), transient thyroiditis (e.g. de Quervains). Q. What treatment options are available? Anti-thyroid drugs: Carbimazole (methimazole), propylthiouracil, or methimazole. Side effects: nausea, bitter taste, hepatotoxiciy, vasculities, agranulocytosis: warn patients to come for FBC if they develop sore throat etc. (0.5% of patients). Radioiodine: given as drink (not if pregnant or breast feeding females or planning to get pregnant in next 4 months), may worsen eye disease in Graves'. Avoid close contact with children and pregnant women. Sleep alone for a week. Surgical: Sub-total or near total thyroidectomy (98% cure rate). Complications: haemorrhage, hypoparathyroidism, vocal cord paralysis, hypothyroidism. 108

Q. What is a thyroid storm? Exxagerated manifestation of hyperthyroidism precipitated by major stressors (20-50% mortality). Symptoms: palpitations, tachycardia, tachyarrythmia, cardiac failure, anxiety, agitation, high fever. Jaundice is a late and ominous sign. Q. What is the immediate management of a suspected thyroid storm? ABC, IV infusion, beta blocker infusion, cooling blankets, paracetamol, propylthiouracil (blocks T4 production), iodine (blocks release of t4)

2.30 Elderly pt collapse (2007)


Instructions to candidate: You are an FY1 in A&E. Take a history of this patient who presented following a collapse. Instructions to actor: You are Bob Holden, a 55 year old unemployed plumber. You recently fell outside your house on the way to get some milk. You remember falling and did not loose consciousness and were not incontinent. No-one saw what happened. you remember hitting the ground. You felt a bit light headed before you fell and had some palpitations. You are a diabetic and are taking metformin. Your GP recently started you on Atenolol for your hypertension. You drink 5 pints every night and have been drinking a bit more since your wife died. Appropriate introduction and establishes rapport Elicit patients name, age & occupation Circumstances of the collapse: o Were there any witnesses? o Did they have any warning? (aura, chest pain, dizziness, palpitations) o What were they doing at the time? Where were they? Watching flashing lights? Just woken up? Straining on the loo? Standing for a long time? Coughing or emotionally excited? Loss of consciousness? If so how long for? Can you remember; falling, being on the floor, getting up? o How long did it take to come round? How long were you down for? Confused or headache afterwards? Did you hit your head? Did your whole body or a part of it shake? Did you bite your tongue? Did you pass any urine? Has this ever happened before? Do you have any ideas about what may have caused this? Ideas Concerns Expectations PMH: Previous falls, hypertension, epilepsy, CVA, angina, diabetes, TIA, Parkinsons DH: antihypertensives, hypoglycaemics, antiepileptics, warfarin, nitrates, allergies SH: Smoking, alcohol (units/week), recreational drugs, social circumstances, usual mobility, mobility aids, ADLs. FH: Anyone in family had similar problems? Any inheroited family disorders? Epilepsy or cardiac problems? Been abroad recently? (cerebral malaria) Systems review: photophobia? headaches? dizziness? bowels & bladder? Recent infections/fever?

Potential questions: Q1. Suggest some differential diagnoses. A. Recently started on beta blockers, hypoglycaemia, alcohol intoxication, vasovagal/situational/cough/micturition/carotid sinus syncope, epilepsy, stokes-adams attack, cardiac cause.... Q2. What investigations would you want to do and why? Obs temp, HR, lying and standing BP, glucose, GC, sats, RR. neuro exam, Investigations: look for injuries, ECG (consider 24hr monitor), FBC, U&Es, CRP, troponin at 12h, CT if focal neurology, GCS or head injury, X-Ray if fracture suspected, echo, ABG if GCS diminished, LFTs.

109

110

2.31 Painless jaundice - pancreatic cancer (2008).


Instructions to candidate: Please take a history from this patient presenting with jaundice. Instructions to actor: You are Mrs Mildrews, a 66 year old retired librarian. A few weeks ago your husband noticed you skin was yellowing. You have noticed dark urine, pale faeces and a generalised itch. You have not had any pain but you have felt nauseous. You have lost some weight but have not been trying to. You have noticed a lump in your RUQ. Make the rest up. Introduction and consent Establish name, age, and occupation HPC: Onset gradual or sudden? Duration? Progression? Who noticed it? Exacerbating / Relieving factors Associated symptoms; o pruritus, dark urine, steathorrea, fever/rigors, lethargy, bruising Gastrointestinal symptoms o appetite, change in diet, weight loss, nausea, voimiting, abdominal pain or distention, altered bowel habit, rectal bleeding, flatulence, dysphagia or odynophagia, regurgitation, indigestion/dyspepsia Investigations and treatments to date o imaging (US, CT, etc), upper GI endoscopy, ERCP, MRCP PMH: JADE TABS MARCH including; o previous jaundice or hepatitis o autoimmune diseases (chronic active hepatitis) o emphysema o bleeding tendency o previous blood transfusions DH: including immunizations, statins, paracetamol overdose, anti TB medication, sodium valproate, MAO inhibitors, halothane, antibiotics, steroids, OCP, chlorproamazine, sulphonylureas, gold FH: First degree relatives with inheritable GI disease eg; alpha-1-antitripsin deficiency, haemochromatosis, Wilsons disease SH: sexual and contact history, travel history, tattoos and piercings, occupation (eg health worker), recreational drug use, alcohol, smoking, home circumstances, ADLs. Systems review: cardiovascular, respiratory, urogenital, central and peripheral nervous system, musculoskeletal, metabolic.

Potential examiners questions: Q. What are your differential diagnoses? Most likely to be cancer of the head of the pancreas. Courvousiers law: in the presence of a palpable gall bladder, painless jaundice is unlikely to be caused by gall stones. Could also be; Pre-hepatic: haemolysis, Crigler Najjar syndrome, Gilbert syndrome Hepatocellular: Hepatitis A, B, C, D, E. alcohol, leptospirosis, autoimmune, cirrhosis Post-hepatic: intrahepatic: primary biliar cirrhosis, primary sclerosing cholangitis, cholangiocarcinoma, sepsis, hepatocellular disease. Extrahepatic: gallstones, benign stricture of CBD, lymphoma. Q. What investigations would you like to do? urine: MSU, bilirubin, urobilinogen. Bloods: FBC, reticulocytes, blood film, clotting, U&Es, LFTs, amylase, lipase, LDH, paracetamol levels, hepatitis serology (ABC), EBV and CMV serology, bld cultures,. Urgent USS abdo (dilated bile ducts, cirrhosis, mets). CT or MRI if abdominal malignancy suspected. Q. How is Ca head of the pancreas treated? < 10% suitable for radical surgery (pancreatoduodenectomy (Whipples)). Endoscopic or percutaneous stent may help jaundice and anorexia. Opiates may be required in large quantities, radiotherapy may help. Referral to palliative care is essential. 111

Q. What is the prognosis like? Mean survival < 6 months. 5 year survival < 2%.

112

2.32 Polymyalgia rheumatica (2005)


Instructions to candidate: You are a GP. The next patient is a 65 year old woman who presents with multiple falls. please take a history of the presenting complaint with a view to making a diagnosis. Instructions to actor: I am 65 years old and have been unwell for about 2 months. Ive fallen 5 times. I do not lose consciousness or go dizzy or have any aura or incontinence. I just feel that my legs do not work properly. I have also developed a dull constant headache principally over my temples but this has become more generalised recently. The headache is poorly relieved with paracetamol but I have not noted any particular exacerbating factor. I also get a very tender scalp, which I notice particularly when combing my hair. Over the last month I have developed a painful tender and stiff shoulder and upper thigh muscles. The stiffness is worse in the morning and sometimes I have to roll out of bed. I have NOT noticed any muscle weakness or wasting. I am worried I might have a brain tumour. Appropriate introduction (full name and role) Elicits patients name age and occupation Establishes reason for patients visit Circumstances of the falls: o Were there any witnesses? Did they have any warning? (aura, chest pain, dizziness, palpitations) o What were they doing at the time? Where were they? Watching flashing lights? Just woken up? Straining on the loo? Standing for a long time? Coughing or emotionally excited? o Loss of consciousness? If so how long for? Can you remember; falling, being on the floor, getting up? o How long did it take to come round? How long were you down for? Confused or headache afterwards? o Did you hit your head? o Did your whole body or a part of it shake? Did you bite your tongue? Did you pass any urine? Elicits the fact that theyve been having headaches. Characterises the headaches o site: bitemporal o radiation o character: dull, constant o exacerbating relieving factors o scalp tenderness Establishes pain, tenderness and stiffness in the shoulders and thighs Excludes muscle weakness and wasting Establishes/excludes joint involvement Establishes/excludes symptoms of giant cell arteritis o Jaw claudication, Limb claudication o Angina o Cerebrovascular symptoms o Mesenteric/abdominal pains o Visual loss o Anorexia, weight loss, malaise Elicits patients concerns, ideas and expectations and responds sensitively Appropriate questioning technique Avoids or explains jargon Summarises history back to patient, including concerns Makes a reasonable attempt at the diagnosis Systematic organised approach

113

Q. What investigations would you like to do? ESR (in PMR), FBC (normochromic normocytic anaemia), CRP (), LFTs, autoantibody screen negative. Temporal artery biopsy (can confirm but not exclude), DEXA scan (assess need for osteoporosis prophylaxis). Q. How would you treat this patient? Initially high dose prednisolone (40-80mg od) for 46 weeks. Reduce by 5mg every 2-4 weeks. When 15mg reached, reduce by 1-2mg month. Most patients require 1-2 years of steroids. Azathioprine and methotrexate may also be used if steroids do not control disease Q. Difference between PMR and Fibromyalgia? Both=muscle pain. PMR is inflammatory, FM is not (abnormal sensory processing) FM chronic, PMR usually resolves. PMR associated with GCA, FM not.

114

2.33 SLE (2006: Yr 4 station)


You are an FY2 working at a GP practice. The next patient is a 23 year old woman who has come to see you because of aching joints. Take a history with a view to making a diagnosis and discussing management. Actor: I am a 23 year old woman and work in IT as a programme developer. I was fit and well until three months ago. Initially I had joint pain, particularily in my fingers and toes, but more recently I have had swelling and severe pain in the small joints of my hands. This affects both hands identically and is associated with feeling generally unwell, tiredness and fever. I also have a rash on my cheeks and painful red eyes and chest pain when I breathe in. I have not had any large joint swelling or pain, and no back pain. I am generally otherwise well and have no bowel or urogenital symptoms. I am on the OCP but no other medications. I do not know anyone else in my family whos had arthritis. I am upset because it has taken weeks to get this appointment and I am now finding it hard to use a keyboard. Appropriate introduction (full name and role). Elicits name age and occupation, explains purpose of interview. Establishes the presenting complaint and duration of symptoms Establishes characteristics of the arthritis o Symmetrical versus asymmetrical o Small joint involvement proximal or distal o Large joints involved upper and lower limbs o Spine cervical, thoracic, lumbar, sacroiliac o Features of acute arthritis swelling, pain, increased temperature, erythema o Early morning stiffness Establishes/excludes systemic associations o Eye/visual problems o Bowel upper/lower GI symptoms o Respiratory symptoms o Urogenital symptoms o Skin,hair and nail problems o General systemic fever, malaise, arthralgia, myalgia Establishes/excludes family history of arthritis Establishes medication history Established previous medical history particularly autoimmune disease Elicits patients concerns and responds sensitively Appropriate questioning technique Avoids or explains jargon Summarises history back to patient, including concerns Makes a reasonable attempt at diagnosis Systematic, organised approach.

Q. What investigations would you like to do? FBC (Hb/WCC/plts), C3+C4, ESR (but CRP normal), ANA +ve (95%), Xray (periarticular osteoporosis), U&Es (check for renal disease), urinalysis (protein, blood, casts) Q. How would you treat this patient? Refer to rheumatologist? High factor sun block. NSAIDS for arthritis. Hydroxychloroquine 00-400mg od if NSAIDs ineffective. Low dose steroids in chronic disease. High dose prednisolone + IV cyclophosphamide in severe flares. Q. What might be making this patients condition worse? OCP (also sulfonamides) Drugs that may cause SLE? include isoniazid, hydrazaline, chlorpromazine, phenytoin, procainamide, minocyline. Q. What is antiphospholipid syndrome and what are the features of it? occurs secondary to SLE in 20-30%. Caused by antiphospholipid antibodies. CLOT Coagulation 115

defect, Livedo reticularis (mottled skin), Obstetric: recurrent miscarriage, Thrombocytopenia (platelets). Rx = low dose aspirin or warfarin (INR 2-3)

116

2.34 Cystic Fibrosis (2006, 12 min)


Instructions to candidate: You are an FY1 working in MAU. Take a history from this 21 year old student who presents with worsening asthma. Take a history with a view to discussing management. See 1.2.1 for counselling Actor: You are George Best a 21 year old student. Your asthma has been getting worse recently and your inhalers do not seem to be making much difference. You are taking salbutamol 200ug/6hr, salmeterol 50ug bd, and beclometazone 400ug bd. You have had breathing problems ever since you were a young child. Your symptoms are; cough; wheeze; recurrent infections; bringing up a lot of phlegm (clear, white, yellow) a little haemoptysis; steatorrhoea; doc said you had gallstones. You are also troubled by nasal polyps; and a bit of joint pain. Youve noticed your fingernails look clubbed. Appropriate introduction (full name and role). Elicits name age and occupation, explains purpose of interview. Establishes the presenting complaint and duration of symptoms Asks appropriate questions regarding condition (CF PANCREAS) o C Chronic cough? Wheeze? Haemoptysis? o F Failure to thrive as a child? Osteoporosis as adult? o P Pancreatic insufficiency - Diabetes mellitus?, Steatorrhoea? Meconium ileus? Gallstones? Cirrhosis? o A Appetite decrease o N Nasal polyps? Sinusitis? o C Clubbing? Chest infections o R Rectal prolapse o E Electrolyte elevation (salty sweat) o A Atresia of vas deferens (infertility) o S Sputum production Establishes/excludes family history of CF Establishes medication history Established previous medical history Elicits patients concerns and responds sensitively Appropriate questioning technique Avoids or explains jargon Summarises history back to patient, including concerns Makes a reasonable attempt at diagnosis Systematic, organised approach.

Potential examiners questions. Q1. What are your differential diagnoses? CF, pneumonia, bronchiectasis, Kartageners syndrome, TB Q2. What is the pathogenesis of CF? autosomal recessive (1:2000 live births) affecting Caucasians. Caused by mutations in the CF transmembrane conductance regulator (CFTR) gene on chromosome 7. chloride secretion and sodium absorption across airway epithelium. Predisposes to chronic pulmonary infections and bronchiectasis. Q3. How is it diagnosed? Sweat test: sweat sodium and chloride >60mmol/L; chloride usually > sodium. Genetics: screening for known common CF mutations should be considered. Faecal elastase is a simple and useful screening test for exocrine pancreatic dysfunction. Q4. What other investigations would you like to do? Blood: FBC, U&E, LFTs; clotting; vitamin A, D, E levels; annual glucose tolerance test. Bacteriology: cough swab, sputum culture. Radiology: CXR; hyperinflation; bronchiectasis. Abdominal ultrasound: fatty liver; cirrhosis; chronic pancreatitis; Spirometry: obstructive defect. Aspergillus serology/skin test (20% develop ABPA, p160). Biochemistry: faecal fat analysis. Q5. How would you manage this patient? Best managed by a multidisciplinary team. Chest: Physiotherapy, antibiotics, mucolytics (eg DNase, ie Dornase alfa, 2.5mg daily nebulized). Bronchodilators. Gastrointestinal: Pancreatic enzyme replacement; vitamin supplements (A, D, E, K); ursodeoxycholic acid for impaired liver function; liver 117

transplantation, Other: Treatment of CF-related diabetes; screening for and treatment of osteoporosis; treatment of arthritis, sinusitis, and vasculitis; fertility and genetic counselling. Advanced lung disease: Oxygen, diuretics (cor pulmonale); non-invasive ventilation; lung or heart/lung transplantation. Q6. Prognosis? Median survival now >30yrs. Many live into 40s Q7. Counsel this patient on CF. See 1.2.1

118

2.35 Pseudomembranous colitis (2007)


Candidate: You are an FY2 working in a GP surgery. Mrs Hibbert, a 65 year old lady presents with diarrhoea. Take a history from her with a view to making a diagnosis and discussing management. Actor: You are a 65 year old lady who has been suffering with diarrhoea for the last three weeks. Six weeks ago you had a chest infection and your GP gave you some antibiotics- you cant remember exactly what they were called butit began with a C. You have lost a bit of weight in the last three weeks and have abdominal cramps. Your stools are green and watery. You have a bit of a temperature. You think you may have noticed a bit of blood in your stool. Ten years ago you had something the doctor called colitis, but this has never bothered you since. You are not on any other medications and are otherwise well. Introduce self to patient and establish rapport Establish name, age, occupation PC/HPC o Time, onset, duration, progression o Aggravating and relieving factors o Frequency of bowel opening o Colour, including blood, mucous or pus o Consistency (soft, watery, unformed, semi-solid) o Quantity o Flatulence o Tenesmus o Straining during defecation o Urgency or faecal incontinence o Abdominal, rectal or anal pain o Abdominal distention, lumps or masses o Associated GI symptoms: dysphagia, regurgitation, dyspepsia, haematemesis, jaundice o Associated systemic symptoms: thirst, weight loss, loss of appetite, nausea or vomiting o Investigations and treatment to date PMH: JADE TABS MARCH including; o Hx of gastrointestinal disease, surgery or radiotherapy, diverticular disease, surgery DH: laxatives, antibiotics, cimetidine, propranolol, cytotoxics, digoxin, OTC remedies. FH: First degree relatives with inheritable GI disorders such as coeliac, IBD, colorectal cancer, familial adenomatous polyps SH: Smoking, alcohol, recreational drugs, diet, travel, sexual history, living accomodation, marital status Systems review: Cardiovascular, respiratory, urogenital, nervous, musculoskeletal, metabolic.

Potential examiners questions Q1. Differential diagnoses? Pseudomembranous colitis, gastroenteritis, IBD, IBS, Malabsorption, Bowel cancer, Diverticular disease, pancreatitis, drugs, hyperthyroidism. Q2. What investigations would you like to do? Obs, U&Es, FBC, glucose, MC&S (C.diff toxin?), LFTs, Ca2+ (in malabsorption), TFTs, CRP, Blood cultures, AXR (obstruction, faecal impaction), sigmoidoscopy if not improving, colonscopy/barium enema if cancer suspected, coeliac screen. Q3. What might you look for on examination? volume satus, cachexia, mouth ulcers, clubbing, jaundice, rashes, pallor, thyroid mass, abdo tenderness, (+/- peritonitis, masses, distention, surgical scars), PR faecal impaction, pain, masses, stool colour, consistency. 119

Q4. How would you treat pseudomembranous colitis? Oral metronizadole 400mg tds (vancomycin 125mg qds). Q5. What complications may occur? Toxic megacolon, performation, high risk of spread to others.

120

2.36 GORD (2005)


Instructions to candidate: You are an FY1 on a GI firm. Take a history from this 47 year old man with a view to making a diagnosis and discussing further investigations and management. Actor: I am a 47 year old carpet salesman with a long history of peptic ulcer disease and gastro-oesophageal reflux. I have a six month history of intermittent difficulty in swallowing foods. I can manage most foods most of the time, but occasionally foods like bread and potatoes seem to stick behind the lower part of my breast bone. I have lost 2-3 kg but have no other associated GI symptoms. I am generally well and have had no major illness or admissions to hospital in the past. I am a non-smoker and drink 5-10 units per week. I am worried that I have cancer. Appropriate introduction (full name and role) Establishes reason for patients visit Establishes duration and nature of presenting complaint Establishes the level of dysphagia, i.e. pharynx, upper, mid or lower oesophagus Establishes the degree of dysphagia, i.e. solids or liquids Establishes the rate and nature of progression Establishes/excludes presence of regurgitation (any gurgling noises = pharyngeal pouch) Establishes history of peptic ulcer disease and reflux Establishes/excludes symptoms of GI bleeding maleana, haematemesis, blood PR Establishes/excludes associated GI features eg abdominal pain, nausea and vomiting, weight loss Excludes risk factors for oesophageal carcinoma Establishes any associayted features of systemic diseases or relevant previous medical history Elicits patients concerns and responds sensitively Appropriate questioning technique Avoids or explains jargon Summarises history back to patient, including concerns Makes a reasonable attempt at diagnosis Systematic, organised approach.

Q1. What are your differentials? Benign stricture secondary to long term GORD ; - Mechanical: malignant stricture, benign stricture, extrinsic pressure (cancer, lymph nodes, aortic aneurysm, goitre, pharyngeal pouch. - Motility: Achalasia, oesophageal spasm, systemic sclerosis, myasthenia gravis, bulbar palsy, Chagas disease. - Others: Oesophagitis, Globus hystericus. Q2. What investigations would you like to do? FBC (anaemia), U&E (dehydration), CXR (mediastinal fluid level), barium swallow (oesophageal stricture) +/- video fluroscopy, upper GI endoscopy and biopsy. ENT opinion if suspected pharyngeal cause. Q3.How are Achalasia and Benign oesophageal stricture treated? Endoscopic baloon dilitation or botox (Achalasia only). Q4. What is Barretts oespohagus? In chronic reflux oesophagitis, columnar gastric epithelium extends upwards replacing normal oesophageal squamous epithelium. Intestinal metaplasia occurs in these cells. Changes visible on endoscopy. There is a 40-fold risk of oesophageal adenocarcinoma. Management: If pre-malignant oesophageal resection generally advocated, especially in younger, fit patients; endoscopic mucosal ablation by epithelial laser or photodynamic ablation is used in others.

121

122

2.37 Discharge planning (2006).


Instructions to candidate. You are a doctor in elderly care. Mr Alfred, an 85 year old man was admitted with confusion and unsteadiness following a UTI and is due to return home today. He has rheumatoid arthritis. Discuss his discharge plans and home situation before letting him leave. First talk me through what you would want to do to prepare yourself. Actor: You are an 85 year old retired fighter pilot. You have RA but it does not stop you from doing your ADLs. You live alone as your wife died 5 years ago. You go down the pub once a week to meet your old RAF friends. The UTI made you confused but youve recovered now. You cook your own meals and someone comes round once a week to help with cleaning and laundry. Preparation Obtain patients notes and familiarise yourself with his PMH Read the minutes form the MDT meeting if you were not there Talk to the nurse and physiotherapist whove been looking after him about his progress and mobility Review his drug chart. Which drugs will he be taking home? Think about side effects, especially those just started History Introduce self and establish rapport Explain that before he goes home you would like to discuss his care at home Enquire about illness that brought him into hospital Enquire about ongoing illnesses and how they affect him Assess cognitive capacity using the MMSE o address to recall, age, time, year, 2 people, DOB, WWII, monarch, hospital, 20-1 Social situation o Who does he live with? o Can he cook or does he have meals of wheels? o Are there any stairs at home and if so, how many? o Has his home been modified? Occupational therapists involved? o Can he do his own shopping and cleaning o Can he wash, dress and feed himself? o Is he continent? Ascertain the level of support he obtains from family and friends Ask if he has any worries about returning home Ideas, concerns, expectations Discuss medication and determine compliance. Consider a dosette box. Are side effects an issue? Ensure an adequate package of care is in place o GP, occupational therapist, physiotherapists, and social services o meals on wheels o carers Arrange a follow up appointment in a clinic in 3 months time (for example) Let the patient know you are going to write to their GP Does the patient have any questions? Is someone coming to pick him up or will he need transport? Elicits patients concerns and responds sensitively Summarises situation back to patient, including concerns Systematic, organised approach.

123

124

2.38 Assessment post overdose

(2007)

You are a junior doctor in A&E and have been asked to see this 26-year-old female who took an overdose, was treated and is about to be discharged. Please take a focused history including a self harm/ suicide risk assessment. Introduce self and establish rapport Elicit patients name age and occupation Set the scene: I understand you took some tablets is that right? About the overdose sorry to go through this all again but... How do you feel about it all now? How long had you been thinking of it/planning beforehand? What tablets? How many? When? All at once/ staggered? With alcohol? What did you expect would happen? Did you go out to buy the tablets or have them in your house already? Did you take all that were available to you? Where were you? On your own? What had happened in the day? i.e. why then? Triggers e.g. arguments? Did you leave a note/ call anyone/ tell anyone what you were going to do? What happened? i.e. how did they come to hospital? Did you expect to be found? Want to be? Wish you hadnt been? What are the chances of you doing it again? If now unlikely, why? Has anything changed? Original triggers been resolved? Is there anything that might stop you? (protective factors) Are there tablets still at home? Is there anyone at home / any friends / family? assess level of social support Screen for depression: o Early morning waking/sleep disturbance o Loss of appetite/weight o Loss of energy and enthusiasm o View of environment, self and future o previous self harm/suicidal intentions o symptoms: mood, anhedonia, sleep, appetite, libido etc. duration. Family or personal history of mental illness Premorbid personality Drug, alcohol and smoking history Appropriate non verbal behaviour, appropriate questioning technique Summarises history back to patient, makes explicit empathic statements Systematic organized approach Ideas Concerns Expectations Summarise C - Check understanding, A Ask questions, L Leaflet, F arrange a Follow up Q. How might you assess if this patient needs psychiatric assessment? Modified SAD PERSONS scale Sex: Male = 1 , Age <19 >45yrs = 1, Depression or hopelessness = 2, Previous suicide attempts or Psychiatric care = 1, Excessive alcohol or drug use = 1, Rational thinking loss (psych or organic) = 2, Separated, widowed, divorced = 1, organised or serious attempt = 2, No social support = 1, Stated future intent (determined to repeat or ambivalent) = 2 Interpretation of Score < 6 may be safe to discharge (depending on circumstances) 6-8 probably requires psychiatirc consultation >8 probably requires hospital admission

125

3 Examinations
3.1 Gait (2005, 2007 previous Yr 4 station)
Examine this patients gait. Present your findings to the examiner as you go along. (Play one of the videos overleaf and ask the candidate to describe the gait). Introduction: Elicit name, age, occupation. Establish rapport Consent: Explain the examination to the patient and seek consent Exposure: Position and expose the patients arms and legs adequately Sitting: First observe the patient whilst sitting and note any postural abnormality or instability Rising: First ask if pt able to stand and walk. Ask patient to stand up. Note any difficulty standing up (e.g. truncal ataxia = lack of co-ordination of muscle movements in the trunk). Offer support if unsteady. Walking: ask patient to walk to end of room, turn round and return. Permit use of an aid if needed. Observe each stage of gait cycle and comment on start, rate, type of gait, arm swinging and how they turn around. Heel to Toe: ask patient to walk in straight line, putting heel of one foot directly in front of toe of the other. Observe if pt veers to one side (cerebellar lesion) or has wide based gait and loss of balance (truncal ataxia) Rombergs test: Ask pt to stand with feet together and arms by side. Explain you will catch them if they fall. Ask them to close their eyes. Negative if less stable with eyes open (Cerebellar disease), positive if less stable with eyes closed (posterior column disease).

Additional tests if appropriate Tiptoe walking: S1 lesion makes this difficult, Heel walking: L4 / L5 lesion makes this difficult If appropriate asking patient to run may accentuate findings The pattern of wear on the shoes provides information about symmetry and nature of abnormalities Excessive lateral wear = genu varum, Excessive medial wear = genu valgum 1. 2. 3. 4. 5. 6. 7.

1 2 3 4

= = = =

Normal wear Pes Planus Pes Cavus Also Pes Cavus

5= Splay foot 6. = Hallux valgus 7. = Hallux rigidus

Thank patient and ask if they have any questions State that you would also wish to do a full neurological examination

Possible questions: 1. Causes of cerebellar disease? MS, stroke, alcohol, space occupying lesions, anticonvulsants 2. Signs and symptoms of ? DANISH dysdiadochokinesia, ataxic gait, nystagmus, intention 126

tremor, slurred speech and hypotonia 3. Cerebellar versus vermis lesions? Cerebellar worse on ipsilateral side, vermis usually bilateral eg truncal ataxia 3.1.1 Abnormal Gaits with video links Abnormalit y Antalgic gait: Trendelenbe rg gait: Bow legs (genu varum) Knock knees (genu valgum) Flat feet (pes planus) In-toeing Out-toeing Toe-walking Hemiplegic gait Diplegic gait Neuropathic gait Myopathic gait Choreiform gait Ataxic gait Parkinsonian gait Foot drop Causes altered to reduce pain. Treat the cause. Contralateral pelvis falls when weight bearing. Polio, paresis of superior gluteal nerve after hip replacement, congenital dislocation of the hip, pain eg osteoarthritis Bowing of tibia up to 3 years (seldom needs treatment). Rickets or Blounts disease. Orthoses or surgical correction may be required. Bilateral common usually physiological cause. Unilateral: often pathological (eg fibrous dysplasia). Usually resolves before the age of 6 Toddlers have flat arch and fat pad usually disappears. May be due to collagen disorder (Ehlers-Danlos synd). Arch support provides symptomatic relief Metatarsus varus (big toes point in no treatment usually required unless > 5 years). Medial tibial torsion (usually self corrects), 3. Persistent anteversion of femoral neck (usually self corrects) . Usually age 6-12 months. Lateral rotation of hips. Usually resolves spontaneously. Common 1-3 years. Cerebral palsy, Achilles t. tight, Duchennes muscular dystrophy. http://library.med.utah.edu/neurologicexam/movies/gait_ab_08.mov Usually due to spastic cerebral palsy, stroke, demyelination http://library.med.utah.edu/neurologicexam/movies/gait_ab_09.mov Usually cerebral palsy http://library.med.utah.edu/neurologicexam/movies/gait_ab_10.mov Diabetes, Alcoholism, HIV, Toxin exposure, Metabolic abnormalities, Vitamin deficiency, Adverse effects of certain drugs http://library.med.utah.edu/neurologicexam/movies/gait_ab_11.mov pelvic girdle weakness, e.g. muscular dystrophy http://library.med.utah.edu/neurologicexam/movies/gait_ab_13.mov Huntingtons disease, antipsychotics, infections http://library.med.utah.edu/neurologicexam/movies/gait_ab_14.mov Trauma, stroke, TIA, Cerebral palsy, MS, chickenpox, Paraneoplastic syndromes, Tumor, Toxic reaction http://library.med.utah.edu/neurologicexam/movies/gait_ab_12.mov aka Festinating gait treatments include L-Dopa with dopa decarboxylase inhibitors, eg Carbidopa and benserazide High stepping gait may be caused by peroneal nerve palsy

127

3.2 Hand ((Rheum arthritis) 2005 (yr 5), 2006, 2008 (Yr 4))
Introduction: Wash hands Introduce self. Elicit name, age and occupation. Establish rapport. Consent: Explain what you would like to do and get consent. Expose: Ask the patient to expose their hands and arms to above the elbows. Pillow: Put a pillow on the patients lap and ask them to rest their hands on it.

LOOK Start with the elbows, and behind the ears in case you forget later (psoriatic arthritis). Examine both the dorsal and plantar surfaces of the hands, then examine from side with palms outstretched. Note any of the following o Nails: nail fold infarcts, clubbing, pitting, onycholysis o Skin: Rashes, thinning, bruising, palmar erythema, dupuytrens contracture, rheumatoid nodules o Muscle: wasting (dorsal interossei, thenar, hypothenar eminences) o Joints: Heberdens (DIPJ) & Bouchards (PIPJ) nodes. Swan neck or boutionnere deformities, z shaped thumb o Wrists: swelling, ganglion, vertical carpal tunnel release scars o Elbows: Psoriatic plaques, gouty trophy, rheumatoid nodules FEEL ASK: Before doing anything check if the patient has any pain! Peripheral pulses, muscle bulk, tendon thickening Skin: Assess temperature over forearm, hands and fingers. Sensation: Test light touch over index finder (median nerve), lateral aspect of thumb (radial nerve) and little finger (ulnar nerve). Request to formally assess pain using pin. Compare both sides. Joints: squeeze carpal and metacarpal joints, then each PIP and DIP in turn. Also anatomical snuff box (scaphoid #), tip of styloid process (de Quervains disease), head of ulna dorsally (exstensor carpi ulnaris tendonitis) MOVE Fingers: o Ask to spread fingers out & test power of extension and abduction o Hold each joint (MCP & IP) between thumb and forefinger flex and extend each joint separately. o Test precision grip strength (finger and thumb) and full grip strength (squeeze finger). Compare sides Thumb o Palms facing upwards test thumb abduction (median nerve) by asking pt to keep it pointing to ceiling against resistance, test opposition asking pt to touch thumb and little finger and trying to pull apart. Then test adduction (thumb across palm) and extension (thumb out to side) Wrist o Active & passive flexion (80) and extension (80). Radial (40) and ulnar (10) deviation, plus pronation and supination. FUNCTION Assess ability to perform tasks such as buttoning a shirt, writing a sentence, picking up a coin etc. SPECIAL TESTS 128

Tinels test tap over median nerve at wrist to reproduce symptoms of Carpal tunnel Phalens test hold wrists hyperflexed for 2 minutes to reproduce Carpal tunnel Froments sign ask pt to hold piece of paper between thumb and index finger. Thumb IPJ will flex if ulnar nerve compression Thank the patient and ask if they have any questions

Possible examiners questions overleaf Q. Name some signs of rheumatoid arthritis? Ulnar deviation, swan neck and boutionnieres, sub cutaneous nodules, Z thumb, metacarpal subluxation Q. Osteoarthritis? Heberden's nodes (DIP), Bouchard's nodes (PIP), squaring of hand Q. Psoriatic arthritis? Inflammation of PIPs and DIPs, nail pitting, onycholysis, hyperkeratosis, ridging, discoloration. Q. Treatments for each? RA: DMARDS (mtx, sulfasalazine, steroids, anti TNF, NSAIDS), OA (NSAIDS, steroid injections, glucosamine, chondroitin) PA: as with RA. Also education, physio, surgery etc. Q. What joints are typically affected by RA? MCPs & PIPs Q. What is carpal tunnel syndrome- causes, signs, symptoms? Entrapment of median nerve in carpal tunnel due to pressure. Usually idiopathic but causes include: hypothyroidism, diabetes, pregnancy, obesity, RA, acromegaly. Burning pain and tingling felt in thumb, index, middle and medial half of ring finger. Usually worse at night, relieved by shaking hand. (Wake + shake) Loss of sensation, wasting of thenar eminance, power of abductor pollicis brevis. Q. What is trigger finger? causes signs symptoms? Flexor tendor gets trapped in opening of its sheath. Middle and ring most commonly affected. Clicking noise when other fingers extended. Q. What is De Quervains disease (stenosing tenovaginitis?) Caused by repetitive abduction and adduction of thumb, e.g. gardening. causes pain at radial side of wrist.

129

3.3 Knee (2007, 2008: Yr 4)


Please examine this patient who is complaining of pain and stiffness in the knee. Report your findings to the examiner as you go along and make an appropriate diagnosis. Introduction: Wash hands, introduce self, elicit name, age, occupation. Establish rapport. Consent: Explain what you would like to do and obtain consent Expose: Ask patient to undress to undergarments LOOK Gait: assess gait by asking patient to walk to end of room and return. Note phases of gait, antalgic gait etc. Standing: with pt still standing look at; o Posture: alignment of shoulder, hips and patella o Joint: look behind knee for a Bakers cyst o Position: neutral, valgus (knock), varus (bow), flexion, hyperextension Lying: ask the patient to lie on the couch and comment on; o Skin: colour, sinuses, scars o Muscle: wasting, fasiculations o Joints: effusions, rheumatoid nodules, psoriatic plaques o Alignment: patellar alignment, tibial alignment o Position: fixed flexion deformity of the knee FEEL Check if the patient has any pain first Temperature, effusions, position. Effusions: milk the suprapatellar pouch, massage the medial aspect, then sweep the lateral and look for bulge Patella tap: milk the pouch again and tap down on patella. If it strikes femur and bounces back up = effusion Joints: flex knee to 90, feel joint line for tenderness, ligaments, synovial thickening, tibial tuberosity, femoral condyles, palpate popliteal fossa for Bakers cyst (may be related to arthritis) Mention youd like to do patellar apprehension test for patellar dislocation (knee extended, apply pressure medial side of patella, passively flexing knee to 30, look for lateral patellar movement or patient 'apprehension'.) MOVE Active: note limited range of movement or pain. Ask pt to bend knee (flexion) back as far as they can and straighten knee as much as they can (extension) Passive: flexion (140) and extension (-10). SPECIAL TESTS Collaterals Sag sign: apply varus and valgus forces at knee. Repeat with knee flexed 20 - excessive movement = torn collateral ligament Cruciate: flex knee 90 - if upper end of tibia sags = PCL tear Drawer test (you know this one); Lachmans test: flex knee to 20 similar to drawer test but more sensitive Meniscus McMurrays: warn pt may hurt a bit, flex knee as far as poss. Externally rotate tibia, push knee in and slowly extend. Repeat with internal rotation & pulling knee out. A painful click felt or heard = torn meniscus. http://www.youtube.com/watch?v=Irg3Cb4JaE8&NR=1 Apleys Grinding test: patient lying prone, knee flexed to 90. Rotate and compress knee joint. Pain suggests torn meniscus. Repeat but rotate and simultaneously pull leg up. Pain indicates ligament damage. http://www.youtube.com/watch?v=v_EscQlgx-U NEUROVASCULAR Pulses: dorsalis pedis and posterior tibial, Sensation and proprioception: compare both sides END PIECES State that you would also like to perform a hip and ankle examination 130

Thank patient and ask if they would like help re-dressing. Ask patient if they have any questions

Potential examiner question Q1. How can you differentiate a Bakers cyst from a popliteal aneurysm? A = absence of pulse

131

3.4 Hip (2005, 2006 previous Yr 4 station)


Please examine this patient who is complaining of pain and stiffness in the hip. Report your findings to the examiner as you go along and make an appropriate diagnosis. Introduction. Wash hands. Elicit name, age and occupation. Establish rapport. Explain what you wish to do an obtain consent Expose patient appropriately

LOOK General: Clues in surrounding area. Ask patient to stand examine alignment of shoulders, hips, ASIS and patella. Look from behind for scoliosis and gluteal wasting. Inspect from side for lumbar lordosis. Gait: ask patient to walk to end of room and return. Comment on use of walking aids, speed, heel strike, stance, push off and swing, stride length, arms swing. Gait eg. Trendenburg (hip drops on non weight bearing side). Trendelenburgs test ask patient to stand on one leg, resting their outstretched hands in yours. Positive if hip drops on non-weight bearing side. Lying down: observe the following; o Skin: scars, sinuses, pigmentation, skin creases o Muscle: wasting fasiculations o Swelling: Effusions o Position: shortened, rotated, fixed flexion deformity Measure: Apparent limb length (xiphisternum to medial malleolus) and true limb length (ASIS to medial malleolus). Apparent limb length discrepancies could be due to fixed adduction deformity of the hip. FEEL Palpate hip feel for temperature effusions and bony landmarks MOVE Note any limited range of movement or the reporting of pain. Normal flexion 130 Thomas test: Patient lying on back, place hand below lumbar lordosis. Ask patient to hold one knee flexed then try to straighten other leg. If unable to fully straighten, theres a fixed flexion deformity of hip on that side. Rotation knee flexed, internal rotation (move foot laterally) and external rotation (move foot medially) Abduction 45 Adduction - 30 END PIECES State that youd also like to examine the back and the knee. State that youd like to test for neurovascular deficits. Dorsalis pedis and posterior tibial. Compare sensation and proprioception. Thank patient and offer to help them re-dress. Ask the patient if they have any questions Potential examiners questions Q. What can osteoarthritis on the hip be secondary to? A: Perthes, Pagets, Rheumatoid Q. What causes Trendelenbergs sign? A: Deficient hip abductor function (ie gluteals) Q. What are the symptoms of Trochanteric bursitis? A: Pain over greater trochanter, worse when lying on affected side Q. What are the signs of an inferiorly anteriorly dislocated femur? A: Hip flexed, abducted and internally rotated. 132

133

3.5 Shoulder (hasnt come up previously)


Wash hands. Intro. Elicit age, name occupation. Establish rapport. Obtain consent Expose patient appropriately. Note if they have any difficulty removing clothing. LOOK: Inspect from front, behind, and sides. Comment on: o Abnormal posture, deformity or wasting. o Asymmetry of shoulders, winging of scapula o Bony prominences o Skin colour, bruising, sinuses, scars o Muscle wasting, lumps, effusions FEEL check if any pain first then: Skin: Compare temperature of both sides with back of hand. Joints: palpate for tenderness and effusions (start at SCJ -> ACJ -> Acromion). Also round scapula Tendons: ask pt flex biceps and palpate bicipital groove (biceps tendinitis) MOVE: Active Abduction: Ask patient to raise both hands sideways making plams touch in middle above head. If pain present establish angle. Midrange = supraspinatus tendonitis / partial rotator cuff tear. End of arc = acromioclavicualr joint arthritis. Adduction: Ask pt to move arm midline and accross body -normal range 50 Flexion: Ask pt to raise arms forwards (normal range 180) Extension: Ask pt to swing arms backwards (65) External rotation: Ask pt to flex elbow to 90, place arms tight to sides and externally rotate shoulder (45) Internal rotation: touch fingers with scapula from below, touch fingers behind neck from above Passive- test the above movements passively, checking for crepitus, pain, and limitation of movement POWER Deltoid: arms like chicken stop me pushing down (C5/C6, axillary nerve). Pectoralis major: Stop me from pushing up (C5/C6) Serratus anterior : Ask patien to push against a wall at waist height and look for winging of scapula (long thoracic nerve C5-C7) External Rotator Cuff (RC) Strength: Position the patient sitting, arms at his sides, elbows at 90 degrees. External rotation is attempted by the patient (the examiner applies internal resistance). Tests infraspinatus Internal RC Strength: Same as above, but the patient is attempting to rotate internally (and examiner resisting externally). Tests subscapularis SPECIAL TESTS Empty can test: Arms out, thumbs down. Get patient to push up against you. Tests for impingement. Apprehension test: Get patient to lie on their back. Abduct arm to 90, place hand underneath shoulder, and gently rotate shoulder by pushing forearm posteriorly. Instability will give sense that humeral head is about to slip out anteriorly and the patient will resist further movement. Thank the patient and let them know they can get dressed

134

Q1. What is impingement syndrome? IMpingement of rotator cuff muscles against coracoacromial ligament. Due to repetitive overhead activities eg swimming, occupational. Pain exacerbated by overhead activities, and at night when lying on affected side. Painful arc between 60 and 120 (but not if arm in full external rotation) Q2. What muscles make up the rotator cuff? Supraspinatus, infraspinatus and subscapularis. Q3. How can you tell a rotator cuff tear from impingement? Cant actively go beyond 60 but passively move arm to 90 abduction and patient will be able to hold it and continue above this level. Q4. What is frozen shoulder? Aka adhesive capsulitis. Pain loss of motion, stiffness. Thickening and contracture of capsule. More common in women aged 40-60. May subside after 9-12 months. Full ROM return by 18 months. Q5. How does shoulder normally dislocate? 95% anteriorly. What nerve is commonly injured and how do you test for this? Axillary test sensation in regimental badge area.

135

3.6 GALS (hasnt come up previously)


Screening Questions: Do you have pain or stifness in joints/muscles/back? Can you dress yourself completely without any difficulty? Do you have any problem getting up and down stairs unaided? GAIT: Assess for symmetry, smoothness, ability to turn quickly From behind look for: Muscle bulk and symmetry (Shoulder, Gluteal, Calf muscles) Straight spine, Level iliac crests Prominent or winged scapula Scaring, ulceration, effusions, swelling From side look for : Normal cervical and lumbar lordosis, thoracic kyphosis Abnormal knee flexion or hyperextension, hind foot deformity From front look for: In anatomical position shoulder bulk, elbow extension, quadriceps bulk and symmetry, knee swelling, foot arches and any mid foot or forefoot deformity SPINE: Feel: Spinous processes Move: Trunc flexion (two fingers on lumbar spinous processes), Head lateral rotation, flexion/extension, ear to shoulder, Temporomandibular joints ARMS: Look: Front observations Move: Hands behind head (shoulder abduction and external rotation), Elbows into side with palms facing downwards. Inspect palms of hands, power grip, squeeze fingers, precision pinch, squeeze MCP joints LEGS: with patient on couch assess: Full knee flexion and internal rotation of hip Patellar tap to exclude knee effusion Inspect soles of feet for callous formation Squeeze MTP joints How to record: Appearance Gait Arms Legs Spine Summarise, Check Understanding, Ask Questions, offer Leaflet, arrange Follow Up Thank the patient and let them know they can get dressed Movement

Possible examiners questions Q. What may cause cerebellar disease? MS, stroke, alcohol, space occupying lesions (tumour, aneurysm, abscess, granuloma, cyst), and anti convulsant medications. Q. What are the signs and symptoms of cerebellar disease? DANISH D Dysdiadochokinesia, A Ataxic gait, N Nystagmus (worse in direction of lesion), I Intention tremor, S Slurred speech, H Hypotonia. Cerebellar lesion causes symptoms worse on ipsilateral side, vermis lesion causes bilateral signs and symptoms.

136

3.7 Spine (hasnt come up before)


Wash hands. Introduce self. Elicit name, age and occupation Explain the examination and seek consent Check if the patient has any pain Ask the patient if they can undress to their undergarments. Note if they have problems with this - offer to help. GAIT Request to examine gait. Ask patient to walk to end of room and return. Comment on use of walking aid, speed, rythm, presence of limp, heel strike, stance, push off and swing, stride length, arm swing. LOOK Inspect with patient standing Skin: scars, sinuses, pigmentation, abnormal hair, unusual skin creases Muscle: wasting, fasiculation Posture: list (lateral deviation of spine), scoliosis (lateral curvature of spine), kyphosis (undue bending) Asymmetry: chest, trunk or pelvis FEEL Palpate full length of spine over spinous processes, paravertebral muscles and interspinous ligaments for any tenderness. Establish site of pain. MOVE ask patient to replicate your movements Extension: stand behind patient and ask to lean backwards as far as they can Flexion: ask patient to touch toes whilst keeping knees straight Lateral flexion: slide hand down leg on each side Rotation: fix pelvis with hands and ask to rotate chest from side to side (or do sitting) Expansion: Normal chest expansion is 7cm. <5cm suggests ankylosing spondylitis SPECIAL TESTS Schobers test: A mark is made at the level of the posterior iliac spine on vertebral column, i.e. approx L5. Place one finger 5cm below this mark and another finger 10cm above this mark. Instruct patient to touch toes. If increase in distance between two fingers < 5cm this is indicative of limitation in lumbar flexion. Straight leg raise test: Tests for lumbosacral nerve root irritation for example, disc prolapse. With patient on back: raise one leg - knee absolutely straight - until pain is experienced in the thigh, buttock and calf. Record angle at which pain occurs (<60 sciatica. Normal 70-120 degrees). Severe root irritation indicated when straight raising of leg on the unaffected side produces pain on the affected side. A central disc prolapse is likely with risk to the cauda equina and consequently, of bladder dysfunction. Pain before 30 degrees cannot be due to disc prolapse as the nerve root is not stretched within this range. Another explanation of nerve root irritation must then be sought. Bragards test: Perform sciatic stretch test - dorsiflex foot at this point of discomfort test is positive if additional pain results Lasegues test: Ascertain degree of straight leg raise that produces pain. Flex knee. Flex hip with knee flexed to same angle. Then straightne knee this will reproduce the pain. Femoral stretch test: Tests for irritation of L4 and above. Patient is positioned lying face downwards, and with the knee flexed, the hip is lifted into extension. Lumbar root irritation tension may cause pain to be felt in the front of the thigh and the back. Neurovascular examination: Cap refill in both feet, peripheral pulses (femoral popliteal, post tibial and dorsalis pedis, power, light touch and reflexes comparing both sides. Request to do a full neurovascular examination and also that you would like to examine the shoulder, elbow and wrist. Thank the patient and let them know they can get dressed Potential examiners questions Q. What are the most common sites for disc prolapses? L4/L5 and L5/S1

137

Q. What are the symptoms? L4/L5 weakness of hallux extension and sensation to outer aspect of leg and dorsum of foot; L5/S1 pain in the calf, weakness to plantarflexion and eversion of the foot, sensationover later aspect of foot and depressed ankle reflexes.

138

3.8 GI exam (2005, 06, 07, 08, 09)


Wash hands. Introduction. Elicit name, age, occupation. Establish rapport. Explain and get consent. Lay patient flat on couch and expose appropriately INSPECTION: General: Scars, distention, masses, stomas (ileostomy, colostomy), hernias, discolouration, catheters, gastric peristalsis or pulsations. Abdominal contour: flat, scaphoid or protuberant. Hands: Feel hands and inspect nails. Look in hands for; o Clubbing (cirrhosis, IBD, coeliac) o Leukonychia (cirrhosis, hypoalbuminaemia) o Koilonychia (iron deficiency) o Palmar erythema (chronic liver disease) o Duputyrens (cirrhosis) o Asterixis (liver failure) Face: Jaundice, anaemia. Brown freckles round lips (Peutz-Jeghers: hereditary intestinal polyposis syndrome) Mouth: central cyanosis (blue tongue), macroglossis (hypothyroidism or acromegaly), atrophic glossitis (iron, folate, B12 deficiency), Dry tongue (dehydration), Ulcers (crohns, coeliac), breath (ketones, ethanol). Body: Inspect for skin changes eg striae (Cushings), spider naevi, gynaecomastia and caput medusa (chronic liver disease) PALPATION: Lymph nodes: Feel for Virchows node in left supraclavicualr fossa (gastric carcinoma) Abdomen: Check for tenderness before proceeding. Warm hands and examine patient at same level. Watch patients face. Lightly palpate all areas area furthest from pain first (R&L Hypochondrium, Epigastrium, R&L Lumbar, Umbilical, R&L Iliac fossa, hypogastrium). The deep palpation feeling for masses and tenderness. Note size, shape, edge and consistency of masses. o Murphys sign pain over right hypochondrium with pressure on breathing in = acute cholecystitis. o Rovsings sign pressure in left iliac fossa results in pain in RIF = acute appendicitis o Abdominal distention 9Fs: fat, faeces, fluid, flatus, fetus, full sized tumours, full bladder, fibroids, falsepregnancy o RIF mass: appendix, colon cancer, Crohns disease, transplanted kidney, tuberculosis mass. Liver: Palpate from RIF, press down during inspiration using flat of hand. If liver edge felt describe (smooth, irregular), size (cms below costal margin), consistency (soft, hard), nodularity and tenderness. (normal=10cm) Spleen: Palpate from RIF. Should be between 9th and 11th rib. Only palpable if enlarged 2-3 times. Feel for a notch, size, consistency, tenderness. Kidneys: Ballott on inspiration. Note tenderness or enlargement. o Differentiating kidney and spleen: Spleen: notched edge, moves early in inspiration, dull to percussion, cannot get above it, enlarges towards RIF. Kidney: smooth shape, moves late in inspiration, resonant, can get above it, enlarges downwards. Aorta: Should be pulsatile but not expansile PERCUSSION: Liver, spleen, bladder, shifting dullness (from middle to edge = resonant to dull. Roll patient towards you with finger on dull spot. Wait 30 secs. Has dullness shifted? 139

Go back towards and beyond midline. Fluid thrill: patient places hand in middle of abdomen. Flick one flank and feel for thrill on opposite side. AUSCULTATION: Bowel sounds, aorta for bruits, renal for bruits. END PIECES: SHRUG I would also like to send off a stool sample if clinically relevant, check for hernias, perform a rectal examination, urinanalysis and examine the external genetalia. Thank the patient and let them know they can get dressed

In previous years this has been: kidney transplant 2007, 2008, 2009; dialysis, 2006; abdo pain 2005 Potential examiners questions overleaf Q. Name some potential causes for hepatomegaly. Large, smooth and tender: hepatitis, chronic heart failure, sarcoidosis, early alcoholic cirrhosis, tricuspid incompetence Large, hard and craggy: primary hepatoma or secondary tumours Large, smooth and non-tender: cirrhosis and lymphoma (n.b. small liver typical in late cirrhosis) Q. Name some potential causes for splenomegaly. Chronic myeloid leukaemia, myelofibrosis, lymphoma, infective (TB, Malaria), Glandular fever. Q. Name some causes of hepatosplenomegaly. Viral hepatitis, infectious monnucleosis, CMV, leukaemia, myeloproliferative disease, lymphoma, amyloidosis, acromegaly, SLE Q. Name some causes of enlarged kidney(s). Polycystic kidney disease, perinephritic abscess, hydronephrosis, malignancy Q. Name some causes of ascites. Transudate (protein < 30g/L): liver cirrhosis, constrictive pericarditis, cardiac failure, nephrotic syndrome. Exudate (protein > 30g/L): malignancy, acute pancreatitis, infective (TB, pneumococcal), Budd-Chari syndrome (caused by occlusion of the hepatic veins. Presents with classical triad of abdominal pain, ascites and hepatomegaly.) Q. Describe this patients abdomen

140

The patient has a large midline scar and a stoma in the left iliac fossa. This would be consistent with having had a Hartmanns procedure. However the bag on the patients left is not collecting solid matter and is therefore not a colostomy. It appears to be collecting urine and is therefore most likely a urostomy. The bag on the patients right appears to be collecting liquid faeces and its location is most consistent with a ileostomy (although a spout can not be seen). Q. What is Hartmans procedure? Excision of lesion in descending or sigmoid colon, over sewing of rectum and creation of a colostomy. Q. What is a loop ileostomy and what is it used for? A type of stoma where both proximal and distal segments of bowel drain on to the skin. May be used to protect and anterior resection (typically reversed after 6 weeks) Q. Complications of stomas? Fluid loss, odour, ulceration, leakage, stenosis, herniation or prolapse, ischaemia, poor B12 absorption, sexual and psychological problems.

141

3.9 CVS - identify murmur (2005, 06, 07, 08, 09)


Wash hands. Introduction. Elicit name, age, and occupation. Establish rapport. Explain and seek consent. Sit patient at 45 and expose appropriately INSPECTION: Observe for abnormal breathing, scars, added sounds, pacemaker. Eg: midline sternotomy (CABG, valve replacement), lateral thoracotomy (mitral valvotomy), malar flush (mitral stenosis), added sounds. Check legs for great saphenous vein harvesting. HANDS: Feel the hands for any temperature change. o Look for; peripheral cyanosis, clubbing (endocarditis, cyanotic congenital health disease, atrial myxoma), osler nodes, janeway lesions, splinter haemorrhages (endocarditis), tar staining. o Feel the pulse: assess rate, rhythm, volume and character rate: brady < 60, tachy > 100 rhythm: regularily irregular (2nd degree heart block), irregularily irregular (AF) volume: low=low cardiac output, heart failure, aortic stenosis; high= thyrotoxicosis, CO2 retention, aortic regurgitation character: check if pain, then raise arm and feel pulse with palm. Collapsing pulse=aortic regurgitation, PDA; slow rising=aortic stenosis; bisferien = aortic stenosis with aortic regurg delay: radio-radial delay = aortic arch aneurysm; aorto-femoral delay=coarctation of aorta ARMS: Indicate that you would like to measure the patients blood pressure sitting and standing (of >20 in systolic or >10 in diastolic after 3 mins standing suggests postural hypotension). FACE: Eyes: anaemia, hyperlipidaemia (xanthelesmata, corneal arcus). Tongue for central cyanosis, dental hygiene (SBE), high arched palate (Marfansaortic dissection/aneurysm, mitral /aortic valve prolapse. NECK: Carotid pulses, JVP (< 4cm). Hepatojugular reflux (>15 seconds) = right ventricular failure. Rise in JVP during inspiration = constrictive pericarditis, also PQRST; o P Pericardial effusion/pulmonary embolism/pericardial constriction o Q Quantity of fluid increased (iatrogenic) o R Right heart failure or congestive heart failure o S Superior venal caval obstruction o T Tricuspid regurgitation/Tricuspid stenosis/Tamponade PALPATION: APEX: 5th intercostal space midclavicular line. May also be; Tapping (Mitral stenosis), Thrusting (Aortic stenosis), Heaving (Mitral/Aortic regurgitation), Diffuse (left ventricual failure, dilated cardiomyopathy). HEAVES AND THRILLS: Thrills (AS, VSD), parasternal heave (right ventricular hypertrophy). AUSCULTATION: Auscultate over aortic, pulmonary, tricuspid, and mitral areas. Time with carotid pulse. Listen for heart sounds, additional sounds, murmurs or rubs. o Aortic: 2nd intercostal space (ics), left sternal edge. Aortic stenosis. Check for radiation to carotids. o Pulmonary: 2nd ics,rse. Listen for pulmonary stenosis with daiphragm. o Tricuspid: 5th isc, lse. o Mitral: 5th ics, mid clavicular line. Listen for mitral stenosis, using bell (low pitch). Ask patient to lean over to left side and hold breath on expiration. Listen for mitral regurg using diaphragm and check for radiation to axilla. o Ask patient to sit forward and hold breath on expiration. Listen to tricuspid area for aortic regurgitation. Murmurs: note timing, intensity, site, character, pitch, radiation, and effect of respiration and position. o Timing: - systolic: ejection (AS, PS, ASD) or pansystolic (MR, TR, VSD) - diastolic: early (AR, PR), mid (MS, TS); - continuous: patent ductus arteriosus 142

Intensity: 1-6; 1=faint and hard to hear, 2=easily heard with steth, 3=loud with steth, no thrill, 4=easily heard with thrill, 5=heard over wide area and thrill, 6=audible without steth. o Site, Character: rumbling (MS), blowing (MR), harsh (AS), Pitch: high (AS), low (MR), Radiation: carotids (AS), lse (AR), back (PDA), Respiration: RILE: Right Inspiration, Left Expiration. Position (AR, MR) o Manoeuvres: valsava increases mitral regurg, squatting increases aortic stenosis Lung Bases: Crepitations, pleural effusion (left ventricular failure). Palpate for sacral oedema and ankle oedema. Pitting = heart failure, nephrotic syndrome, cirrhosis, malnutrition, severe anaemia. Non pitting = lymphatic obstruction, DVT, myxoedema End pieces: Palpate for peripheral pulses, ECG, CXR, dipstick uine, fundoscopy (hypertension), sats and temp Thank the patient and let them know they can get dressed Potential examiners questions overleaf Q. What causes S1 and S2? S1 = mitral and tricuspid valves closing, S2 = aortic and pulmonary valves closing. o Q. What causes a 4th heart sound? Atrial contraction into hypertrophied or non compliant ventricle; heart failure, MI,cardiomyopathy, hypertension. Q. What causes a 3rd heart sound? Normal in chldren/young adults or stiff dilated ventricle; heart failure, MI, cardiomyopathy, hypertension, mitral/aortic regurg, constrictive pericarditis. Q. Name the five signs of endocarditis? clubbing, splinter haemorrhages, murmurs, splenomegaly, microscopic haematuria, oslers nodes, roth spots, janeways lesions. Q. What are the causes of AF? Ischaemic Heart Disease, Rheumatic Heart Disease, thyrotoxicosis Q. What are the two causes of an irregularily irregular pulse and how could you distinguish between them without an ECG? AF or ectopics. Excercise the patient and ectopics will Q. Why does the radial and ventricular rate sometimes differ? Pulse deficit (i.e. some radial beats not palpable). Q. Whats the target INR for AF? for prosthetic valve? 2-3, 3-4 Q. How could you assess the risk of a stroke in a patient with AF? CHADS2 C Congestive heart failure 1 , H Hypertension more than 160 Hg (or treated hypertension) 1 , A Age >75 years 1 D Diabetes Mellitus 1, S2 Prior Stroke or TIA 2. If 0=Low Aspirin daily, If 1=Moderate - Aspirin or Warfarin Aspirin daily or raise INR to 2.0-3.0, if =>2 - Moderate or High Risk - Warfarin -Raise INR to 2.0-3.0, unless contraindicated. Q. What are the signs of mitral stenosis? malar flush, AF, Apex beat not displaced, Tapping Apex beat, LUB! de derrrr. Prominent left atrium on CXR Q. What are the signs of mitral regurguitation? Apex beat usually displaced, quiet 1st heart sound, pansystolic murmur, radiates loudly to axilla, 2nd heart sound not heard separately. Q. What are the signs and common causes of aortic regurgitation? Collapsing pulse, apex beat displaced, diastolic murmur follows second sound lub taaaarr. Causes: rheumatic heart disease, tertiary syphilis, endocarditis, connective tissue disease (Marfans etc).

143

Q. What are the signs and common causes of aortic stenosis? Slow rising pulse, low volume pulse, low pulse pressure, JVP not elevated, apex beat forceful but not displaced, ejection systolic murmur. Causes: degenerative calcification, rheumatic heart disease, can be congenital. Q. What are the different types of heart vale replacements? Biological (eg porcine xenograft) or mechanical prosthetic: ball and cage (eg Starr Edwards) or bi-leaflet (St Jude) Q. What are the complications of an MI? Sudden death of PRAED sudden death, pump failure, rupture of papillary muscle or septum, aneurysm or arrythmias, embolism, Dresslers syndrome (pericarditis that develops 2 to 10 weeks after MI) Q. What are the signs and symptoms of acute LVF? Patient will look ill, cold clammy peripheries, frothy blood stained sputum, orthopnoea, wheeze, tachycardia or AF, systolic hypotension, cardiomegaly, 3rd & 4th heart sounds, right sided or bilateral pleural effusions. Q. What might you use if a standard echo did not give you a good view of the mitral and aortic valves in suspected endocarditis? transoesophageal echo Q. How would you treat an acute MI? MORE GAS Morphine, Oxygen, Reassurance, Explanation, GTN, Anti-platelets, Streptokinase or PCI

144

3.10 Respiratory (bronchiectasis 05, 07, 08, lobectomy 06, ? 09)


Wash hands. Introduction. Elicit name, age, and occupation. Establish rapport. Explain and seek consent. Sit patient at 45 and expose appropriately

INSPECTION: General: O2 masks, nebulizers, sputum pots etc. Breathing at rest, cough, wheeze, stridor. Scars: e.g. thoracotomy, Shape: barrel chest, pectus excavatum, pectus carinatum. Use of accessory muscles, intercostal recession (asthma, COPD), pursed lips, RR (normal = 16-25) tachypnoea > 25. Hands: Temperature (warm and well perfused?), Tremor (beta agonist salbutamol), peripheral cyanosis, tar staining, CO2 retention flap, clubbing. o Causes of clubbing: ABCDEF: Asbestosis, Bronchiectasis/Bronchial Carcinoma, Cystic fibrosis, Decreased O2, Empyema, Fibrosing alveolitis Pulse: rate and rhythm of radial pulse (bounding pulse = CO2 retention) Arms: Indicate you would like to measure BP sitting and standing Face and Neck: anaemia, cyanosis, JVP (cor pulmonale = right ventricle damage due to pulmonary pressure) PALPATION: Lymph nodes: Enlarged cervical/supraclavicualr lymph nodes = TB bronchial cancer Trachea: warn patient, look for tracheal deviation. Apex beat: normally 5th intercostal space mid clavicular line (displaced = pleural effusion, pneumothorax) Expansion: normal chest expansion > 5cm PERCUSSION: Chest: Upper, middle and lower, axillae and apices. Compare sides. o Stony dull = pleural effusion; dull = consolidation, fibrosis, collapse; hyper-resonant = COPD, pneumothx Vocal fremitus: ask pateint to say 99 and use ulnar border of hand. in consolidation & fibrosis, in pneumothorax, COPD, absent in collapse or effusion. AUSCULTATION: Chest: request pt breathe through mouth. Listen in all areas for breath sounds/added sounds; o wheeze (asthma, bronchitis); crackles (fine: heart failure, fibrosing alveolitis; coarse: bronchietasis, pneumonia, bronchitis) pleural rub: pneumonia, pulmonary embolism. o Vocal resonance 99 see vocal fremitus above o Repeat on back Examine for sacral oedema and ankle oedema. Check no pain when pressing. END PIECES: peak flow, CXR, sats, temp chart, sputum MC&S. Thank the patient and let them know they can get dressed Potential examiners questions Q. What might the following colours of sputum suggest: a) Black, b) Frothy pink, c) Rusty golden? a) aspergillosis, b) acute pulmonary oedema, c) pneumococcal pneumonia. Q. What signs might you expect in pulmonary fibrosis? chest expansion , vocal fremitus , Percussion dull, bronchial breath sounds, coarse crepitations. Q. What are the causes of a pleural effusion? Transudates (<25g/L) and exudates (>35g/L). - Transudates may be due to venous pressure (cardiac failure, constrictive pericarditis, fluid overload), hypoproteinaemia (cirrhosis, nephrotic syndrome, malabsorption), hypothyroidism, Meigs' syndrome (right pleural effusion and ovarian fibroma). 145

- Exudates mostly due to leakiness of pleural capillaries secondary to infection, inflammation, or malignancy. Causes: pneumonia; TB; pulmonary infarction; rheumatoid arthritis; SLE; bronchogenic carcinoma; malignant metastases; lymphoma; mesothelioma; lymphangitis carcinomatosis. See OHCM for more possible questions!

146

3.11 Lower peripheral vascular examination


Wash hands. Introduction. Elicit name, age, and occupation. Establish rapport. Explain and seek consent. Stand patient up to examine their legs and expose appropriately

INSPECTION Shape: swelling suggestive of oedema and venous compromise Varicose veins: location, distribution. Long saphenous (groin to medial malleolus), short saphenous (popliteal to lateral malleolus) Gaiter area: skin changes in lower third of leg just above medial malleolus o Venous stars: dilitation of superficial venules spreading from ankle o Eczema: above medial malleolus and lower calf o Ulcers: especially over medial malleolus o Ankle swelling o Pigmentation: brown discolouration = haemosiderin o Lipodermatosclerosis: fibrosis of skin and subcutaneous fat o Scars: previous vascular surgery? PALPATION: Temperature: note any temperature changes Veins: Feel along long saphenous and short saphenous for tenderness (ask first) or hardness (thrombosis) Junctions: feel saphenofemoral junction (4cm below& lateral to pubic tubercle) for a saphena varix (dilitation in saphenous vein as it joints femoral vein). Ask patient to cough, Impulse = saphenopopliteal incompetence. Peripheral pulses: Examine peripheral pulses to assess arterial blood supply SPECIAL TESTS: Tap test: Place one finger at bottom of long varicose vein and tap above this site. Impulse demonstrates superficial vein incompetence. Trendelenburgs: Ask pt to lie flat. Raise and milk leg. Occlude saphenofemoral junction and ask patient to stand. Remove fingers. If superficial veins refill = incompetence at saphenofemoral junction. Tourniquet test: Do as above but use tourniquet at different levels until veins below tourniquet remain collapsed (incompetent perforators lie above tourniquet) Perthes test: With patient standing and veins filled, place tourniquet around mid thigh and ask pt to stand up and down on the spot x10. If superficial veins collapse the deep veins are patent and communicatuing veins competent. If unchanged, both saphenous and communicating veins are incompetent. If the veins become more prominent, then the deep veins are occluded. Hand held doppler: Place the doppler on the vein. Squeeze the calf, blood will flow up through vein. Release the calf. Retrograde flow > 1 sec suggests valve incompetence. END PIECES 147

Request to perform an abdominal and pelvic exam (PV in females, genitalia in males masses may cause IVC obstruction). Thank the patient and cover legs. Acknowledge patients concerns. Summarise findings to examiner.

see over for questions Potential examiners questions Q. What are varicose veins? Dilated, tortuous, and prominent veins of the superficial venous system. Q. How common are they? Around 25% of women and 15% of men will get them. Q. What are the risk factors? Most varicose veins are primary - they have no identifiable cause. Incompetent valves play a role. Increasing age, prolonged standing, pregnancy, obesity, family history and the Pill all increase the risk. Pelvic masses, trauma, and previous deep venous thrombosis are also recognised causes. Q. What are the symptoms? Pain, cramps, tingling, heaviness and restless legs slightly commoner. Q. Name some complications of varicose veins. Haemorrhage, ulceration, lipodermatosclerosis, phlebitis, eczema, calcification. Q. Where is the saphenofemoral junction? It can be found by first locating the femoral artery between the anterior superior iliac spine and the pubic tubercle. The vein is medial to the artery and the saphenofemoral junction about two fingers' breadths below the inguinal ligament. Q. What is venous claudication? Acute, bursting pain on walking that is relieved by rest and leg elevation. This is called venous claudication. Q. What causes lipodermatosclerosis and what does it result in? Chronic venous hypertension when fibrin deposition results in progressive sclerosis of the skin and subcutaneous fat. Q. What are the treatment options? - Education: avoid prolonged standing; support stockings, lose weight, regular exercise - Injection sclerotherapy ethanolamine is injected and vein compressed ot avoid thrombosis. UNsuitable for perforation sites. - Laser coagulation especially for small varicosities and thread veins - Surgery is still the gold standard e.g. saphenofemoral ligation Q. How should thrombophlebitis be treated? With NSAIDs, not antibiotics. A hard, inflamed area above the ankle in the presence of varicose veins may well be inflammatory liposclerosis - not thrombophlebitis Q. What are reticular veins? Dilated non-palpable subdermal veins smaller than 4 mm in diameter. Q. How should lipodermatosclerosis be managed? Use of well fitted, below knee, graduated compression hosiery (normally Class 2) and a moisturiser (once or twice a day) and/or referral for treatment of varicose veins. Analgesic and anti-inflammatory medication may help the symptoms. Antibiotic treatment is unnecessary. Early referral to a vascular specialist is indicated to consider treatment for the underlying venous hypertension. Q. Hard, tender varicose veins indicate... thrombophlebitis. 148

149

3.12 Lower peripheral arterial examination


Wash hands. Introduction. Elicit name, age, and occupation. Establish rapport. Explain and seek consent. Expose patients legs and ask to lie down

INSPECTION Note colour, shiny skin, hair loss, ulcers, thinning of skin, gangrenous patches, oedema, amputations, loss of subctuaneous fat. Pressure points: check heel, malleoli, head of first metatarsl, lateral side of foot, toes, dorsum of foot for ulcers (if found describe size, shape, depth, edge, base) PALPATION Temperature: run back of hand along both limbs and soles of feet. Compare sides. Signs/symptoms of acute limb ischaemia; 6 Ps Painful, Pulseless, Pallor, paralysis, Parathesia, Perishing cold. Capillary refill: < 2 secs normal Pulses: posterior tibial, dorsalis pedis, popliteal, femoral. Compare sides. SPECIAL TESTS Guttering: elevate patients legs about 15 and look for venous guttering (arterial insufficiency) Buergers test: elevate the leg further and look for the angle at which it becomes pale (Buergers angle) < 30 indicates severe ischaemia. Sit patient up and hang legs over bed. Note time taken to return to normal colour. Reactive hyperaemia 2-3 mins suggests chronic lower limb ischaemia ABPI: ankle/brachial blood pressure using doppler. >=1 normal, 0.9-0.6 = claudication, 0.3-0.6 = rest pain, <=0.3 = impending gangrene. END PIECES Indicate that you would like to examine the rest of the peripheral vascular system (radial and carotid pulses, radio-femoral delay (coarctation of aorta), carotid bruit. State that you would like to perform a full cardiovascular examination and an abdominal examination (AAA). Thank the patient and cover their legs Potential examiners questions Q. What is the cause of intermittent claudication? Due to narrowing of the vessels due to atherosclerosis. Q. What is critical ischaemia? Continuous and aching pain in the leg at rest. Due to gross narrowing of vessels due to atheroclerosis. May be eased by hanging leg down. Q. Name some features of ischaemic (aterial) ulcers. Usually painful. Absenceof peripheral pulses. Deep with punched out edge. Found at pressure points or over tips of toes. Surrounding tissue cold. Discharge is serous or pus. Q. Name some features of venous ulcers. Found especially above medial malleolus. May be associated with lipodermatosclerosis and haemosiderin pigmentation. Shallow and flat with irregular pale purple or blue sloping edge. Base usually contains either fibrous or granulation tissue. Discharge often seropurulent in nature. See also OHCM!

150

3.13 Cranial nerves II, III, IV, & VI and Fundoscopy (2005, 2006, 2007, 2008)
Wash hands. Introduction. Elicit name, age, and occupation. Establish rapport. Explain and seek consent. Ask patient if their have any problems with vision and if they wear glasses /contacts (leave on/in). INSPECTION Eyelids: symmetrical? Eyelashes ingrowing (trichiasis)or loss/thining. Skin around eyes: any scars, inflammation, crusting, swelling. Drooping eyelid? Eyes: Any jaundice, redness of the eye, any foreign objects in the eye? Pupils of equal size? Strabismus? If yes, try the cover test. II OPTIC NERVE (Very Very Big Fat People Are Squidgey) Visual acuity: Use a Snellen chart or alternatively record smallest type that can be read at 30cm. If patient cannot read Snellen chart, bring to 3m. If patient cannot read at 3m ask to count fingers at 1m. If unsuccessful, test if they can see hand movements and if still unsuccessful ask if they can see the light from a pen torch at 1m. Mention that youd also like to test near sightedness using a book. Assess colour vision using Ishihara plates. Visual fields: test one eye at a time by confrontation using a white hat pin. Blind spot: test using the red hat pin Fundoscopy (save this for later) Pupillary light reflex: Direct and consensual one eye at a time. Relevant afferent pupillary defect. Accomodation reflex: Ask patient to look at object in distance then quickly focus on near object. Sensory inattention: hold fingers out to side and waggle one or both. Which moved? Occulomotor (III), Trochlear (IV), & Abducens (VI) nerves Ask pt to keep head fixed whilst following your finger. Make an H shape. Any pain, double vision? Nystagmus? If double vision are the images up and down or side by side? FUNDOSCOPY Explain: I would like to check your vision by having a look in your eyes. I will be using an opthalmoscope, which is really just a torch with a magnifying glass that will let me look a the back of your eye. It will not be painful, but I will need to get close up to your face to be able to do this. Would the be OK? Check the fundoscope is working and set to zero. Use right hand to view right eye and finger to focus. Ask the patient to focus on a distant object Red reflex: Focus on the pupil 12 inches form the patients eyes. Absence suggests presence of cataract. Let patient know you are going to get close to them and remind them to breathe and blink normally. Keep the beam pointing nasally to help focus on optic disc. Follow blood vessels from optic disc to periphery (or vice versa to find it first). Then look at all four quadrants. Look for microaneurysms, venous beading, arteriolar narrowing, AV nipping, copper or silver wiring, haemorrhages or exudates. Ask patient to look directly into the light to visualise the macula. Note its colour. Repeat for the other eye. Thank the patient. Acknowledge any concerns. Summarise findings to the examiner. Potential examiners questions Q. What might cause inflammation, crusting, and/or swelling around the eyelids? A. belpharitis, cysts, chalazions, styes). 151

Q. What might cause a drooping eyelid? A. 3rd nerve palsy, Horners Q. What are the different types of strabismus, and how would you test for them? A. Esotropia: eye turns inwards, Exotropia: outwards, Hypertropia: upwards, Hypotropia: downwards). In the cover test, the affected eye will readjust and look forward when the unaffected eye is covered. Q. When reporting the results from a Snellen chart, which number do you report first? A. The first figure is distance in metres, second is the number on line. 6/6 is normal. If patient can read line 9 at 6m then this would be 6/9. Q. What is the best thing to use when testing peripheral vision at the bedside and why? A. A white hat pin. peripheral vision is monochrome and central vision is colour. P.T.O. Q. Describe the location of the lesion for the following; (1) Unilateral vision loss, (2) Bitemporal hemianopia, (3) Right homonymous hemianopia, (4) Upper right quadrantanopia, (5) Lower quadrantanopia, (6) Right homonymous hemianopia with sparing of the macula.

(1) Unilateral vision loss, (2) Bitemporal hemianopia (compression of optic chiasm), (3) Right homonymous hemianopia (optic tract lesion), (4) Upper right quadrantanopia (lesion- lower fibres of the optic radiation) (5) Lower quadrantanopia (lesion- upper fibres of optic radiation), (6) Right homonymous hemianopia with sparing of the macula due to lesion of the optic radiation in the posterior part of the parietal lobe. Q. What would cause the blind spot to be enlarged? Give some possible conditions? A. A central scotoma. Possible causes; multiple sclerosis, methyl alcohol, nutritional causes -e.g. B12 deficiency, vascular lesions, glioma of optic nerve, glaucoma. Q. What is a Marcus Gunn pupil, what causes it, and what condition might it be associated with?

152

A relative afferent pupillary defect can be tested for by moving a torch quickly from pupil to pupil. If incomplete damage to afferent pathway (eg due to optic neuritis in multiple sclerosis), affected pupil will paradoxically dilate when light is moved from normal eye to abnormal eye. Due to reduced afferent input from the affected eye - the consensual pupillary relaxation response from the normal eye predominates. This phenomenon is also known as the Marcus Gunn sign. Q. Which nerves are involved in the accommodation reflex and in actual pupil constriction? A. a. II and III, b. III Q. How is diabetic retinopathy classified? Background retinopathy: Microaneurysms (dots), haemorrhages (blots), and hard exudates (lipid deposits). Pre-proliferative retinopathy: Cotton wool spots (infarcts), haemorrhages, venous beading. Proliferative retinopathy: New vessels form. Maculopathy: often not visible at early stage. Suspect if visual acuity.

153

3.14 Capacity and Consent


Instructions to candidate: Mrs Bloggs has a fractured femur and you have been asked to consent her for a hemi-arthroplasty. Assume that you are qualified to carry out this operation yourself. Be ready to answer questions regarding consent and capacity. Instructions to actor: Talk gibberish. You have absolutely no idea what is going on and you cant remember anything for any longer than about 30 seconds. Introduction Introduction, name, age, occupation Establish what the patient knows about their condition Try to ensure that the patient understands; o what the intervention is o why it is being proposed o the alternatives, including doing nothing o the principal benefits and risks (also regarding the alternatives) o the consequences of the intervention and alternatives Establish if patient can retain information long enough to weigh it in the balance and reach a reasoned decision Ensure the patient is not subject to coercion or threat Attempt techniques which may improve communication for example o Using diagrams o Asking for a friend or relative to be present o Improving the patients environment o Adjusting the patients medication Potential examiners questions Q. What is capacity, and what is competence? Capacity is the legal presumption that adults have the ability to make decisions, competence is a clinical determination of a patients ability to make decisions about his treatment. Q. How do you know if someone has capacity? Someone who has capacity must; be able to take in and retain information relevant to a decision and understand consequences of refusal believe the information be able to weigh up the information and make a reasoned decision Q. What might interfere with someones capacity? neurological pathology, eg. dementia, learning difficulties, psychiatric illness; acute states such as delerium, sever pain, alcohol and drugs (recreational and iatrogenic) Q. What is Gillick competence? If a child under 16 meets the criteria for capacity outlined above, they may be considered to have Gillick competence. It is advisable to include parent or guardian however. Q. Can a patients closest relative make a decision on their behalf if they lack capacity? No, but their views should be listened to. Q. If a patient does not have capacity, what should you do next? Look for an organic cause (e.g. UTI). Listen to their views. Involve colleagues, carers and relatives. Seek senior advice or contact the MDU/MPS. In extreme cases you may have to take it to the courts. Q. Supposing someone decides that they are happy not to have their broken hip treated, even if it might mean they could die could you over-rule their decision? If they displayed capacity the decision reached is irrelevant, so we could not always over-rule this decision, even if we do not feel it to be in a patients best interest.

154

Q. What does the Court of Protection do? It is part of the Office of the Public Guardian and has the power to decide whether a person has capacity to make a particular decision for themselves Q. Difference between dementia and delerium? Delerium is an acute confusional state, dementia is a chronic condition and entails progressive deficits in several cognitive domains.

155

3.15 Neuro exam - legs - periph neuropathy (6 min)


General Inspection Postural abnormalities and deformities Muscle wasting Fasciculation Abnormal movements Tone Check that the patient does not have any pain before starting Rotate leg at hip Flex and extend knee Rotate ankle through full range of movement Test for clonus (hold leg and dorsiflex foot quickly) Power Hips extension (L5, S1, S2) Hips abduction (L5, S1) Hips adduction (L2, L3, L4) Hips flexion (L1, L2, L3) Knee flexion (L5, S1, S2) Knee extension (L2, L3, L4) Ankle dorsiflexion (L4, L5, S1) Ankle plantarflexion (L5, S1, S2) Big toe extension (L5, S1) Can they go up on to toes and rock back on to heels? (ask) Can they hop? (ask) Coordination Run the heel of one leg up the shin of the other and viceversa Tap each foot on ground Reflexes Knee (L3,4) Ankle (S1) Plantar reflex (S1,S2) Sensation Light touch Pain Temperature
upper part of the upper leg (L2) lower-medial part of the upper leg (L3) medial lower leg (L4) lateral lower leg (L5) sole of foot (S1) Back of upper leg (S2)

Performed/Identi fied

Performed/Identi fied

Performed/Identi fied

Performed/Identi fied

Performed/Identi fied

Performed/Identi fied

Proprioception joint position sense Rombergs (stand feet together eyes closed proprioception) Vibration (using a tuning fork of 128Hz) 156

Observe gait If they have a particular complaint, e.g. difficulty in writing, watch them performing this task. See dermatome map 2 pages ahead

157

3.16 Neuro exam arms


General Inspection Postural abnormalities and deformities Muscle wasting Fasciculation Abnormal movements Tone Check that the patient does not have any pain before starting Flex and extend hand at wrist Flex and extend arms at elbow Rotate arm at shoulder Power Pronator drift (arms up, palms up) (UMN disease) Shoulders abduction Shoulders adduction Elbows flexion Elbows extension Wrist flexion Wrist extension Fingers - flexion Fingers - extension Abduction and adduction of fingers Abduction of thumb Coordination Finger to nose test Rapid alternating movements at wrist Tendon reflexes Biceps (C5,6) Triceps (C7) Brachioradialis (C5,6) Sensation Light touch Pain Temperature
posterior aspect of the shoulders (C4) lateral aspect of the upper arms (C5) tip of the thumb (C6) tip of the middle finger (C7) tip of the little finger (C8) medial aspect of the lower arms (T1) medial aspect of upper arms (T2)

Performed/Identi fied

Performed/Identi fied

Performed/Identi fied

Performed/Identi fied

Performed/Identi fied

Performed/Identi fied

Joint position sense Vibration (using a tuning fork of 128Hz) See dermatomes picture next page 158

159

3.17 Dermatomes

160

4 Formulary
Subgroup Analgesics Opiates generally Dose Fre q Rout e 24h max Common side effects Nausea, vomiting, constipation, dry mouth, biliary spasm. Larger doses; muscle rigidity, hypotension, respiratory depression. Also bradycardia, tachycardia, palpitation, oedema, postural hypotension, hallucinations, vertigo, euphoria, dysphoria, mood changes, dependence, dizziness, confusion, drowsiness, sleep disturbances, headache, sexual dysfunction, micturition difficulty, urinary retention, ureteric spasm, miosis, visual disturbances, sweating, flushing, rash, urticaria, and pruritus. see above Contraindications Acute respiratory depression, risk of paralytic ileus, raised intracranial pressure, head injury

Co-codamol non/opiate 30/500

1-2 tablets

46hr

PO

8 tablets

Codeine phosphate

30-60mg

46hr

PO/I M

240mg see above

see above. Caution with: hypotension, asthma and respiratory function, prostatic hypertrophy; shock; myasthenia gravis; obstructive or IBD; biliary tract disease; dose in elderly, hypothyroidism, adrenocortical insufficiency; convulsive disorders; cardiac arrhythmias; acute abdomen; gallstones; alcohol dependence See above. Caution with cardiac arrhythmias; acute abdomen; gallstones 161

Diamorphine opiate

1-5mg

14hr

IV/SC / IM Rout e PO/P R IM

n/a

see above, also anorexia, taste disturbance; syncope; asthenia, raised intracranial pressure; myocardial infarction

see above, also severe diarrhoea; toxic psychosis, CNS depression; severe cor pulmonale Contraindications NSAIDS generally; Contraindicted: previous or active peptic ulceration, pregnancy, severe heart failure, ischaemic heart disease, cerebrovascular disease, peripheral arterial disease, history of hypersensitivity to aspirin or NSAIDS, coagulation defects. Caution in: Hepatic impairment, Renal impairment, breastfeeding Diclofenac (150 mg daily) and ibuprofen increase risk of thrombotic events. see opiates above see NSAIDS above see opiates above see opiates above caution in alcohol dependence and hepatic impairment

Subgroup Diclofenac NSAID

Dose 50mg 75mg

Fre q 8hr 12h r

24h Common side effects max 150mg NSAIDS generally; GI 150mg disturbance inc. discomfort, nausea, diarrhoea, bleeding, ulceration. Also hypersensitivity reactions, headache, dizziness, nervousness, depression, drowsiness, insomnia, vertigo, hearing disturbances such as tinnitus, photosensitivity, haematuria. blood disorders, fluid retention, BP 240mg see opiates above 2.4g n/a see NSAIDS above see opiates above see opiates above

Dihydrocodeine opiate Ibuprofen NSAID Morphine opiate Oramorph10mg/ opiate 5ml Paracetamol non-opioid

30mg 300400mg 5-10mg 5ml 0.5-1g

46hr 68hr 4hrl y 4hrl y 46hr

PO PO IV/SC

PO/IV

4g

Pethidine opioid Tramadol opioid

25150mg 50100mg

prn 46hr

PO/S C/IM PO/I M/IV

rare; rashes, blood disorders (thrombocytopenia, leucopenia, neutropenia); hypotension; liver damage and renal on overdose 400mg see opiates above 400mg see opiates above. also diarrhoea; fatigue; retching, gastritis, flatulence; rarely: anorexia, syncope, BP, bronchospasm, dyspnoea,

see opiates above. also phaeochromocytoma see opiates above, also: uncontrolled epilepsy; acute porphyria

162

wheezing, seizures, paraesthesia, weakness; blood disorders Anti emetics Cyclizine antihistami ne 50mg tds PO/I M/IV 150mg Anti-histamines generally; drowsiness, headache, psychomotor impairment, urinary retention, dry mouth, blurred vision, and GI disturbance. Rare: hypotension, palpitation, arrhythmias, dizziness, confusion, depression, sleep disturbance, tremor, convulsions, hypersensitivity, blood disorders, liver dysfunction, angle-closure glaucoma. 24h Common side effects max 30mg extrapyramidal effects especially in children and young adults, hyperprolactinaemia, occasionally tardive dyskinesia; also, anxiety, confusion, drowsiness, restlessness, diarrhoea, depression, neuroleptic malignant syndrome, rashes, pruritus, oedema; cardiac conduction abnormalities; rarely methaemoglobinaemia 32mg constipation; headache; flushing; injection sitereactions; hiccups, hypotension, bradycardia, chest pain, arrhythmias, movement disorders, seizures; rarely dizziness, transient Anti-histamines generally: prostatic hypertrophy, urinary retention, angle-closure glaucoma, and pyloroduodenal obstruction. Caution in: epilepsy, acute porphyria

Subgroup Metoclopramide antiemetic

Dose 10mg

Fre q tds

Rout e PO/I M/IV

Contraindications gastro-intestinal obstruction, perforation or haemorrhage; 34 days after gastro-intestinal surgery; phaeochromocytoma

Ondansetron 5HT3 antag.

8mg

tds

PO/I M

QT interval prolongation

163

Prochlorperazine anti-emetic

5-10mg

tds

PO/I M

30mg

Subgroup Stemetil anti-emetic Buccastem anti-emetic Hypnotics/sed atives Diazepam anxiolytics

Dose 5mg 3mg

Fre q tds bd

Rout e PO/I M PO

24h max 30mg 24mg

visual disturbances Antipsychotics generally; Extrapyramidal, e.g. parkinsonian, dystonia, dyskinesia, akathisia, tardive dyskinesia, hypotension, interference with temperature regulation, neuroleptic malignant syndrome (hyperthermia, fluctuating consciousness, muscle rigidity, and autonomic dysfunction with pallor, tachycardia, labile blood pressure, sweating, urinary incontinence) Others; drowsiness; apathy; agitation, excitement, insomnia; convulsions; dizziness; headache; confusion; GI disturbances; nasal congestion; antimuscarinic symptoms, endocrine effects; blood dyscrasias; corneal/lens opacities, and purplish skin/eye pigmentation. Common side effects brand name for prochlorperazine brand name for prochlorperazine Drowsiness, lightheadedness; confusion, ataxia, amnesia, dependence; paradoxical aggression muscle weakness; occasionally: headache,

Antipsychotics generally; Caution in cardiovascular disease; Parkinsons disease, epilepsy, depression, myasthenia gravis, prostatic hypertrophy, angle-closure glaucoma. Respiratory disease, jaundice, blood dyscrasias, comatose states, CNS depression, and phaeochromocytoma; Hepatic impairment, renal impairment, Pregnancy, Breast-feeding

Contraindications brand name for prochlorperazine brand name for prochlorperazine

2mg 10mg

tds >4h r

PO IM/IV

30mg 30mg?

Respiratory depression; marked neuromuscular respiratory weakness including unstable myasthenia gravis; acute pulmonary insufficiency; sleep apnoea syndrome; severe hepatic 164

Temazepam anxiolytics

10-20mg

on

PO

40mg

Zopiclone anxiolytics

7.5mg

on

PO

7.5mg

vertigo, hypotension, salivation changes, GI disturbances, visual disturbances, dysarthria, tremor, changes in libido, incontinence, urinary retention; blood disorders, jaundice; skin reactions; on intravenous injection, pain, thrombophlebitis, and rarely apnoea; drowsiness and lightheadedness the next day; confusion and ataxia (especially in the elderly); amnesia may occur; dependence taste disturbance; nausea, vomiting; dizziness, drowsiness, dry mouth, headache; rarely amnesia, confusion, depression, hallucinations, nightmares; light headedness, incoordination; paradoxical effects GI irritation, nausea, epigastric pain, constipation or diarrhoea. Accidental overdose in children. nausea (can be reduced by administration with water, fruit juice or with meals), vomiting, flatulence, cramps, and abdominal discomfort

impairment; not for chronic psychosis; should not be used alone in depression or in anxiety with depression; avoid injections containing benzyl alcohol in neonates

as above

marked neuromuscular respiratory weakness including unstable myasthenia gravis, respiratory failure, severe sleep apnoea syndrome

Gastrointestin al Ferrous sulphate oral iron

200mg

od

PO

65mg iron

Lactulose osmotic laxative

15mL

bd

PO

lactose intolerance

165

Subgroup Lansoprazole PPI

Dose 15-30mg

Fre q od

Rout e PO

24h max

Common side effects

Contraindications may mask the symptoms of gastric cancer. avoid in pregnancy

Loperamide antimotility

2-4mg

prn

PO

Omeprazole PPI Ranitidine H2 receptor antagonists

20mg 150300mg 50mg

od bd qds

PO PO IM/IV

PPIs generally; GI disturbance, headache. Dry mouth, peripheral oedema, dizziness, sleep disturbances, fatigue, paraesthesia, arthralgia, myalgia, rash, pruritus. Rarely; taste disturbance, stomatitis, hepatitis, jaundice, hypersensitivity reactions, fever, depression, hallucinations, confusion, gynaecomastia, interstitial nephritis, hyponatraemia, blood disorders, visual disturbances, sweating, photosensitivity, alopecia, Stevens-Johnson syndrome, and toxic epidermal necrolysis. May risk of GI infections (including C diff). Lanzoprazole specifically: glossitis, pancreatitis, anorexia, restlessness, tremor, impotence, petechiae, and purpura; rarely colitis, raised serum cholesterol or triglycerides 16mg abdominal cramps, dizziness, drowsiness, and skin reactions including urticaria; paralytic ileus and abdominal bloating also reported 120mg see PPIs generally, also agitation and impotence 600mg H2-RAs generally: diarrhoea, 200mg GI disturb, LFTs, headache, dizziness, rash, tiredness. Rarely: acute pancreatitis,

conditions where inhibition of peristalsis should be avoided, where abdominal distension develops, or in conditions such as active ulcerative colitis or antibiotic-associated colitis see PPIs generally might mask symptoms of gastric cancer avoid in pregnancy

166

Subgroup Senna stimulant laxative Antihistamines Cetirizine antihistami ne Chlorphenamine antihistami ne

Dose 2-4 tablets 5mg 10mg 4mg

Fre q od bd od 46hr

Rout e po po po

24h max

bradycardia, AV block, confusion, depression, halluc. esp. in elderly/ v. ill, hypersensivity reactions, blood disorders, skin reactions. Common side effects Contraindications abdominal cramp; diarrhoea, hypokalaemia intestinal obstruction, shortly after bowel anastomosis. see anti-histamines above. Ceterizine is non-sedating see anti-histamines above.

Promethazine

sedating antihistami ne may be used short term for insomnia

10-20mg 25-50mg 25-50mg

tds

po IM IV

see anti-histamines above. Ceterizine is non-sedating 24mg see anti-histamines above; also exfoliative dermatitis and tinnitus reported; injections may cause transient hypotension or CNS stimulation and may be irritant see anti-histamines above; 100mg avoid extravasation with 100mg intravenous injection; severe coronary artery disease

see anti-histamines above

Cardiovascular Aspirin antiplatlet

75150 mg

od

po

Atorvastatin statin

10-80mg

od

po

300mg bronchospasm; gastrointestinal irritation, gastrointestinal haemorrhage (occasionally major), also other haemorrhage (e.g. subconjunctival) 80mg statins generally; rare: muscular effects, eg myositis, rhabdomyolysis. also GI disturbance, pancreatitis. LFTs, hepatitis, jaundice. Sleep disturbance, headache, dizziness, depression, paraesthesia, asthenia, peripheral neuropathy,

<16 years unless antiplatlet (Reye's syndrome), active peptic ulceration, haemophilia and other bleeding disorders, Hypersensitivity reactions to NSAIDs, breastfeeding Caution in: Hypothyroidism, history of liver disease, high alcohol intake. Risk factors for myopathy or rhabdomyolysis. Avoid in acute porphyria but rosuvastatin is thought to be safe. Avoid in pregnancy as congenital anomalies have been reported and the decreased synthesis of cholesterol 167

Clopidogrel antiplatelet

75300mg

od

po

Dipyridamole antiplatelet

300-

t/qd

po

amnesia, fatigue, sexual dysfunction, thrombocytopenia, arthralgia, visual disturbance, alopecia, and hypersensitivity reactions. Interstitial lung disease; chest pain; back pain; anorexia, malaise, weight gain, hypoglycaemia, hyperglycaemia, tinnitus; rarely cholestatic jaundice, peripheral oedema; very rarely taste disturbances, gynaecomastia, hearing loss, Stevens-Johnson Syndrome, and toxic epidermal necrolysis 300mg dyspepsia, abdominal pain, diarrhoea; bleeding disorders; less commonly nausea, vomiting, gastritis, flatulence, constipation, gastric and duodenal ulcers, headache, dizziness, paraesthesia, leucopenia, decreased platelets (very rarely severe thrombocytopenia), eosinophilia, rash, and pruritus; rarely vertigo; very rarely colitis, pancreatitis, hepatitis, acute liver failure, vasculitis, confusion, hallucinations, taste disturbance, stomatitis, bronchospasm, interstitial pneumonitis, blood disorders (including thrombocytopenic purpura, agranulocytosis and pancytopenia), and hypersensitivity-like reactions 600mg GI effects, dizziness, myalgia,

possibly affects fetal development. Avoid in breast-feeding

Cautions: increased risk of bleeding from trauma, surgery or other pathological conditions; concomitant use of drugs that increase risk of bleeding; Contra-indications: active bleeding, hepatic impairment, severe hepatic impairment, pregnancy

Cautions: rapidly worsening angina, 168

600mg

Simvastatin statin

10-80

noct

po

80mg

throbbing headache, hypotension, hot flushes and tachycardia; worsening symptoms of coronary heart disease; hypersensitivity reactions such as rash, urticaria, severe bronchospasm and angioedema; increased bleeding during or after surgery; thrombocytopenia reported see statins above also rarely anaemia

aortic stenosis, recent MI, left ventricular outflow obstruction, heart failure; may exacerbate migraine; hypotension; myasthenia gravis; coagulation disorders; concomitant use of drugs that increase risk of bleeding

see statins above

Subgroup Respiratory Beclometasone corticostero id

Dose 200400mcg

Fre q bd

Rout e inh

24h max 800mc g

Common side effects Prolonged high doses: adrenal suppression. Lower RTIs. bone mineral density, osteoporosis, growth restriction, glaucoma, cataracts, hoarseness, candidiasi, hypersensitivity reactions, Very rarely paradoxical bronchospasm, anxiety, depression, sleep disturbances, hyperactivity and irritability. Nausea, vomiting, flushing, agitation, anxiety, fear; transient BP & heart-rate; very rarely convulsions (particularly in those with epilepsy), hypersensitivity reactions including

Contraindications ?

Flumazenil benzodiaze p-ine antagonist

100600mcg

IV

1mg

Cautions: short-acting (repeat doses may be necessary, benzo withdrawal symptoms. Caution in prolonged benzodiazepine therapy for epilepsy (risk of convulsions); history of panic disorders, following major surgery; head injury; elderly; 169

Hydrocortisone steroid Ipratropium antimuscari nic bronchodila tor

100mg 2040mcg 40mcg 250500mcg

qd t/qd t/qd prn

IV a inh p inh neb 160mc g 320mc g 1mg

Naloxone opioid antagonist

0.42 mg

prn

IV

10mg

Prednisolone bronchodila tor Subgroup Salbutamol b2 agonist

40-50mg

od

po

60mg

anaphylaxis This entry for acute/severe asthma. see steroids above. Dry mouth; less commonly nausea and headache. Constipation, tachycardia, palpitation, paradoxical bronchospasm, urinary retention, blurred vision, angle-closure glaucoma, and hypersensitivity reactions including rash, urticaria, pruritus, and angioedema occur rarely. Nausea and vomiting, Sweating, tachycardia, Hyperventilation, orBP, ventricular arrhythmias, pulmonary oedema This is an emergency dose. see steroids above Common side effects fine tremor, nervous tension, headache, muscle cramps, palpitation. Tachycardia, arrhythmias, peripheral vasodilation, myocardial ischaemia, and disturbances of sleep and behaviour. Paradoxical bronchospasm, urticaria, angioedema, hypotension, and collapse. high doses hypokalaemia. haemorrhage, thrombocytopenia, rarely rebound hyperlipidaemia

children This entry for acute/severe asthma. see steroids above. Caution in patients with prostatic hyperplasia, bladder outflow obstruction, and those susceptible to angle-closure glaucoma.

Caution: physical dependence on opioids; cardiac irritability; naloxone is short-acting see steroids above

Dose 4mg 500mcg 250mcg 1200mcg 2400mcg 2.5-5mg

Fre q t/qd s 4 hrs qds qds qds

Rout e po sc/im iv inh a inh p nebs

24h max 8mg/d 3mg 800mc g 1.6mg 20mg

Contraindications caution in hyperthyroidism, cardiovascular disease, arrhythmias, susceptibility to QT-interval prolongation, and hypertension. caution in diabetesmonitor blood glucose (risk of ketoacidosis, especially when beta2 agonist given intravenously).

Heparins Dalteparin (Fragmin) DVT prophylaxis

2500-

od

SC

5000 u

haemophilia and other haemorrhagic disorders, thrombocytopenia, recent cerebral 170

DVT/PE treatment Unstable angina parenteral anticoagula nts (LMWH) Enoxaparin (Clexane) DVT prophylaxis DVT/PE treatment Unstable angina

5000 units 750018000 units 120 u/kg

od bd

SC SC

by weight 10 000 u bd

following withdrawal, priapism, hyperkalaemia, osteoporosis, alopecia on prolonged use, injection-site reactions, skin necrosis, and hypersensitivity reactions see heparin above

haemorrhage, severe hypertension; peptic ulcer; after major trauma or recent surgery to eye or nervous system; acute bacterial endocarditis; spinal or epidural anaesthesia; hypersensitivity to heparin or to low molecular weight heparins see heparin above see heparin above see heparin above

Tinzaparin (Innohep)

20-40mg or 24000 u 1.5 mg/k g (150 u/kg ) 30 mg / 3000 unit s then 1 mg/kg OR 750mcg/ kg 3500 u

od od

SC SC

4000 u

see heparin above see heparin above

onc e bd bd od

IV

max see heparin above 100mg 7500 mcg 4500 u see heparin above

see heparin above

SC SC

see heparin above

171

System GI

Cardiovasc ular

Type Anatacid H2 Antagonist PPI Anti-diarrhoeal agents Laxatives Loop diuretics Thiazide diuretics Potassium sparing diuretics Beta blocker Calcium antagonist ACE inhibitor AT1 blocker Nitrates Cardiac glycosides Anti-dysrhythmics Anti-platelet agents Thrombolytics Heparins Oral Anticoagulants Statins

Example Gaviscon Ranitidine Omeprazole Loperamide Mesalazine Furosemide Bendroflumethiazin e Spironolactone Atenolol Dilitiazem Ramipril Losartan Isosorbide mononitrate Digoxin Amiodarone Aspirin Clopidogrel Streptokinase Enoxaparin Warfarin Simvastatin Salbutamol Ipratropium bromide Prednisolone

System Endocrinology

Type Insulin Sulphonylureas Biguanides Thyroxine Anti-thyroid drugs Bisphosphates Calcium & Vitamin D

Example Gliclazide Metformin Carbimazole Alendronate

Pain

NSAIDs Opiates

Diclofenac Codeine, Morphine Paracetamol Amoxicillin Cephradine Doxycycline Gentamicin Erythromycin Ciprofloxacin Metronidazole

Antibiotics

Non-opioids Penicillins Cephalosporins Trimethoprim Tetracyclines Aminoglycosides Macrolides Quinolones Anaerobic antimicrobials

Respiratory

Beta 2 agonists Anticholinergics Corticosteroids

172

Oxygen Neurologic al L DOPA/peripheral decarboxylase inhibitors Anticonvulsants

Antiemetics

Phenytoin Carbamazepine Sodium Valproate Lamotrigine Metoclopramide

173

5 BMJ Learning modules


The following learning modules cover topics that have come up in previous years OSCEs.

5.1 Counselling
Smoking cessation - http://tinyurl.com/ycfgveo Complaints procedure - http://tinyurl.com/y9j8x57 Epilepsy - http://tinyurl.com/ykwe5sd Diabetes - http://tinyurl.com/yehvosu Breast Cancer - when to refer - http://tinyurl.com/ye8sp8t - screening - http://tinyurl.com/yd8utxx Post MI management - http://tinyurl.com/ye4kbdz MRSA - http://tinyurl.com/yb5j742 Warfarin - http://tinyurl.com/ydvaf29

5.2 Clinical Skills


Prescribing - http://tinyurl.com/ygw9dll ECG interpretation - diagnostic picture tests - http://tinyurl.com/yfpmpn2 - diagnosis and management - http://tinyurl.com/ybvqhn4 - Atrial Fibrillation - http://tinyurl.com/ybpsgr5 Addisons disease - http://tinyurl.com/ydq4gxl Blood tests - http://tinyurl.com/yeng4ym Needlestick injuries etc - http://tinyurl.com/ydh62y9 Drug errors - http://tinyurl.com/y9tull2 Blood gas interpretation - http://tinyurl.com/ycgl3q2 Fluid management Fluid challenge: http://tinyurl.com/yzw4ges Chest X-Ray interpretation Part 1 - http://tinyurl.com/y8tuy39 Part 2 - http://tinyurl.com/ydf6se4 Renal failure Management of chronic CKD - http://tinyurl.com/yaaehl8 Acute kidney injury - http://tinyurl.com/yavjwjy

5.3 Histories
Thyrotoxicosis Hyper - http://tinyurl.com/y9lctre Hypo - http://tinyurl.com/yb3bryr BPH - http://tinyurl.com/ybfadgf Polymyalgia rheumatica GCA - http://tinyurl.com/y9dpuzh Intermittent claudication - http://tinyurl.com/yc95mwx Asthma Acute - http://tinyurl.com/yhtbn2u Chronic - http://tinyurl.com/ycfl2xu Rheumatoid arthritis - http://tinyurl.com/yza5k2p Back pain Acute - http://tinyurl.com/y8p8kf4 Chronic - http://tinyurl.com/y9agds4 Collapse http://tinyurl.com/yey9879 and http://tinyurl.com/y9tb97f Pseudomembranous colitis c.diff - http://tinyurl.com/y926dqo 174

Jaundice Interpreting LFTs - http://tinyurl.com/ybghwgk and http://tinyurl.com/yck6yre TIA management guidelines - http://tinyurl.com/ykavuf6 (stroke) Chrons - http://tinyurl.com/yfsuu36 IBD management Chrons - http://tinyurl.com/yexq9la Ulcerative Colitis - http://tinyurl.com/yavmy97

5.4 Examinations
Varicose veins - http://tinyurl.com/yczavqf Bronchiectasis COPD - http://tinyurl.com/yk29cmf Pneumonia - http://tinyurl.com/yzfrosw Homonoymous/bitemporal hemianopia - http://tinyurl.com/yeufuj4 Lung cancer - http://tinyurl.com/ybfc3u9 Competence to consent - http://tinyurl.com/ycoy4kj

brian.mcmillan@doctors.org.uk

175

Вам также может понравиться